A small sceptical voice in the lukewarm wilderness

Posted: October 4, 2014 by tallbloke in atmosphere, climate, Education, Ocean dynamics, radiative theory

bare-arsed2No reply from the eminent atmospheric physics professor… any takers?

Dear Professor xxxxxxx,

I am trying to better understand the physics underlying atmospheric science and wondered if you could answer a question for me.

The Modtran model successfully predicts the local temperature throughout the troposphere, but how do we determine the extent to which the level of radiative activity at a particular altitude is the effect of the local temperature at that altitude rather than the cause of it?

I ask because it appears from my limited reading and understanding that the theoretical underpinning of radiative-convective models neglects the effect of sea level pressure on the rate of evaporation, which must surely have a significant effect on the rate at which the ocean is able to shed the energy input to it by solar radiation.

I have been told that since the average insolation to the surface is around 240W/m^2, the highest temperature the surface would be able to reach without back-radiating gases in the atmosphere at altitude would be 255K. But it appears to me that since the Earth is a spinning grey-body with a significant near surface heat capacity, illuminated by a point source from one side at an intensity of 1362W/m^2 at TOA zenith, the average surface temperature would be able to rise considerably higher than 255K . Indeed, it would be forced to do so due to the ocean being forced to rise in temperature to the point where it can radiate, convect and evaporate at the requisite rate for equilibrium.

If my reasoning is correct, then how can we resolve the issue of the degree to which the mass of the atmosphere rather than its optical depth is the cause of the warmth of the Earth’s surface compared to that of the Moon?

Many thanks for any light you can shed on this question for me.

Roger Tattersall
BA(hons) Hist/Phil Sci. R.Eng.

Comments
  1. Will said:

    “Do you really think that electromagnetic radiation in and out determines the whole energy content of this planet? Check the variations in angular momentum. It is obvious from the ice ages that many other processes are in play.”

    For climate purposes we need only consider the amount of energy retained within an atmosphere (including oceans) which is what I mean by system energy content. Variation in energy from internal movement such as geothermal arising from changes in angular momentum might affect the rate at which energy flows through the atmosphere but not the amount of energy that the atmosphere will retain.

    For a given atmospheric mass and given strength of gravitational field then electromagnetic radiation in and out does determine the energy content of the atmosphere because the mass of the atmosphere is the only constraint on energy radiating straight out again after one has accounted for surface absorption of incoming electromagnetic energy via the S-B equation.

    The ice ages have been attributed to Milankovitch cycles which do affect electromagnetic radiation in and out. They are additionally associated with cloudiness and albedo changes but that is another story.

  2. Ben Wouters says:

    wayne says: October 28, 2014 at 3:35 am

    Hello wayne

    read your response re adiabatic lapse rates.
    We’re moving to spend November on Bali. Response next week (internet permitting)

  3. wayne says:

    Stephen: “You can’t do that !!!

    Volume changes constantly being one of the variables tied to other variables by the specific gas constant for our atmosphere.”

    Sigh… keep telling yourself that Stephen if it makes you feel better.

    Yes you can, and I do set it as constant for any one year analysis. You never really read my words Stephen or maybe just don’t understand, not really. I have told you over and over that I am speaking of climate that is averaged over many years and in that context the volume of the atmosphere as a whole or the column volume at one given latitude does not change enough the show up in at least six or seven digits of precision. Such changes are lost in the round off, I use seven digits of precision always.

    I understand, you are an attourney and not a scientist. On the science side we all realize if you add carbon between two oxygen atoms that already exist in our atmosphere you will increase the column mass and the volume but you seem to refuse to pick up a calculator and compute just how much we are speaking of that would affect the Rspecific. But since you will never do that, let me do it for you:

    Rspecific at 400 ppmv of CO2 is assumed to be 287.053 J/kg/K for a base line value. When CO2 gets to 800 ppmv what will be the value of the Rspecific and how much will that affect the temperature?

    Rspecific = 287.053 = 8.31432 J/mol/K / 0.0289644 kg/mol so the increase in the carbon in CO2 will increase the 0.0289644 upward to 0.0289644 + 0.0004 * 0.012011 g/mol = 0.0289692 kg/mol.

    So of course everone realizes that and also you are right, it does affect the Rspecific but by that changing the density and lapse rate also change very slightly. But, it only affects it from 0.0289644 to 0.0289692 which is a ratio of 0.0289692/0.0289644 = 1.000166 portion and the Rspecific is now 287.055 J/kg/K instead of 287.053 J/kg/K . Let’s see now how much that would affect the temperature somewhere vertically in the atmosphere. I have all of those figures and equationsa programmed into a model I can change by changing the input parameters. Well, it does affect the height at a given pressure like the tropopause by about a meter and it does affect the density and the lapse rate but only out in the sixth digit, but wait… the temperature at any point in the atmosphere does not change AT ALL…. get that? Even out to twelve digits of precision.

    See Stephen, without you actually calculating what such changes of adding mass from the C in CO2 you never really understand what is actually occuring to long term averaged climate.

    So rest easy my friend, adding mass from the carbon in CO2 makes temperatures change by ZERO. One more time through so you can hopefully understand. Adding mass from carbon taken from the ground and placing it between two oxygen atomes in the atmosphere does not affect the mass of the Earth AT ALL!. Gravity acceleraction changes by ZERO! The radius of the Earth changes by less than a micron so lets say that is also zero. The molecular mass changes but that likewise changes the Rspecifc that likewise changes the hydrostatic constant and the constant volume specific heat which all occur to CANCEL any changes in that polytropic exponent I keep speaking of that dictates it all, the log(P/P0)/log(T/T0). So… when you feed all of this back down the chain of equations that perfectly describe Earth’s, Venus’s, Jupiter’s and Titan’s thick troposphere’s pressure/temperature profiles you will get ZERO change from such a factor of mass in the temperatures at any and all altitude levels.

    Just don’t go off and claim all that I’ve pured into this very topic as your own. You have a bad habit of doing that.

  4. wayne,

    I did say that the mass changes that occur from changes in CO2 are too small to affect the usefulness of the standard atmosphere.

    You have shown it nicely with your above calculation. I agree that the associated volume change from adding CO2 is insignificant but it isn’t a constant.

    My point, however, is that it would only be the addition of more mass from CO2 in large enough amounts to alter the Rspecific of the entire atmosphere that would alter the thermal structure of the atmosphere at a given strength of gravitational field and intensity of insolation.

    That is clearly impossible at the tiny level of CO2 in our atmosphere which is a point I have made many times in the past. That is one of my fundamental objections to AGW theory and has been clearly expressed by me in multiple locations since I first became involved in about 2007.

    You seem to agree with that so I don’t understand your somewhat emotional tone.

    Therefore, radiative characteristics must have no effect and you seem to agree with that too.

    I am not claiming that all the work you have done is my own. I am suggesting that all your work is going to boil down to the same conclusion that I reached by other means some time ago.

    I’d be grateful if you would cease trying to discredit me on the basis of my day job. I have been an avid weather and climate enthusiast for 60 years and have honed my multidisciplinary approach throughout that time.

  5. wayne,

    My article about the gas constant includes this:

    “our CO2 emissions would give rise to circulation changes affecting regional and local climates but not average global surface temperatures and the scale of their effect would be insignificant since our emissions comprise such a small portion of atmospheric mass which is the primary determinant of surface temperature.”

    Which you appear to agree with.

  6. wayne says:

    Stephen:
    I see your are assuming that I agree with all you say and that many times is not true. You should ask me instead. Like here I have to disagree with you again:

    S: “My point, however, is that it would only be the addition of more mass from CO2 in large enough amounts to alter the Rspecific of the entire atmosphere that would alter the thermal structure of the atmosphere at a given strength of gravitational field and intensity of insolation.”
    What this is showing is absolutely no amount that you add, not even “it would only be the addition of more mass from CO2 in large enough amounts” as you stated it. This is showing at this point that the differential in temperature is zero, not just small.

    S: “Therefore, radiative characteristics must have no effect and you seem to agree with that too.”

    If you see me agreeing with that statement you are quite wrong. Radiation has everything to do with this subject of climate, read R&C, it is just that the lower troposphere where a linear lapse exist are not affected by radiation, at all.

    I am really glad you had this all figured out in your mind back in 2007. As I have said before, put some numbers and equations together and publish it so you can take some credit from the scientific community.

  7. Stephen Wilde says: October 30, 2014 at 7:05 am

    (Will said: “Do you really think that electromagnetic radiation in and out determines the whole energy content of this planet? Check the variations in angular momentum. It is obvious from the ice ages that many other processes are in play.”)

    “For climate purposes we need only consider the amount of energy retained within an atmosphere (including oceans) which is what I mean by system energy content. Variation in energy from internal movement such as geothermal arising from changes in angular momentum might affect the rate at which energy flows through the atmosphere but not the amount of energy that the atmosphere will retain.”

    So you claim, with no justification, just like the Climate Clowns. All of the energy in the atmosphere comes from H2O with an average water column of 24 mm at 2400 J/gm. Check that for total atmospheric energy. Have you checked the variations in angular momentum for comparison?

    “For a given atmospheric mass and given strength of gravitational field then electromagnetic radiation in and out does determine the energy content of the atmosphere because the mass of the atmosphere is the only constraint on energy radiating straight out again after one has accounted for surface absorption of incoming electromagnetic energy via the S-B equation.”

    You are claiming, just like the Climate Clowns, that the atmosphere is some restraint on EMR. It is not such a constraint ! The atmosphere is a better radiator to space than the surface can be, all adjustable by the amount of WV. The S-B equation used for a planet with an atmosphere, is but yet another lie by the ClimAstrologists! The amount of sensible heat absorbed and dispatched from the surface can be measured but is not because this would distroy the ClimAstrology fixation radiation. None of the latent heat of evaporation is ever part of the surface sensible heat and none is returned to the surface as you mistakenly claim.

  8. wayne says that altering the average Rspecific of an atmosphere by altering the average molecular weight of molecules within the atmosphere has zero effect on temperature.

    I must respectfully disagree since the weight of an atmosphere is linked to the total amount of energy that the atmosphere needs to retain in order to hold it off the ground at a given height and that total amount of energy is relevant to surface temperature.

    Will says that the mass of an atmosphere has zero effect on the free flow of electromagneic energy through it and thus zero effect on temperature.

    I must respectfully disagree because conduction and convection within an atmosphere (which do not need water vapour to occur) operate as an impediment to the free flow of electromagnetic energy and thus raise surface temperature.

    At least we have identified the specific points of disagreement at last.

  9. wayne says:

    Stephen, please one point at a time.

    You say that Will Janoschka said: “Will says that the mass of an atmosphere has zero effect on the free flow of electromagneic energy through it and thus zero effect on temperature.”

    First I can not imagine Will Janoschka saying such a thing so probably you must have misunderstood him in some manner or other but you should know better yourself.

    All mass impedes EMR. If you would the time to read at least some of Max Planck’s “Theory of Heat Radiation” (http://www.gutenberg.org/ebooks/40030) you would find Planck himself even says much on this subject and near page 10: “We only take account of the fact that every ray on its path through any medium loses a certain fraction of its intensity.” He is speak of mass and mass absolutely does impede EMR.

    The more mass EMR passes through the greater the scattering and absorption… all frequencies. All astronomy texts speak of this same effect of scattering and absorption of sunlight. Same for IR and even more so that SW. I am sorry you have ended up believing in such a concept that the matter of gases does not affect EMR itself therefore temperature. Might also see Beer-Lambert law, density and the length of the path.

  10. wayne,

    Read Will’s post as follows:

    “You are claiming, just like the Climate Clowns, that the atmosphere is some restraint on EMR. It is not such a constraint ! The atmosphere is a better radiator to space than the surface can be, all adjustable by the amount of WV.”

    I agree with you on this point. The more mass EMR passes through the greater the scattering and absorption.

    Will is suggesting that an atmosphere accelerates energy through faster than it would leave from a surface beneath a vacuum which is clearly false.

  11. Stephen Wilde says: October 31, 2014 at 11:42 am

    “wayne,Read Will’s post as follows:

    (“You are claiming, just like the Climate Clowns, that the atmosphere is some restraint on EMR. It is not such a constraint ! The atmosphere is a better radiator to space than the surface can be, all adjustable by the amount of WV.”)

    “I agree with you on this point. The more mass EMR passes through the greater the scattering and absorption. Will is suggesting that an atmosphere accelerates energy through faster than it would leave from a surface beneath a vacuum which is clearly false.”

    Please read what Im wrote! I am claiming that because of Kirchhoff’s law of mass at radiative equalibrium there is no restraint on EMR. The limitation on surface exitance comes from the opoposing radiance of atmospheric WV not from absorption as you mistatenly claim. In addittion to not impeding the flux from a higher temperature source nomattern how small that mat be because of opposing radiance the WV in the atmosphere also radiates to space an additional flux because all points in the troposphere, if not the whole atmosphere, are at a higher temperature than they would be from radiative flux alone because of the additional sensible heat added to the atmosphere by thermals and the conversion of latent heat to sensible heat within the atmosphere.
    this additional heat energy plus the greater radiating solid angle of the cross-sectional area of the atmosphere forces the atmosphere to radiate more exitance to space at a lower thempeature than the surface can possibly radiate to space. The atmosphere replaces the surface with a more efficent electromagnetic radiative flux.

    wayne says: October 31, 2014 at 9:40 am

    “Stephen, please one point at a time.”

    ‘You say that Will Janoschka said: (“Will says that the mass of an atmosphere has zero effect on the free flow of electromagneic energy through it and thus zero effect on temperature.”)

    “First I can not imagine Will Janoschka saying such a thing so probably you must have misunderstood him in some manner or other but you should know better yourself.”

    I hope the above explains the point I was making about “this” atmosphere. Other atmospheres may do something similar, I have not measured the others.

    “All mass impedes EMR. If you would the time to read at least some of Max Planck’s “Theory of Heat Radiation” (http://www.gutenberg.org/ebooks/40030) you would find Planck himself even says much on this subject and near page 10: “We only take account of the fact that every ray on its path through any medium loses a certain fraction of its intensity.” He is speak of mass and mass absolutely does impede EMR. The more mass EMR passes through the greater the scattering and absorption… all frequencies.”

    Planck and Maxwell are correct about absorption and scattering of any matter not in radiative equilibrium. Rapid amplitude modulation of a thermal source enforces a condition of non-equilibrium and would result in such signal loss. Forward scattering to a lower temperature does not reduce flux in the direction of lower temperature. Back scattering does not happen as it is apparent reflection and becomes part of any opposing radiance or field strength. Absorption must result in a change in sensible heat (temperature) of that atmospheric mass resulting in non-equilibrium. As per Kirchhoff’s laws, at equilibrium no such “flux” is interfered with even if that mass is totally opaque at that frequency. All absorbing mass in the troposphere radiates more flux to space than is required for radiative equilibrium, by other transfer. The atmosphere is at a higher temperature than required by radiative equilibrium. The big difference is that fluctuation (signal) is attenuated but flux is not. That is how badly the ClimAstrologists screwed up. Flux is limited by an opposing field strength. Tree leaves work a bit different, as they dissipate EMR absorption via latent heat rather than increasing in temperature.

  12. wayne says:

    Right Will, I understand perfectly well what you are saying. That is why I knew it was Stephen misunderstanding once again.

    That is the same reason in the lower troposphere here in our atmosphere (from an averaged viewpoint Stephen, averaged over long periods, climate scale) from the surface up to about 11,000 meters is in radiative equilibrium, if that were not so you would see a warp in the enviromental lapse and you don’t, it is nearly perfectly linear. What Will is saying, or the way I see what Will is saying, is that just because that large portion 0 to 11km is in radiative equilibrium with itself that does not mean there is zero EMR flowing through that portion, and that small flux from the surface to space (you will recognize if as 63Wm-2 from TFK Stephen), has no resistance or attenuation for that small flux.

    Am I reading that correct Will?

  13. wayne says: October 31, 2014 at 10:49 pm

    “Right Will, I understand perfectly well what you are saying. That is why I knew it was Stephen misunderstanding once again.”
    “That is the same reason in the lower troposphere here in our atmosphere (from an averaged viewpoint Stephen, averaged over long periods, climate scale) from the surface up to about 11,000 meters is in radiative equilibrium, if that were not so you would see a warp in the enviromental lapse and you don’t, it is nearly perfectly linear. What Will is saying, or the way I see what Will is saying, is that just because that large portion 0 to 11km is in radiative equilibrium with itself that does not mean there is zero EMR flowing through that portion, and that small flux from the surface to space (you will recognize if as 63Wm-2 from TFK Stephen), has no resistance or attenuation for that small flux. Am I reading that correct Will?”

    Pretty close Wayne, I must have some escape hatches! As best I can tell, the average is 32 W/m^2 of exitance from the surface. 13 W to space and another 19 W to the 2/3 cloud cover. All in the 8- 13.5 micron window. At all other wavelengths the total is less than 100mW/m^2 from the surface, due to the high opposing WV radiance within 10 meters of the surface. This same atmospheric WV can and does provide all needed thermal exitance to space from every altitude. Exit flux accumulates all the way to 220 Km altitude. Space has little radiance! What a wonderful planet!

  14. I don’t think either of you needs to go to such lengths to explain the lapse rate.

    The reason radiation in the troposphere does not affect the lapse rate is because convection causes kinetic energy to convert to gravitational potential energy with height and that convection varies so as to retain radiative equilibrium in the troposphere.

    You don’t need to consider net radiative fluxes at all yet both of your are referring to opposing radiances which is where the AGW proponents also go wrong.

    The radiative loss to space from GHGs including water vapour simply reduces the reconversion of GPE to KE on the descent but that is offset by DWIR for a zero net change in surface temperature.That is a clear and simple solution which integrates both radiative and non radiative processes.

    The situation is different above the troposphere because ozone in the stratosphere responds directly to incoming solar radiation but even there convection does occur to keep the reversed lapse rate in the stratosphere from destroying overall radiative eqilibrium.

    By trying to square the radiative fluxes with the lapse rate slope in the way you do it is you two who are effectively accepting the basic radiative premises of AGW theory.

    If you understood my hypothesis you would see that the exchange of energy via non radiative processes between atmosphere and surface is in balance whilst the exchange of radiative energy between the planet and space is in balance and convection switches the flows between non radiative and radiative processes to maintain that stability.

    In efffect, incoming solar energy gets a free pass right through the system.

    By trying to explain the lapse rate in exclusively radiative terms you two are having to resort to the highly convoluted verbal confusions shown in your recent posts.

  15. wayne says:

    Stephen, you are still not separating macroscale kinetic and graviational potential energy with microscopic molecular kinetic energy. Please don’t try to explain how that is not correct and just sit in on some base level physics courses, they are free on the internet.

    I do understand your hypothesis… trouble is it is not even physically correct. Take some courses Stephen, find out where you are not correct yourself, prove yourself wrong as Feynman insists and be honest enough to admit it when you find your flaws

    “In efffect, incoming solar energy gets a free pass right through the system.”.

    NO! No free pass! Right at 1/3 of clear dry day zenith solar radiation never even makes it to the surface,

    “… and convection switches the flows between non radiative and radiative processes to maintain that stability.”

    NO! Not convection! The switch is due to the thinning of the spectal lines by decreases in the pressure broadening. Take some courses please Stephen, they are free or just stop typing for just a little while and read, there are libraries.

  16. “NO! No free pass! Right at 1/3 of clear dry day zenith solar radiation never even makes it to the surface,”

    Not relevant. Any energy absorbed by the atmosphere forms part of the energy exchange between atmosphere and surface. It doesn’t need to reach the surface in order to participate because once in the atmosphere it adds to the potential adiabatic warming on descent.

    That exchange is in balance which leaves solar in and longwave out also in balance which is effectively a flow of solar energy straight through which is what I mean by a free pass.

    “NO! Not convection! The switch is due to the thinning of the spectal lines by decreases in the pressure broadening”

    If the latter creates an imbalance the former throws a switch to correct it. Forget the maths and consider the concepts.

    “you are still not separating macroscale kinetic and gravitational potential energy with microscopic molecular kinetic energy”

    I am.

    The macroscopic is not what matters in creation of the lapse rate. It is the microscopic moleculsar energy that switches between KE and GPE. As a molecule rises it vibrates less and the loss of KE through reduced vibrsation becomes GPE. Reverse that when it falls.

    I think your facility with words and concepts is worse than my facility with the maths. At least I understand the broad principles behind the maths IMHO.

    Not much point going any further here.

  17. Stephen Wilde says: November 1, 2014 at 8:13 am

    “I don’t think either of you needs to go to such lengths to explain the lapse rate.”

    Only you try to explain lapse rate, the rest accept the gravitational potential.

    “The reason radiation in the troposphere does not affect the lapse rate is because convection causes kinetic energy to convert to gravitational potential energy with height and that convection varies so as to retain radiative equilibrium in the troposphere.”

    Would you like to demonstrate the conversion of sensible or latent heat to gravitatuonal potential energy? Why does not a thermos of coffee in an aircraft not do that? Only momentum is converted, never heat, heat has no momentum. All would love to discover thermal inertia, then we could observe temperature oscillating about equilibrium..

    “You don’t need to consider net radiative fluxes at all yet both of your are referring to opposing radiances which is where the AGW proponents also go wrong.”

    Bullshit! You refuse to understand the difference between radiance and radiative flux. You still have radiative flux going in a direction of higher radiance, Just like the ClimAstrologists.

    Stephen Wilde says: November 1, 2014 at 4:29 pm

    “Not much point going any further here.”

    I certainly hope you mean that, warmist!

  18. Ben Wouters says:

    Stephen Wilde says: November 1, 2014 at 4:29 pm

    ” It is the microscopic moleculsar energy that switches between KE and GPE. As a molecule rises it vibrates less and the loss of KE through reduced vibrsation becomes GPE. Reverse that when it falls.”
    What does a molecule do to “gain altitude” ? A bird flaps his wings, an airplane burns fuel to run its engines.
    There is no CONVERSION of KE into PE for a rising parcel of air.
    Pse get to grips with the basics of convection.

  19. Ben Wouters says:

    wayne says: October 28, 2014 at 3:35 am

    ” Ben: “Do you really agree that the adiabatic lapse rates have only significance for the temperature change WITHIN rising or sinking air?”

    At first I was going to pick some words, line adiabatic lapse rates being plural, but after re-reading your question a few times.. yes, I absolutely agree as long as that doesn’t include any state changing or radiative effects which are what make the entire process not adiabatic but polytropic.”

    I’m on a kind of “operational” internet connection, so I’ll give it a try.
    Lapse rateS (plural) is for the two different adiabatic lapse rates, dry and saturated. (DALR and SALR)
    The difference DOES include the state change from wv to waterdroplets (condensation), all happening within a volume of rising air. The latent heat that is released during condensation is exactly what makes the difference between the dry and saturated adiabatic lapse rates.

    If we can agree on this, I propose to leave convection out of the discussion, and concentrate on the mechanism for the average ELR. Once we have an understanding for that it is simpler to discuss convection, the temperature inversion in the stratosphere etc.etc.

  20. “What does a molecule do to “gain altitude” ? A bird flaps his wings, an airplane burns fuel to run its engines.There is no CONVERSION of KE into PE for a rising parcel of air.
    Pse get to grips with the basics of convection.”

    A group of molecules acquire more energy from an unevenly heated surface than surrounding molecules, becomes lighter and starts to rise.

    “In meteorology, convective available potential energy (CAPE),[1] sometimes, simply, available potential energy (APE), is the amount of energy a parcel of air would have if lifted a certain distance vertically through the atmosphere.”

    from here:

    http://en.wikipedia.org/wiki/Convective_available_potential_energy

    That CAPE is drawn from KE during the process of uplift (conservation of energy) which is why the molecules in the parcel vibrate less and cool.

  21. I think the problem here is that you chaps don’t know any meteorology which is a specialist discipline that I have been studying for 60 years.

    It is the study of how the underlying physics actually plays out within a planetary atmosphere and it plays out in unexpected ways.

    Until you get a grip on meteorology your knowlegde of physics and maths won’t enable you to see the validity of my comments.

    I notice that each of you has a good grip on certain relevant areas of knowledge but due to your lack of meteorological knowledge you are not able to link it all together in a multidisciplinary fashion which is what is required for any sort of conceptual understanding of weather and climate.

    I recommend that you all go and read some meteorology text books and put the physics and maths on the back burner for a while.

  22. Ben Wouters says:

    Stephen Wilde says: November 2, 2014 at 10:16 am

    ” I think the problem here is that you chaps don’t know any meteorology which is a specialist discipline that I have been studying for 60 years.”

    Sorry, but if 60 years study still doesn’t give a clue about how convection actually works, I suggest you switch to another field or hobby.

    ” That CAPE is drawn from KE during the process of uplift (conservation of energy) which is why the molecules in the parcel vibrate less and cool.”
    CAPE is the BUOYANCY POTENTIAL. (chance that buoyancy will develop and how ‘ violent’ it will be)
    Has nothing to do with KE conversion into PE or similar.

    I do agree that a lot of posters here could do with at least a basic understanding of meteorology.

  23. Stephen Wilde says: November 1, 2014 at 4:29 pm

    “Not much point going any further here.”

    Stephen Wilde says: November 2, 2014 at 10:16 am

    “I think the problem here is that you chaps don’t know any meteorology which is a specialist discipline that I have been studying for 60 years. It is the study of how the underlying physics actually plays out within a planetary atmosphere and it plays out in unexpected ways. Until you get a grip on meteorology your knowledge of physics and maths won’t enable you to see the validity of my comments.”

    Please try to make some comments that would be considered valid by others. You study what experience do you have? Meteorologists do a good job at what? Are you claiming that meteorologists know about physics? Here I thought you study meteors! You have not demonstrated any knowledge of engineering or hard sciences.

    “I notice that each of you has a good grip on certain relevant areas of knowledge but due to your lack of meteorological knowledge you are not able to link it all together in a multidisciplinary fashion which is what is required for any sort of conceptual understanding of weather and climate. I recommend that you all go and read some meteorology text books and put the physics and maths on the back burner for a while.”

    How about those that would not touch your version of your science, with your ten foot pole?
    Your type of “conceptual understanding of weather and climate” is not desired. The patterns and movements seem to be somewhat deterministic and perhaps even useful some day, but seem to have nothing to do with the states and processes that determine what the atmosphere is and does. Most of your claims parrot those of ClimAstrologists, who also study only meteors, with help of a playstation-64.

  24. Ben,

    CAPE is defined thus in that link:

    “the amount of energy a parcel of air would have if lifted a certain distance vertically through the atmosphere.”

    i.e. it is the energy contained by the parcel in potential form AFTER it has been uplifted.

    It has to come from somewhere (conservation of energy) so it comes from KE which is turned into CAPE during the uplift process which involves work being done against gravity.

    It is a micro process at the molecular scale.

    As pressure reduces during uplift the molecules vibrate less which means they carry less KE and become cooler. That KE is replaced by CAPE. During the descent work is done with gravity.

    Increasing pressure on the subsequent descent makes the molecules vibrate more and they warm up as CAPE is reconverted back to KE.

    Molecules of gases vibrate more as pressure increases which is why their sensible temperature rises as one approaches the surface beneath the atmosphere.

    That is why wayne’s equations work but he doesn’t understand the concept.

    Will has no idea at all.

  25. Ben Wouters says:

    Stephen Wilde says: November 2, 2014 at 3:23 pm

    Try this link: http://www.tornadochaser.net/cape.html

    CAPE is a measure for convective potential.
    It is just the difference (area) between the ELR and the DALR/SALR.
    In a stable atmosphere CAPE is zero.

  26. Ben Wouters says:

    Suggest all interested to follow the class available in this link:
    http://www.tornadochaser.net/capeclass.html
    so we can finally put all nonsense about the DALR/SALR and adiabatic atmospheres to rest.

  27. Ben, your link says this:

    “More specifically, CAPE represents the amount of buoyant energy available to speed up a parcel vertically, or the amount of work a parcel does on the environment”

    Where do you think CAPE ends up AFTER the uplift has occurred ? You cannot destroy energy.

    It ends up as potential energy in the molecules at the new height and lower temperature as per the definition I referred you to.

    Furthermore it is recoverable on the descent except in so far as it might be radiated to space from GHGs or particulates within the atmosphere.

    The thing is that my comments are consistent with what you and wayne are saying but neither of you can see it.

  28. Stephen Wilde says: November 2, 2014 at 4:08 pm

    “Ben, your link says this:”
    —————————————————————————————————
    “More specifically, CAPE represents the amount of buoyant energy available to speed up a parcel vertically, or the amount of work a parcel does on the environment”
    —————————————————————————————————
    “Where do you think CAPE ends up AFTER the uplift has occurred ? You cannot destroy energy.
    It ends up as potential energy in the molecules at the new height and lower temperature as per the definition I referred you to. Furthermore it is recoverable on the descent except in so far as it might be radiated to space from GHGs or particulates within the atmosphere. The thing is that my comments are consistent with what you and wayne are saying but neither of you can see it.”

    Wow,
    “You cannot destroy energy….. except in so far as it might be radiated to space”

    The Earth’s atmosphere is radiating 10^17 Joules/second to space continuously. Even at 0.5% by mass the latent heat of WV is five times your CAPE at its most violent. More guesses more fantasy! Certainly is consistent with everyone that cannot comprehend your Unicorn!

  29. Kristian says:

    Stephen Wilde says, November 2, 2014 at 4:08 pm:

    “Ben, your link says this:

    “More specifically, CAPE represents the amount of buoyant energy available to speed up a parcel vertically, or the amount of work a parcel does on the environment”

    Where do you think CAPE ends up AFTER the uplift has occurred ? You cannot destroy energy.

    It ends up as potential energy in the molecules at the new height and lower temperature as per the definition I referred you to.”

    No, Stephen! It’s right there in the text you’re quoting, right there in front of you: “(…) or the amount of work a parcel does on the environment.”

    When the parcel does work on its surroundings, expanding into them upon rising, it LOSES energy (it is transferred from the parcel to its surroundings by way of WORK) and accordingly, it COOLS.

    How many times has this basic piece of information on the adiabatic process been provided to you, Stephen? And how many times have you simply ignored it and moved on with your self-invented brand of quasi-physics? This is HOW the adiabatic process works! No energy transferred (cooling/warming) as HEAT, energy transferred (cooling/warming) rather by WORK being done. That’s the whole point!

    Stephen, I’m sorry and with all due respect, but you’re so far off the deep end here it’s frankly embarrassing to behold. I wish, for your own sake, that you would just stop being so pigheaded about this, stop writing for a while, lower your shoulders, go read a book and try to actually comprehend what it’s saying. Learn something about how the atmosphere actually works before flaunting your ignorance of basic physical processes governing the climate system. We all see it, Stephen, and your posts here are only becoming ever more cringeworthy. You have no grasp whatsoever of the concepts at hand, we have all been trying to point this out to you (and point you in the right direction) now for quite some time, but you absolutely refuse to take any of our advice. It’s all just our problem, we apparently don’t understand anything, we don’t understand your genius theory, we don’t understand meteorology, convection, the adiabatic process, we are unable to see the big picture as you do. Once again, Stephen, take a hint: We understand perfectly well what you’re trying to say. We’re not stupid. We can read. That is exactly why we are able to tell you: You’re wrong! Take this from fellow sceptics that do agree with you on many fundamental levels. Please read up, Stephen! You will have to go back to your drawing board on this one …

  30. Kristian,

    Heat is not transferred from a rising parcel to the surroundings during uplift and is not taken from the surroundings by the parcel during descent.

    The work during ascent is not done against the surroundings because no work is required when a parcel moves into a region of lower pressure. The work is done against gravity.

    The work done on descent is not done against the surrounding because the parcel shrinks as it descends. The work is done with gravity.

    Instead the energy changes from KE to PE on ascent and from PE to KE on descent.

    Will,

    Energy converted to PE during uplift cannot be radiated to space because the parcel becomes cooler and lower temperatures radiate less. PE does not radiate, only KE radiates.

    I am not being pigheaded, I am describing established science that is well known in meteorology.

  31. Kristian,

    The term ‘environment’ refers to the gravitational field and NOT surrounding molecules.

  32. Of course no process is perfectly adiabatic. There will always be some work done on the surrounding molecules but by far the majority of the work done is with or against gravity and that portion converts KE to PE and back again without radiation to space occurring whilst the energy is in PE form.

  33. Kristian said:

    “When the parcel does work on its surroundings, expanding into them upon rising, it LOSES energy (it is transferred from the parcel to its surroundings by way of WORK) and accordingly, it COOLS.”

    So the expanding air loses energy and cools?

    Then the surrounding molecules must acquire that energy and warm.

    That is a transfer of energy as heat which is forbidden in an adiabatic process.

    “An adiabatic process is a process that occurs without the transfer of heat or matter between a system and its surroundings”

    Checkmate.

    The work is instead done against gravity which converts KE to PE and the parcel cools though there is always some minor leakage from that adiabatic process.

  34. Ben Wouters says:

    Stephen Wilde says: November 2, 2014 at 4:08 pm

    ” Where do you think CAPE ends up AFTER the uplift has occurred ?”
    After the uplift (IF it occurred) CAPE will still be on the diagram paper it was drawn on, or in the output of the computer program that calculated it.

    Study “hydrothermal balance” and “pressure gradient force” for a basic understanding of convection.
    try http://www.recreationalflying.com/tutorials/meteorology/section1a.html#pressure_gradient
    for an entry level explanation.

    Parcels or molecules don’t flap their wings to gain altitude. They are PUSHED up, not doing any work against gravity themselves.
    If I ride an elevator upward, my PE also increases, without me doing any work against gravity.

  35. Ben Wouters says: November 2, 2014 at 11:55 pm
    Stephen Wilde says: November 2, 2014 at 4:08 pm

    (” Where do you think CAPE ends up AFTER the uplift has occurred ?”)

    “After the uplift (IF it occurred) CAPE will still be on the diagram paper it was drawn on, or in the output of the computer program that calculated it. Parcels or molecules don’t flap their wings to gain altitude. They are PUSHED up, not doing any work against gravity themselves. If I ride an elevator upward, my PE also increases, without me doing any work against gravity.”

    Thank you for your clear explanation of that part of air movement! I apologise for the grief I have caused, sorry! I still have many questions about the speed that the atmosphere can re-thermalise under rapid changing illumination conditions like a Solar eclipse. Seems way to fast for actual atmospheric mass transport, more like the speed of sound. The hydrostatics of a compressible fluid baffle me.

  36. Ben Wouters says:

    Will Janoschka says: November 3, 2014 at 12:51 am

    ” The hydrostatics of a compressible fluid baffle me.”

    Well, that explains why you insult someone in almost every comment you make and why you regard professional meteorologists as babbling idiots.

  37. “Parcels or molecules don’t flap their wings to gain altitude. They are PUSHED up, not doing any work against gravity themselves”.

    When some air parcels or molecules are heated more than others (that adjoin them in the horizontal plane) by conduction from an unevenly heated surface they become lighter and rise spontaneously.

    No PUSHING required.

    Having risen they leave an area of lower pressure beneath and cooler air is PULLED in from the sides at ground level.

    Pure meteorology.

    Get a text book.

    My comments are consistent with the established science of hydrothermal balances and pressure gradient forces.

  38. Kristian says:

    Stephen Wilde says, November 2, 2014 at 9:21 pm:

    “Then the surrounding molecules must acquire that energy and warm.”

    Indeed.

    “That is a transfer of energy as heat which is forbidden in an adiabatic process.

    “An adiabatic process is a process that occurs without the transfer of heat or matter between a system and its surroundings””

    *Sigh*

    Indeed no, Stephen. That is a transfer of energy as WORK (like I said) which is NOT forbidden in an adiabatic process. (Notice? Your quote includes “heat” and “matter” only. Specifically doesn’t include “work”.) ‘Heat’ [Q] and ‘work’ [W] are different modes of energy transfer in a thermodynamic process. Both transfers lead to warming or cooling, because they both change the internal energy [U] of the systems involved:

    http://hyperphysics.phy-astr.gsu.edu/hbase/thermo/heat.html

    Again, how many times has this simple fact of thermodynamics been pointed out to you? And how many times have you just gone lalalalalala and moved on with your silly misconceptions as if they were som new kind of esoteric Truth?

    You need to stop this, Stephen.

  39. Ben Wouters says:

    Stephen Wilde says: November 3, 2014 at 8:13 am

    “When some air parcels or molecules are heated more than others (that adjoin them in the horizontal plane) by conduction from an unevenly heated surface they become lighter and rise spontaneously.

    No PUSHING required.”

    What you call ” rise spontaneously” is actually the pressure gradient force (PGF) pushing the parcel up. If the PGF is higher than the downward force of gravity, the parcel rises, if lower the parcel sinks.
    A rising parcel encounters reducing pressure, so the parcel must expand to equalize the pressure.
    Here comes your Ideal Gas Law: V increases rapidly, so the temperature drops.
    The rate is 9,8K/km: the DALR.
    If condensation occurs, the release of latent heat reduces the temperature drop as long as condensation is happening: the SALR.

    No conversion of KE into PE or other mumbo jumbo inside the parcel occurring.

  40. wildeco2014 says:

    Kristian,

    No work is done by rising molecules on surrounding molecules because the rising molecules are moving into a region of lower pressure.

    That is well established science.

    The work is done against gravity and not against the surrounding molecules.

    Ben,

    I referred you to this :

    “the amount of energy a parcel of air would have if lifted a certain distance vertically through the atmosphere.”

    So:

    What kind of energy does the parcel contain after being lifted higher to a colder location ?

    Here is a clue:

    Convective Available POTENTIAL Energy.

    What part of the word POTENTIAL do you not understand ?

    After uplift you have a colder parcel (less KE) but it contains more PE.

    Conversion of KE to PE is a well established scientific fact.

  41. wildeco2014 says:

    Kristian said:

    “That is a transfer of energy as WORK (like I said) which is NOT forbidden in an adiabatic process”

    Sigh.

    Work would be involved (if your account were correct) but the transfer is of energy as heat (albeit via work) whereby the parcel cools and the surrounding molecules warm. How is that not a transfer of energy as heat?

    To prevent a transfer of energy as heat one must do the work against gravity which does not transfer energy as heat in and out of the parcel. Instead it transforms the energy within the parcel from KE to PE.

    A molecule at the surface contains all KE and no PE. The higher it rises the less KE and the more PE.

    At high levels there is little KE and lots of PE.

  42. Trick says:

    Stephen 8:13am: ”Get a text book.”

    Excellent advice Stephen, you too, a modern one on meteorology. In it you will see that the conserved quantity as the parcel rises is energy of enthalpy = PE + KE + p*V as I’ve pointed out to you many times. Any change in altitude of the parcel requires tracking changes in pressure PLUS change in volume. In basic math that change in p*V becomes d(p*V) = p*dV + V*dp. Texts explain that in detail. You have to earn the pre-req.s in basic calculus to understand, be conversant with the infinitesimal change operator symbol d.

    4:17pm: ”At high levels there is little KE and lots of PE.”

    No kidding. Plus lots of parcel volume and little pressure. They change too. At ground energy of enthalpy in parcel conserved = energy of enthalpy at altitude for no change in conserved parcel enthalpy:

    Ground level enthalpy (~0 + KE + p*V ) = at high levels (PE + ~0 + p*V)

    Read a text book Stephen, learn how to deal with the change operators. The p*V term was added to molecular kinetic theory for a gas think it was around 1868 by J.C. Maxwell, you are just behind in your reading. This term pretty much vanishes in solids and liquids changes which is why the p*V term was not noticed in testing until locomotive & boiler room steam engines et. al. issues needed efficiency improvement & Maxwell reasoned p*V out with balloon testing.

  43. Kristian says:

    wildeco2014 says, November 3, 2014 at 3:57 pm:

    “No work is done by rising molecules on surrounding molecules because the rising molecules are moving into a region of lower pressure.

    That is well established science.”

    It might be well-established on the planet where you live, Stephen. On planet Earth and in the world of regular physics, however, it is well-established science that a volume of air that for some reason expands, does so INTO its surrounding air masses, thus ‘pushing them to the side’ to take their place, doing work on them in the process. (This is how atmospheric circulation (and a normal convection cell) works.) And, in thermodynamics, when you do work on something, you transfer some of your energy to the thing you do work on. The particular kind of thermodynamic work that expanding (rising) air does on its surroundings is called PV work and is fairly basic knowledge for people who at some point in life went to school. No matter how low the surrounding bulk pressure that the expanding volume of air faces, it is still a resisting pressure, an external force that counters the expansion of the volume of air. OF COURSE rising/expanding air does work on its surroundings, Stephen!

    http://en.wikipedia.org/wiki/Work_(thermodynamics)

    “Mechanical thermodynamic work is performed by actions such as compression (…). In the simplest case, for example, there [is] work of change of volume against a resisting pressure (…). Pressure-volume work (or PV work) occurs when the volume V of a system changes. (…) As for all kinds of work, in general PV work is path-dependent and is therefore a thermodynamic process function. The statement that a process is reversible and adiabatic serves as a specification of the path, but does not determine the path uniquely, because the path can include several slow goings backward and forward in volume, as long as there is no transfer of energy as heat. The first law of thermodynamics states dU = δQ – δW [or dU = δQ – PdV]. For an adiabatic process, δQ = 0 and thus the integral amount [of] work done is equal to the change in internal energy. For a reversible adiabatic process, the integral amount of work done during the process depends only on the initial and final states of the process, and is the one and the same for every intermediate path.”

    Stephen, you also say:

    “The work is done against gravity and not against the surrounding molecules.”

    We need to get a few things straight here, it seems. You are thoroughly confused on this matter.

    The adiabatic process is inherently a thermodynamic process, not a Newtonian process. A thermodynamic process specifically involves the transfer of energy in the form of either ‘heat’ [Q], ‘work’ [W], or both. In thermodynamics, ‘heat’ is directly interchangeable with ‘work’, both being dynamic processes, transfers of energy. Refer to the Carnot cycle. A Newtonian process, on the other hand, involves forces, kinetic and potential energy (KE & PE). Thermodynamics basically describes ‘temperature’ and how it changes, while Newtonian mechanics describes ‘motion’ and how it changes. Work is an entity in both areas, but while, in a thermodynamic process, work done can be translated directly into a change in temperature, in a Newtonian process, work done is rather translated directly into a change in motion (speed, KE) or position (PE).

    This is where your “work against gravity” and “KE turned into PE and back” comes in. If you lift a ball up in the air, against gravity, you surely do work on it, and some of its KE is surely turned into PE. But it doesn’t get any cooler just from moving higher, or to any other position for that matter. There is no change in temperature at all specifically from this process. (If there were, the ball should’ve warmed (not cooled) and your hand should’ve cooled, from the direction of work transfer during the lifting. But this is not how Newtonian work functions.) This isn’t a thermodynamic process. It’s a mechanical (Newtonian) process. All that’s changed is the position. The ball has gained some gravitational potential energy: PE_grav = mgh (weight times height). If you then drop the ball, it will accelerate towards the ground. As its speed increases, PE progressively transforms back to KE. The more KE gained, the more work can potentially be done by the ball on the ground when it finally hits it.

    An adiabatic process, Stephen, does not operate like this, through a change from KE to PE and back to KE again. It operates through expansion and compression. That’s how the air cools and warms. The KE & PE thing is indeed a real physical phenomenon, but oh so irrelevant to the adiabatic process.

    Like I pointed out, in thermodynamics, whenever work is done, energy is transferred from one system to another, making the receiving system warmer in the process. The classical way of illustrating this kind of work is the compression of air inside a piston – an adiabatic process. No energy is transferred as ‘heat’ [Q], but the gas inside is still getting warmer. Because work is being done on it. Energy is transferred to it as ‘work’ [W]. This is what an adiabatic process is all about. PV work.

  44. Kristian says:

    wildeco2014 says, November 3, 2014 at 4:17 pm:

    “Work would be involved (if your account were correct) but the transfer is of energy as heat (albeit via work) whereby the parcel cools and the surrounding molecules warm. How is that not a transfer of energy as heat?”

    Holy moly! It’s not because it’s not, Stephen. It’s a transfer of energy as WORK. How many times must this simple fact be repeated before it finally starts sinking in …?

    Go read a book, Stephen. Preferably one on basic thermodynamic principles. Maybe particularly on the adiabatic process.

    Energy is never transferred as ‘heat’ via ‘work’. Where do you come up with these things!? Energy is either transferred as ‘heat’ [Q]. Or as ‘work’ [W]. I repeat: Energy is either transferred as ‘heat’ [Q]. Or as ‘work’ [W]. They are strictly separate, but interchangeable processes that both cause a change in internal energy [U] – and thus a change in T – in the systems involved.

  45. Gases behave differently to solids when pressure changes.

    They vibrate less and cool when pressure falls and vibrate more and warm when pressure increases.

    A rising parcel moving into a region of lower pressure need do no work on the surrounding molecules because it can more easily move higher against gravity.It is the resistance from those surrounding molecules together with lower pressure above that causes further uplift to be ‘preferred’ as against work being done on those molecules.

    The parcel follows the path of least resistance which is upward.

    As a parcel rises it cools at the same rate as the surrounding air and retains the same temperature and density differential as it had at the surface.

    Rising continues indefinitely unless a temperature inversion is reached whereupon the temperatures and densities equalise and rising stops, such as at the tropopause.

    There can be some minor leakage of energy to surrounding molecules because no physical process is perfectly adiabatic but by far the majority of the KE at the surface converts to PE as height is gained.

    Regrettably you are totally confused about adiabatic processes in a convecting atmosphere.

  46. Kristian said:

    “Energy is either transferred as ‘heat’ [Q]. Or as ‘work’ [W]”

    Then how do you propose to make the surrounding molecules warmer whilst the rising molecules become cooler ?

    That is a transfer of energy as heat.

    If work is an alternative to heat then the work is done INSTEAD of heat being transferred but by your account you think work is done AND heat is transferred.

    You can’t have it both ways.

    As I told you, the work is done against gravity and so no heat is transferred because KE turns to PE instead.

    You need to work on your logic skills and your verbal expression of concepts. That would help you to correct your apparent misunderstanding of thermodynamics.

    Actually there is much that we agree on and the same goes for Ben but the two of you need to adjust your understanding of the adiabatic process and then it should fall into place for you.

    Will and Trick are lost causes.

  47. I think this is the nub of the problem:

    “Thermodynamics basically describes ‘temperature’ and how it changes, while Newtonian mechanics describes ‘motion’ and how it changes. Work is an entity in both areas, but while, in a thermodynamic process, work done can be translated directly into a change in temperature, in a Newtonian process, work done is rather translated directly into a change in motion (speed, KE) or position (PE).”

    Gases moving up against gravity are involved in a purely Newtonian process because they are changing position (PE) relative to the gravitational field.

    Since the temperature differential with the surrounding molecules is maintained during uplift (the parcel cools with height at the same rate as the surrounding molecules) there is no thermodynamic exchange between the rising parcel and the surrounding molecules.

    That seems to be where you are going wrong 🙂

  48. Stephen Wilde says:..November 3, 2014 at 9:33 pm

    I think this is the nub of the problem:

    “Thermodynamics basically describes ‘temperature’ and how it changes, while Newtonian mechanics describes ‘motion’ and how it changes. Work is an entity in both areas, but while, in a thermodynamic process, work done can be translated directly into a change in temperature, in a Newtonian process, work done is rather translated directly into a change in motion (speed, KE) or position (PE).”

    OK, but do not confuse the two. Although heat is considered “kinetic energy” by some, it is never Newtonian kinetic energy, that involves both mass and inertia but never temperature.

    “Gases moving up against gravity are involved in a purely Newtonian process because they are changing position (PE) relative to the gravitational field.”

    Yes but only the mass of that gas, never its specific heat, sensible heat or temperature!

    “Since the temperature differential with the surrounding molecules is maintained during uplift (the parcel cools with height at the same rate as the surrounding molecules) there is no thermodynamic exchange between the rising parcel and the surrounding molecules.”

    That is precisely correct! In that uplift or down trend there is no “work”, and no change in entropy..
    The process is adiabatic, isentropic, and reversible. There is simply no exchange of thermal energy between masses, even with an identifiable change in temperature and pressure. All else is but confusion promoted by ClimAstroligists!

    Yes Stephen, That seems to be where you are going wrong 🙂 There is no identifiable thermodynamic “process” except by water vapor, and the radiative exitance of entropy to space.

  49. Will,

    I was quoting Kristian. He thinks adiabatic uplift involves a thermodynamic exchange between the rising parcel and surrounding molecules. I was trying to explain why that is not right to any significant degree.

    There is no work (or negligible work) against surrounding particles but there is work against gravity on the ascent and with gravity on the descent contrary to what you seem to be saying.

    That makes all the difference.

    Where I also depart from you is that the process occurs even without water vapour simply from density variations in the horizontal plane caused by uneven surface heating.

    Water vapour being a GHG and producing a condensate does allow leakage of radiation to space from within the adiabatic exchange so you are correct to say that it does introduce a thermodynamic element.

  50. Trick says:

    Stephen 9:06pm: “Gases behave differently to solids when pressure changes.”

    Not in regards enthalpy as shown in text books; for solid, liquid, gas parcel conserved quantity enthalpy = KE + PE + p*V.

    p*V changes for liquid and solid are so small they were not noticed for gas until ~1868 as Maxwell demonstrated first by intuition then test to scientific method.

  51. Trick,

    It is the scale of the gas response to pressure and volume changes as compared to that of solids and liquids which matters, as you seem to agree.

    Due to the amount of space between molecules and their relatively free movement the issue of KE and PE becomes far more significant when gas moves up or down within a gravitational field.

    Your point is correct but trivial.

  52. Kristian says:

    Stephen Wilde says, November 4, 2014 at 12:16 am:

    “He thinks adiabatic uplift involves a thermodynamic exchange between the rising parcel and surrounding molecules.”

    I don’t think it, Stephen. That’s how a rising parcel of air interacts with the surrounding air, the air into which it moves (rises and expands). I’m telling you. But you absolutely refuse to even read the links and quotes I provide you with. Hence the ongoing confusion.

    Other than that, the process of uplift itself is not an adiabatic one. The expansion/compression of the air parcel lifting/sinking is the adiabatic part. That’s what we’re talking about. So all you need to do, Stephen, is to read about ‘The Adiabatic Process’ and realise what basic physical mechanism leads to cooling/warming of the air involved.

    “There is no work (or negligible work) against surrounding particles but there is work against gravity on the ascent and with gravity on the descent contrary to what you seem to be saying.

    That makes all the difference.”

    No, what makes the difference, Stephen, is that you apparently at all costs avoid reading passages/paragraphs/links/quotes that might counter your belief system about this. That is disconcerting.

  53. Kristian says:

    Stephen Wilde says, November 3, 2014 at 9:33 pm:

    “Since the temperature differential with the surrounding molecules is maintained during uplift (the parcel cools with height at the same rate as the surrounding molecules) there is no thermodynamic exchange between the rising parcel and the surrounding molecules.

    That seems to be where you are going wrong 🙂 “

    No, this is where you’re absolutely blind to basic thermodynamic concepts. And you don’t read what I’m trying to say to you about it. About ‘heat’ [Q] and ‘work’ [W] and ‘internal energy’ [U]. You still don’t get it.

    Once more:

    YES, THERE IS A THERMODYNAMIC EXCHANGE OF ENERGY BETWEEN THE EXPANDING PARCEL AND THE SURROUNDING AIR. ONLY NOT AS ‘HEAT’ (HENCE ‘a-diabatic’), ONLY AS ‘WORK’.

    Again, read up on the adiabatic process. A transfer of energy as ‘work’ [W] ALSO changes temperature, Stephen, because it changes ‘internal energy’ [U]. Dear God!

  54. Ben Wouters says:

    wildeco2014 says: November 3, 2014 at 3:57 pm

    “What part of the word POTENTIAL do you not understand ?”
    Being a non native English speaker it seems I understand the word potential better than you do.
    Potential means ‘currently unrealized ability’.
    In the case of CAPE we’re looking at the as yet unrealized POTENTIAL for convection, used in forecasting. Read the whole link you provided.

  55. Trick says:

    Stephen 8:41am: “Your point is correct but trivial.”

    Correct and NON-TRIVIAL p*V change with z for gas! That’s my whole point Stephen – there is only trivial enthalpy change for total energy in a block of steel elevated 0 agl to orbit of CERES as steel internal p&V negligible change, but p&V hugely change & NOT negligible total energy for a gas parcel elevated similarly. Missing this p*V term causes Stephen to draw nonsensical conclusions obvious to anyone who HAS read a modern text chapter dealing with gas parcels in meteorology.

    If p*V component of enthalpy were trivial for gas state, Maxwell’s ~1868 paper advancing kinetic theory above billiards would have been trivial. That paper was NOT trivial. The conclusions were useful to meteorology. Read up Stephen.

  56. Ben Wouters says:

    Stephen Wilde says: November 3, 2014 at 9:33 pm

    ” Since the temperature differential with the surrounding molecules is maintained during uplift (the parcel cools with height at the same rate as the surrounding molecules)”

    During the 60 years you studied meteo have you ever looked at an actual temperature profile of the atmosphere as measured twice daily around the world?
    Noticed they can be all over the place?
    A rising parcel only rises as long as its temperature is higher than that of the surrounding air.
    Its internal temperature cools according the DALR or the SALR after condensation starts.
    So most of the time convection is not happening at all, or stops shortly after it started because the ELR has a stable profile (CAPE = 0 )

  57. From the above links:

    “Adiabatic process – Expansional cooling or compressional warming of air parcels in which there is no net heat exchange between the air parcels and the surrounding (ambient) air. >>>”

    Kristian insists that there is a net heat exchange.

  58. wayne says:

    Stephen Wilde says: Kristian insists that there is a net heat exchange.

    Now that is a bald face lie Stephen, apologize to Kristian! He just said exactly the opposite this morning to you.

    Kristian said to you Stephen:

    Once more:

    YES, THERE IS A THERMODYNAMIC EXCHANGE OF ENERGY BETWEEN THE EXPANDING PARCEL AND THE SURROUNDING AIR. ONLY NOT AS ‘HEAT’ (HENCE ‘a-diabatic’), ONLY AS ‘WORK’.

    Are you really that dense? It is pV work and anyone daring to sit in on a Thermodynamics 101 class within the first four or five lectures knows this… you don’t. The classes are free Stephen.

    Stephen says earlier: “I was quoting Kristian. He thinks adiabatic uplift involves a thermodynamic exchange between the rising parcel and surrounding molecules. “

    Of course Kristian, myself and among many others here think so Stephen, everyone that knows anything past the first four or five lectures into any Thermodynamics 101 class knows the same. Tried and true for over a hundred years. It is YOU that is so very royally messed up. Get your 60 years of dues back from the Royal Meteorological Society if it is they who fed such thoughts into your mind.

    And expanding gases do not always cool! Get that? See free expansion. Why not Stephen?

    I regret, but I finally wandered over to your “New Climate Model” site Stephen (http://www.newclimatemodel.com/new-climate-model/) under the link under your name. Holy moly! No wonder you are taking all of the time of various commenters here on TalkShop trying to straiten you out! You have absolutely no intention to ever change, learn and advance… you are defending you screwed up “New Climate Model” like an attorney! Nearly every paragraph is full of mis-concepts and mistakes. There in the first few posts you state that the Stefan-Boltzmann Constant has the power to predict and think the thermodynamics laws, like the Ideal Gas Law, need to be changed to align with your mis-concepts and even the units themselves you are using are not even correct (but I do remember Stephen, you were told all of this over a year ago). Sorry, I’m no longer following this silly visage.

    I mean it…. apologize to Kristian!

  59. Kristian said that the parcel cooled and the surrounding air warmed.

    That is a net transfer of heat.

  60. wayne,

    Try reading the links I have provided.

    http://www.theweatherprediction.com/habyhints/33/

    “An adiabatic process assumes no heat, mass or OR MOMENTUM pass across the air parcel boundary”

    If no momentum crosses the boundary no work is being done on the surrounding molecules.

    and see here:

    http://en.mimi.hu/meteorology/adiabatic_lapse_rate.html

    “The Adiabatic lapse rate is the rate of temperature change that occurs in an atmosphere as a function of elevation”

    If it is a function of elevation it cannot also be a function of any interaction with the surrounding molecules.

    I’m afraid that, astonishingly, none of you have learned about adiabatic ascent and descent in atmospheres.

    I don’t recognise your comments about my New Climate Model either. You are not interpreting the words correctly.

  61. Maybe it will help if I pont out that an interaction between the parcel and the surrounding molecules would involve movement in the horizontal plane and that would be diabatic. As such it would be governed by the Laws of Thermodynamics.

    In contrast a temperature change caused exclusivly by movement in the vertical plane via work done with or against gravity is purely Newtonian and so not involved in any thermodynamic processes.

    The term ‘adiabatic’ is exclusively allocated to the thermal effect of movement in the vertical plane and so must by definition not include any interaction there might be with surrounding molecules.

    Look at this link:

    http://www.theweatherprediction.com/habyhints/33/

    which says:

    ” there are two separate phenomena occurring at the same time. One phenomenon is the parcel of air cooling at the DALR. The second phenomenon is the parcel warming through latent heat release of condensation. These two processes partially offset each other.”

    The former process is adiabatic and the latter process is diabatic but by definition the former is a result only of movement against gravity.

    All energy used in the adiabatic portion of uplift against gravity goes to PE which can be recovered again on descent. That energy forms no part of any other thermodynamic exchange that there may be with surrounding molecules.

  62. Trick says:

    Stephen 7:36pm: “In contrast a temperature change caused exclusivly by movement in the vertical plane via work done with or against gravity is purely Newtonian and so not involved in any thermodynamic processes.”

    No. Prior Maxwell’s ~1868 paper this was generally accepted as true; after his ~1868 paper the thermodynamic p*V term was added to the parcel’s total energy renamed enthalpy and the kinetic theory of molecules as simple billiard balls was improved by adding this thermodynamic process on the parcel path from initial state to final state which became generally accepted.

    It is possible constant reminder will help you; but I doubt it is probable. Enthalpy is the physical quantity of relevance in meteorological applications whether your imagination can handle it or not.

  63. Trick,

    The initial change in temperature resulting from movement in the vertical plane is not a consequence of thermodynamics but once the change in temperature has occurred then it will become involved in thermodynamics.

    As usual you are obfuscating rather than clarifying.

  64. Trick says:

    Stephen 8:11pm: “The initial change in temperature resulting from movement in the vertical plane is not a consequence of thermodynamics ..”

    No. The parcel p and V also change with vertical movement. The parcel initial thermodynamic T,p,V state changes to the final state.

    “…once the change in temperature has occurred then it will become involved in thermodynamics.”

    Nonsense. The meteorological parcel was involved in thermodynamics from the beginning to the end. p,V,T changed continuously! Not just the position of some billiard balls.

    “..you are obfuscating rather than clarifying.”

    That Stephen can not correctly imagine even the most basic meteorological concept of parcel enthalpy is not an example of obfuscation; this is an example of reading a text book & relating what one finds because Stephen won’t & can’t do so. Stephen calling the parcel process “purely newtonian” is obfuscation after the correct text book explanation is presented.

  65. The fact is that a parcel of gas which moves up or down within a gravitational field changes its temperature, pressure and volume without any thermodynamic interaction with surrounding molecules.

    That is the nub of the issue.

    That is what adiabatic means.

    That is what does not appear to have been taught or learned.

    All of you are criticising my comments on the basis that there ‘must’ be a thermodynamic relationship with surrounding molecules when a gas parcel moves up or down within a gravitational field but there isn’t any such thermodynamic relationship with surrounding molecules for an adiabatic process.

    It is accepted that no process is completely adiabatic so in reality there will always be SOME thermodynamic interaction between the moving parcel and surrounding molecules but that interaction is diabatic and not adiabatic.

    The thing is that the nature of the adiabatic process is fundamental to atmospheric temperature (KE), energy content (KE+PE), circulation (climate zones and jet streams) and structure (lapse rates) but it is entirely missing from the radiative theory of gaseous atmospheres.

    I have put the adiabatic process back into its proper place and, having done that, simple logic and observation causes the real scenario to fall into place in the manner that I have unsuccessfully attempted to show you.

  66. wayne says:

    “The fact is that a parcel of gas which moves up or down within a gravitational field changes its temperature, pressure and volume without any thermodynamic interaction with surrounding molecules.”

    So with that fact that you claim if you expand a packet of air with no surrounding gas particles to push out of the way (no interaction) it’s temperature, pressure and density all decrease?

  67. “So with that fact that you claim if you expand a packet of air with no surrounding gas particles to push out of the way (no interaction) it’s temperature, pressure and density all decrease?”

    Correct, why would you doubt it ?

    If you inject gas into a vacuum it will expand, pressure will reduce and it will cool.

    Well established science.

  68. wayne says: November 5, 2014 at 8:03 am

    “The fact is that a parcel of gas which moves up or down within a gravitational field changes its temperature, pressure and volume without any thermodynamic interaction with surrounding molecules.”)

    “So with that fact that you claim if you expand a packet of air with no surrounding gas particles to push out of the way (no interaction) it’s temperature, pressure and density all decrease?”

    Wayne, As you have explained, the whole atmospheric processes, cannot be within the comprehension of mear earthlings. Give it up, relax with a beer, and admit, “Wad da fuck,over!”

  69. Ben Wouters says:

    Stephen Wilde says: November 5, 2014 at 7:34 am

    ” The fact is that a parcel of gas which moves up or down within a gravitational field changes its temperature, pressure and volume without any thermodynamic interaction with surrounding molecules.”
    Wrong again. The parcel moves up or down within an atmosphere that is in hydrostatic balance against gravity. As long as you don’t get this, you’ll never understand how convection works.

    No reaction on my CAPE comment. Finally figured out what CAPE is all about?

  70. Ben,

    The initial surface warming that causes the parcel to detach from the surface is diabatic.

    After that, all further uplift is adiabatic until the parcel reaches a warmer level such as the tropopause.

    During adiabatic uplift and descent the hydrostatic balance is unchanged because all uplift is matched by descent for a zero net effect.

    I understand the concept of hydrostatic balance, indeed my comments implicitly rely on it. The conversion of KE to PE and back again is integral to the maintenance of hydrostatic balance.

    As regards CAPE:

    There can be no Convectively Acquired Potential Energy at the surface because at that point there has been no convection. When a CAPE measurement is taken at the surface one actually measures the amount of local ‘excess’ kinetic energy and then one calculates how high it would cause the parcel to rise as a result of the induced buoyancy.

    The CAPE figure is then the amount of potential energy that the parcel will have acquired by virtue of the convection that results from the buoyancy.

    That convectively acquired PE is in the air at the top of the column after it has risen and has been created from the KE that was previously at the surface.

    If the parcel descends again then the CAPE reconverts to KE lower down.

  71. wayne says:

    Aawww, guys, quiet! I really do like Stephen, he’s always polite and I am usually polite but sometimes I want to grab him by his lapels and shake him… “Come on Stephen, come back to reality partner.. had a few to many huh”. I had the perfect way for Stephen to correct himself and you’all are going to blow my plan! 😉 All that is needed is for Stephen to give back to us the rules his ‘new climate model’ operates under… all model have set rules, math has rules, logic is a set of rules, physics has it’s rules, get it? In him trying to formulate the rules he should see where he cannot and maybe, just maybe he might see where he went off track if he cannot come up with a coherent set of rules, his own rules.

    Still like his hypothesis on the latitudes of the jet streams. I’ve been watching them bi-weekly now for the last some eight years and that one does seem to hold true so far. They used to hover over the US/Canada border and now that the sun went dormant they are now sometime even down touching Mexico! And it is slowly getting colder.

  72. Thanks wayne, I was getting dispirited by all the animosity when I really do know what I am talking about.

    My New Climate Model relies on basic meteorology and so far complies with observations.

    I have not made up any rules.

    All I have done is join dots to make them into a coherent story which will be either rebutted or confirmed by real world events.

    If I have made a fundamental scientific error then I welcome attempts to identify it but you don’t seem to have managed that so far.

    The knowledge here about meteorology seems to be pretty limited.

  73. wayne says:

    Ok, but without rules you have no model at all, even just verbal rules since you are not big on math are better than no rules at all. Write them out for yourself so you never forget them. Just saying it is built on meteorology doesn’t cut it Stephen though you can use meteorology’s rules and equations but list them.

    A big hint… listen to Richard Feynman’s explanation of his q.e.d. model in some spare time (http://vega.org.uk/video/subseries/8). This is not to get you to understand quantum electro dynamics but to see how he takes a fairly complicated subject and breaks it down to a few simple rules that someone who doesn’t even know math can follow and understand. He lists his rules out very simple, one by one, and knowing those simple rules then anyone can understand the entire model.

    Forming your rules may not be quite as easy and simple as it first appears for you need to make damn sure you never waffle or have to backup and change and violate some earlier rule you wrote down. Be very precise and picky.

    Now… you never answered my question above and it can be your first rule… what does your expanding gas packet do to the T, P and ρ as I described the situation above? I need to know to understand your model.

  74. The rules are set out in the Model.

    Unless the real world follows the sequence I have set out then the Model will be falsified.

    In reply to your question:

    If you inject gas into a vacuum it will expand, pressure will reduce and it will cool.

    Well established science.

    What is your story ?

    And how do you think that links to the much broader phenomenological description contained in my Model ?

  75. Ben Wouters says:

    Stephen Wilde says: November 5, 2014 at 10:26 am

    ” The initial surface warming that causes the parcel to detach from the surface is diabatic.

    After that, all further uplift is adiabatic until the parcel reaches a warmer level such as the tropopause.”
    Correct, and during uplift the parcel cools according the DALR and SALR.
    The tropopause is the ultimate top for most convection, most convection doesn’t even come close.

    ” During adiabatic uplift and descent the hydrostatic balance is unchanged because all uplift is matched by descent for a zero net effect.”
    This shows clearly that you do NOT understand hydrostatic balance (HB) and buoyancy.
    A parcel rises BECAUSE the HB is OUT OF balance for the parcel.
    The pressure difference between bottom and top of the parcel pushing it upward is greater than the downward pull of gravity. This makes the parcel rise.

    ” There can be no Convectively Acquired Potential Energy at the surface because at that point there has been no convection.”
    CAPE in meteorology stands for Convective AVAILABLE Potential Energy.
    Don’t know where your Acquired comes from.
    http://en.wikipedia.org/wiki/Convective_available_potential_energy
    CAPE is normally calculated for parcels at the surface and indicates the POTENTIAL for development of convection.

  76. wayne says:

    Stephen:

    “If you inject gas into a vacuum it will expand, pressure will reduce and it will cool.
    Well established science.
    What is your story ?”

    My story? Well, you couldn’t have listened to Feynman’s methods of explaining a model or theory as I suggested, only twenty minutes went by. Do you not care for me trying to help you? Slow down and listen. I fell asleep listening to be fresh on any questions of his methods to form rules that should arise from you. And I am guessing you also didn’t spend much time forming your first rule.

    I look up many references to check your first rule’s assumptions of the temperature meaningfully decreasing and each and every one of the references says just the opposite, the beginning and ending temperatures are equal in the case of adiabatic and in ideal gases like you find in our low pressure atmosphere.

    Here, John Hopkins:
    http://www.pha.jhu.edu/~broholm/l35/node6.html : “If we are dealing with an ideal gas then the absence of a change in the internal energy implies that the temperature is the same before and after the expansion”

    Davidson University:
    http://webphysics.davidson.edu/physlet_resources/bu_semester1/c27_process_expansion_sim.html : “In a free expansion, gas is allowed to expand into a vacuum. This happens quickly, so there is no heat transferred.” [equation] “There is no change in internal energy, so the temperature stays the same.” “Because the temperature is constant, the connection between the initial and final states is”

    http://www.phys.uwosh.edu/rioux/genphysI/pdf/lost_work.pdf : “Adiabatic Free-Expansion” [lot of equations] “TA = TB”

    Since our atmosphere is at a relatively low pressure and tests very close to an ideal gas I tend to lean on their side, MIT and Stanford where I have sat through some TD classes say the same.

    Stop right here on your Rule#1 and find out provably why you are right and all of the universities are wrong or vice versa. Just describing what is up in your head and calling it a model with no substance or well thought out proofs and rules does not make anything but just your story.

  77. Ben,

    “CAPE is defined thus in a previous link:

    “the amount of energy a parcel of air would have if lifted a certain distance vertically through the atmosphere.”

    i.e. it is the energy contained by the parcel in potential form AFTER it has been uplifted.”

    I think one could validly look at it both ways namely:

    From a point of view at the surface it indicates the POTENTIAL for development of convection.

    From a point of view at the top of the column it indicates the amount of energy the air will be carrying in POTENTIAL form after the convection has run its course.

    The point being that what you say doesn’t detract from my point at all. It is just a matter of perspective.

    My slip as regards ‘acquired’ and ‘available’.

    Given that convection goes down as well as up the term applies at both top and bottom of the vertical column.

    You said:

    “A parcel rises BECAUSE the HB is OUT OF balance for the parcel.”

    Of course it does but so what ?

    I was referring to the hydrostatic balance of the entire atmosphere because that seemed to be what you were referring to.

    And what makes the HB out of balance locally ?

    Too much KE at the bottom and too much PE at the top so convection is created to swap them around and restore overall balance for the atmosphere as a whole.

    The thing is that there is nothing you say that contradicts what I say, You just need to refine the way you look at it all.

    It is the concepts that give rise to the maths that matter and they are best expressed verbally and pictorially.

  78. wayne,

    Internal energy comprises both KE and PE so that is indeed unchanged before and after expansion.

    Also, a complete adiabatic cycle is both up and down so for an ideal gas with no leakage the temperature when the air leaves the surface will be the same as the temperature when the air returns to the surface.

    Those quotes refer to free expansion where no work is done so free expansion implies no work against gravity either so it doesn’t apply here. If work is done against gravity the temperature does change.

    The temperature changes because work done against gravity changes sensible energy (KE) to non sensible energy (PE) in a reversible process.

  79. Free expansion also ignores the work done when molecules move apart against the forces of attraction between molecules.

    It is well known that such moving apart converts KE to PE thus reducing temperature whilst keeping internal energy stable.

  80. Ben Wouters says:

    Stephen Wilde says: November 5, 2014 at 4:58 pm

    “CAPE is defined thus in a previous link:
    “the amount of energy a parcel of air would have if lifted a certain distance vertically through the atmosphere.”
    i.e. it is the energy contained by the parcel in potential form AFTER it has been uplifted.”

    Notice the words WOULD and IF. IT IS A CALCULATION OF THE BUOYANCY POTENTIAL.
    It has NOTHING to do with the increase of PE of the parcel.

    ” From a point of view at the top of the column it indicates the amount of energy the air will be carrying in POTENTIAL form after the convection has run its course”
    There is no column to consider. A parcel is small.
    When convection stops, the parcel ‘floats’ at that level, and has the same PE as the surrounding air.
    It has become part of the static atmosphere.

    “You said:
    “A parcel rises BECAUSE the HB is OUT OF balance for the parcel.”
    Of course it does but so what ?”
    ????? So far you have continuously stated that the parcel rises because internal KE is converted into PE, without giving the mechanism. Reason of course being that there is no such mechanism.
    The parcel is pushed upward by the surrounding atmosphere, which is doing the work against gravity.

    Pse stop being so dense, so we can finally start a much more relevant discussion on the underlying reason for the average ELR of ~6,5 k/km and how energy flows through the atmosphere from surface to space. Convection is but a small player in that flow.

  81. Ben Wouters says:

    @Stephen

    see http://www.recreationalflying.com/tutorials/meteorology/section1a.html#pressure_gradient
    specifically:
    ” As the pressure decreases with height so, in any parcel of air, the downwards pressure over the top of the parcel must be less than the upwards pressure under the bottom. Thus within the parcel there is a vertical component of the pressure gradient force acting upward. Generally this force is balanced by the gravitational force, so the net sum of forces is zero and the parcel floats in equilibrium. This balance of forces is called the hydrostatic balance. When the two forces do not quite balance, the difference is the buoyancy force. This is the upward or downward force exerted on a parcel of air arising from the density difference between the parcel and the surrounding air.”

    Read this very carefully, and point out what you do not understand / agree with.
    This is all well understood and accepted science. Your version of convection makes no sense at all.

  82. wayne says:

    “Free expansion also ignores the work done when molecules move apart against the forces of attraction between molecules.”

    Shallow. True but in our atmosphere with dinitrogen, dioxygen and argon as the primary components there is no meaningful attraction so that does not apply to any significant degree. Also does not apply because the inter-molecular spacing is about 1000 times the molecular cross sections. No cookie there. But don’t discard what the free expansion experiments should show you so clear, it IS the work against surrounding molecules that cause the pV cooling.

    “It is well known that such moving apart converts KE to PE thus reducing temperature whilst keeping internal energy stable.”

    No. Not the way you stated that, as an absolute. It is only well known if there are strong attractions and in our atmosphere that does not apply so anything else is not well known… in fact it is well known that in such eleastic cases this does not apply. In water vapor’s case it does apply due to the hydrogen bonds from the electron distribution. No cookie there either.

    Stephen, you are not fooling anyone with your words, even if others never take the time to comment back and correct you. Don’t you understand that?

    Sorry but I can’t take any further time right now to attempt the apparent hopeless but I do wish you could stop, learn and advance but you must first and foremost be honest with yourself as to what you really ‘know’ or don’t really ‘know’ in all of the small details.

  83. Ben said:

    “So far you have continuously stated that the parcel rises because internal KE is converted into PE, without giving the mechanism. Reason of course being that there is no such mechanism.
    The parcel is pushed upward by the surrounding atmosphere, which is doing the work against gravity.”

    The parcel rises because uneven surface heating creates density differentials in the horizontal plane so the lighter parcels rise spontaneously above heavier parcels.

    Work is done by the parcel against gravity (there is the mechanism) so the parcel cools due to KE becoming PE. It cannot cool by transferring heat outside the parcel as Kristian thinks because that would then be a diabatic process not adiabatic

    If you cannot transfer heat in or out you can only cool the parcel by converting sensible energy (KE) to non sensible energy (PE).

    The parcel is not pushed up by anything. The spontaneously rising air leaves lower pressure beneath so heavier air flows in from the sides. That is the classical description of a low pressure cell in meteorology.

    “There is no column to consider. A parcel is small.”

    When convection stops the parcel is seated above a column of air.

    “When convection stops, the parcel ‘floats’ at that level, and has the same PE as the surrounding air.
    It has become part of the static atmosphere.”

    The parcel gets pushed aside by warmer parcels coming up from below and being denser than the air coming up from below it begins to sink back to the surface . That is the classical description of a high pressure cell in meteorology.

    Please read a meteorology text book and it will all fall into place for you.

    I have just been informed that my 91 year old mother has passed away so I will not have time to continue this.

  84. Ben Wouters says:

    Stephen Wilde says: November 6, 2014 at 9:27 am

    Condolences with your mother

    You have apparently chosen to remain clueless on the subject of convection,
    so this will indeed be the end of this discussion with you.

  85. One final point.

    Your link says this:

    “This balance of forces is called the hydrostatic balance. When the two forces do not quite balance, the difference is the buoyancy force. This is the upward or downward force exerted on a parcel of air arising from the density difference between the parcel and the surrounding air. A visible application of this principle is readily apparent in the hot-air balloons and airships of sport and recreational aviation.”

    The buoyancy force is created within the parcel due to its relative lightness. It is a force acting on the parcel from INSIDE the parcel.

    It is not a force acting on the parcel from OUTSIDE as you seem to think.

    That is why you can create uplift or descent by changing the density WITHIN a hot air balloon or airship.

    Once you change density within the parcel then the parcel rises or falls spontaneously.

    No pushing or pulling required from the surrounding air.

  86. Ben Wouters says:

    Stephen Wilde says: November 6, 2014 at 9:27 am

    ” The buoyancy force is created within the parcel due to its relative lightness. It is a force acting on the parcel from INSIDE the parcel.”
    Sure, probably molecules flapping their wings, energized by their KE.
    Stop this nonsense and concentrate on your family matters.
    If this is what 60 years studying meteo has brought you, you’ll never learn.
    You have shown to be clueless about:
    – hydrostatic balance
    – pressure gradient force
    – buoyancy.

  87. Ben Wouters says: November 6, 2014 at 4:02 pm
    Stephen Wilde says: November 6, 2014 at 9:27 am

    ” The buoyancy force is created within the parcel due to its relative lightness. It is a force acting on the parcel from INSIDE the parcel.”
    Sure, probably molecules flapping their wings, energized by their KE.
    Stop this nonsense and concentrate on your family matters.
    If this is what 60 years studying meteo has brought you, you’ll never learn.
    You have shown to be clueless about:
    – hydrostatic balance
    – pressure gradient force
    – buoyancy.

    Ben,
    I again apologise I thought you were like Stephen, I have studied and measured much of how the EMR works in this atmosphere, A speciality. Please continue to teach your knowledge of how this atmosphere works from your POV! From you, I have learned that latent heat converts to sensible heat “only” when it must, perhaps a bit later, with much instability! Thank you! Even idiots can learn, if they stop stepping on the same damn rake!

  88. Ben Wouters says:

    Will Janoschka says: November 6, 2014 at 10:25 pm

    Ok, apologies accepted, hoping we can have a civilized discussion.

    I have an imo pretty complete outline for a mechanism that could explain our very high temperatures on Earth relative to the moon. I like to discuss the atmospheric basics that could explain the AVERAGE atmospheric temperature profile of 6,5K/km.
    It accommodates old-fashioned meteorology, since meteo is not concerned with the HOW of the temp. profile. It just measures it and works from there on.

    I’ve started commenting on the abuse of the DALR and SALR almost 3 years ago now:

    Hans Jelbring: The Greenhouse Effect as a function of atmospheric Mass


    and I’m not open for discussions of adiabatic atmospheres, convection pushing the tropopause up and all related nonsense.

    I hope Wayne will chime in, and see if we can get something useful going.

  89. wayne says:

    First, Stephen, so sorry to hear about you mother. I personlly know how you must feel but it sounds like she had a full and long life. How there was little suffering involved. Will lay off you now.

    Notice a post by Will and Ben and I’ll now read those.

    Our thoughts are with you Stephen.

  90. wayne says:

    Ben, I too notice Stephen’s complete lack of seeing the physics involved and this is not to batter Stephen. Surely there is some way we Will, you and I can get him to visualize what is actually, physically occurring.

    He is ruined by the thoughts driven in by most meteorology texts of “Packets” and “Parcels”, he has become “packet-centric” in his mind, so much so, he cannot see anything else. That is more than apparent when he said the words “force is created within the parcel” and that is as far as I needed to read. Boom. False.

    For some reason he refuses to realize that gravity is the only force involved here and it only pulls things downward, never upward, and there is never any actual upward original force existing… ever, it is a differential of vector forces that are all downward but the difference is upward and that is the apparent buoyancy. You and I know that any layer of equal density that is not perfectly tangent to the spherical body’s surface will create a somewhat false-anti-gravitational-force called buoyancy and this is always because gravity always strives to make that equal-density ‘surfaces’ (in the physics conceptual sense) level and smooth and always tangent to the surface or otherwise negative differential ‘force’ vectors (really accelerations) spontaneously exist upward. But how do we get Stephen to realize this so clearly he never forgets ever again?

    He simply cannot see how forces that are all literally downward move less dense things upward. Differentials and vector math always seem to throw him.

    I probably could spend thirty more minutes saying this shorter and clearer but you both should get my meaning.

    I spend far too much time struggling with that… how to straighten Stephen’s mind out for I know it would help him so much.

  91. Just passing by in between family commitments.

    Heating molecules reduces density.

    Uneven surface heating places groups of moolecules of differing densities next to one another.

    Lighter groups rise above heavier groups spontaneously in adiabatic uplift.

    Once the surface energy has been transferred to the warmer, lighter group of molecules the resulting buoyancy is within the heated group of molecules and not imposed from some external source which you all seem to think pushes the lighter parcel up.

    If the uplift were being caused by a force external to the parcel then pressure would be highest beneath the rising parcel because of the resistance of the weight of the parcel to that external force.

    In fact, pressure is always lower beneath the rising parcel which means that heavier air is being pulled in beneath and therefore cannot be doing any pushing.

    That is how we get low pressure weather systems.

    Warmer air rises spontaneously having become lighter, cooler denser surface air flows in and forms a spiral due to the coriolis force.

  92. Kristian says:

    wayne,

    I would rather suggest that what Stephen fails to grasp is the essence of the adiabatic process, exemplified by his constant talking about ‘adibatic uplift’, as if it were the uplift itself that created the cooling. His argument, after all, is that if you move air higher up, it will cool from simply going higher, because some mechanical KE is transformed into mechanical PE. He is unable to separate between thermodynamic processes (the adiabatic part) and Newtonian processes (the uplift part). The adiabatic part is simply the air expanding and being compressed by changing the external pressure on it. In this regard, air lifting and sinking in the troposphere is analogous to a piston pushing down on a contained volume of gas and then popping back up, only within the atmosphere you will have to physically move the ‘air packet’ in question up and down along a pressure gradient in order to change the external pressure and consequently its volume. Stephen appears blind to this.

  93. “I would rather suggest that what Stephen fails to grasp is the essence of the adiabatic process, exemplified by his constant talking about ‘adibatic uplift’, as if it were the uplift itself that created the cooling.”

    http://en.mimi.hu/meteorology/adiabatic_lapse_rate.html

    “The Adiabatic lapse rate is the rate of temperature change that occurs in an atmosphere as a function of elevation”

    If it is a function of elevation it cannot also be a function of any interaction with the surrounding molecules.

    “dry adiabatic lapse rate—The rate of decrease of temperature with height when unsaturated air is lifted adiabatically (due to expansion as it is lifted to lower pressure). ”

    The uplift itself does cause the cooling.

  94. “A thermal column (or thermal) is a column of rising air in the lower altitudes of the Earth’s atmosphere. Thermals are created by the uneven heating of the Earth’s surface from solar radiation, and are an example of convection, specifically atmospheric convection. The Sun warms the ground, which in turn warms the air directly above it.[1] Dark earth, urban areas and roadways are good sources of thermals.

    The warmer air near the surface expands, becoming less dense than the surrounding air mass. The mass of lighter air rises, and as it does, it cools due to its expansion in the lower pressure of the higher altitude. It stops rising when it has cooled to the same temperature as the surrounding air. Associated with a thermal is a downward flow surrounding the thermal column. The downward moving exterior is caused by colder air being displaced at the top of the thermal.”

    from here:

    http://en.wikipedia.org/wiki/Thermal

    Which is almost a word for word repeat of what I have been saying.

    The air cools from expansion and NOT from losing heat to surrounding molecules. To the extent that there is some minor heat loss to surrounding molecules (via work done against those molecules) then that is a diabatic process.

    All expansion arising from uplift in the vertical plane and thus via work done against gravity is adiabatic and reversible on descent. That portion involves conversion of KE to PE.

  95. Stephen Wilde says: November 7, 2014 at 1:04 pm

    “I would rather suggest that what Stephen fails to grasp is the essence of the adiabatic process, exemplified by his constant talking about ‘adibatic uplift’, as if it were the uplift itself that created the cooling.” http://en.mimi.hu/meteorology/adiabatic_lapse_rate.html

    “The Adiabatic lapse rate is the rate of temperature change that occurs in an atmosphere as a function of elevation”

    “If it is a function of elevation it cannot also be a function of any interaction with the surrounding molecules.’

    It is not a function of elevation, it is a function of altitude, The top of Everest is cold and thin, but not because of the climbers.

    ——“dry adiabatic lapse rate—The rate of decrease of temperature with height when unsaturated air is lifted adiabatically (due to expansion as it is lifted to lower pressure)”——
    “The uplift itself does cause the cooling.”

    This is what is not correct! Wayne, Kristin, Ben, chime in if you agree or disagree! That original “change” is properly called “difference” The temperature does not “change” because of some physical process. It is “different” (lower at elevation) because of thermostatic state. N2 and O2 will both increase in temperature when compressed, along with an increase in pressure. This is adiabatic compression as in a auto engine The air is being worked on by the inertia of the flywheel All of this work is returned to the flywheel on the down-stroke.
    This is completely different than anything in the atmosphere. In this atmosphere gravitational attraction provides a static force that insists that N2, O2 temperature and pressure, both, because of the above airmass, increase, because of the above air mass compressing the air mass below. There is no work done, in this thermostatic state. That and only that is the adiabatic part of this atmosphere at every location. Transferring heat, whether sensible or latent is the diabatic. Any motion of that diabatic molecule involves no transfer of energy. It is gravitational potential maintaining the adiabat with a completely isentropic process. There is no work involved.

    Stephen,
    Perhaps you have been corrupted by the post modern “kinetic theory of everything” that HEAT (the noun) is the same as Newtonian kinetic energy.
    Heat has no relationship to the laws of motion within a gravitational potential. Heat has no momentum to exchange with positional potential within a gravitational force field. The molecular mass can have such, never the heat.
    The fools cannot know if molecular Brownian motion is caused by temperature or the electromotive and magnetomotive forces that are some weird function of temperature, however expressed. Perhaps it is “temperature” that is some function of electrical properties.
    Earth to low level, higher temperature, air parcel. “Go somewhere to rid yourself of that entropy”, If that is not clear, “Get the fuck off of this planet”.

  96. wayne says:

    Kristian, yes, the adiabatic part but I think until Stephen understands first why there is even a rising column of air, no packets (I once was a sailplane pilot for a few years and I know his concept is so wrong) he will never “get it”. He is always in a non-physical mindset.

    Stephen, you say:

    Heating molecules reduces density.

    Uneven surface heating places groups of molecules of differing densities next to one another.

    Ok, you have two sentences correct. Then…

    “Lighter groups rise above heavier groups spontaneously in adiabatic uplift.”

    Damn no, you are getting off topic, leave adiabatic out of it for right now. Lighter groups are not the actor here, the denser air that was not heated is the actor with the energy and force to do something, there is now a dent in the equal-density ‘surface’ and by all of physics that causes inflow of that dense not-heated air to fill that ‘dent’ and in doing so the lighter air is rather insignificant and is pushed out of the way and the only place for it to go is up. You end up with a up pointing vector because you have subtracted to unequal downward pointing vectors and the difference is upward but it is all caused be the denser surroundings, not the lighter warmed air.

    You then start to really go off the rails.

    “Once the surface energy has been transferred to the warmer, lighter group of molecules the resulting buoyancy is within the heated group of molecules and not imposed from some external source which you all seem to think pushes the lighter parcel up.”

    Now I won’t go any further till you can see that is incorrect. You have the “packet” with some imagined power which needs energy to act on it’s own and there is none there Stephen in your “packet”. Molecule-level and random velocities of molecules never have upward vectors, they all cancel. Gravity on the denser surrondings is where the actual energy lies whose differential vector is to push your “packet upward”.

    If wiki disagrees, then wiki is not correct. If all of your meteorology text disagree, they are also incorrect and are what led you off into never-never land.

  97. If the uplift were being caused by a force external to the parcel then pressure would be highest beneath the rising parcel because of the resistance of the weight of the parcel to that external force.

    In fact, pressure is always lower beneath the rising parcel which means that heavier air is being pulled in beneath and therefore cannot be doing any pushing.

    That is how we get low pressure weather systems.

    Warmer air rises spontaneously having become lighter, cooler denser surface air flows in and forms a spiral due to the coriolis force.

  98. “You have the “packet” with some imagined power which needs energy to act on it’s own and there is none there ”

    Warmer air rises due to it being less dense and thus lighter. It does not need to be pushed up.The lighter a gas the higher it will rise against gravity at a given temperature.

    Water vapour is lighter than air at the same temperature and so rises spontaneously faster and further than the air would have done.. It does not need to be pushed up.

    The fact of low pressure beneath the rising parcel is proof that the parcel rises before denser air flows in below.

  99. wayne says:

    Furthermore, you don’t even need any heat for this exact same process. Instead of warmer air being less dense if I that a tank of helium out into a huge asphalt parking lot and constantly release that helium that causes the same ‘dent’ that nature always want to level out and the helium starts to accelerate upward… not from some ‘energy’ within the helium!!! God, can’t you see that??

    And if I don’t turn the spigot off there are no “packets”, you get a constant column of helium accelerating and moving upward non stop until I do turn the spigot off. The spigot is the solar energy heating the surface and therefore the air in you case but THAT ENERGY is ONLY to make it less dense so the heavier can inflow, like when as kids you squeeze the swimming pool water and you get a spout of water UPWARD… it’s not the waters action, it is your muscles energy that CAUSED the upward spout.

    Surely, surely, Stephen you can visualize that. Otherwise, I’m about to give up on you.

  100. “If wiki disagrees, then wiki is not correct. If all of your meteorology text disagree, they are also incorrect and are what led you off into never-never land.”

    If you dismiss wiki and meteorology texts then it isn’t me who is out of line with the established science.

  101. wayne says:

    By the way Stephen, I was releasing that helium at NIGHT! Yes, thermals at night in that case. Cold helium rising, not warmer air.

  102. wayne says:

    That is called an equivalence principle. You cannot tell the difference between the two cases.

  103. ” the helium starts to accelerate upward… not from some ‘energy’ within the helium!!! God, can’t you see that??”

    Helium being lighter than air will rise spontaneously higher than air against the gravitational field at a given temperature.

    The surrounding air does not push the helium up otherwise highest pressure would be at the base of the helium column just as highest pressure would be in your hand squeezing the swimming pool water.

    You really have got it completely wrong.

  104. “Cold helium rising, not warmer air.”

    Being lighter than air it rises spontaneously to a greater height at the same temperature.

  105. Wayne’s World:

    Where storms occur in regions of high pressure and settled weather occurs in regions of low pressure.

    ROFL

  106. wayne says:

    Yes, I also know you have not an inkling of scale. In Wayne’s World, the actual world. Come visit some time when you get back to down to Earth. (but on the bright side at least I now know for certain that you are really nothing but a troll here) No more food.

  107. wayne says: November 7, 2014 at 7:29 pm

    “By the way Stephen, I was releasing that helium at NIGHT! Yes, thermals at night in that case. Cold helium rising, not warmer air.”

    Helium, increases temperature upon expansion, at temperatures above 55 Kelvin. Way neet gas!

    Are we having fun yet? The Glenfiddich was much better than the beer! I doubt that Stephen even read my post. He is such a self assured attorney! I doubt that he has presented anything to a jury!

  108. wayne says:

    I was thinking exactly the same thing the other day Will. If I were in need of a good logical attorney I now know one place definitely not to look. When even the judge started laughing at my council… that would be just too much to bear.

    But sad day really. The student that could never learn.

  109. Nothing but ad hominems left.

  110. wayne says: November 7, 2014 at 8:05 pm

    “I was thinking exactly the same thing the other day Will. If I were in need of a good logical attorney I now know one place definitely not to look. When even the judge started laughing at my council… that would be just too much to bear.”

    Indeed Wayne,
    I am likely this world’s expert on how to screw up a thermometric measurement via radiometric means. I can claim, I have forgotten more mistakes than anyone else can make! All I remember is that each one was painful. I do not wish to go back to repeatedly stepping on the same damn rake, or again go down the same dirt road, leading only to the swamp.

    “But sad day really. The student that could never learn.”

    Wayne, there are several on this blog!

  111. Eric Barnes says:

    Bravo for your persistence Stephen. Wayne and Will are dipping into the gutter now because their arguments are empty.

  112. Thank you, Eric.

    The thing that so many do not understand is that a gravitational field neither adds energy to, nor takes energy away from gas molecules within it.

    Yet moving gas molecules relative to that gravitational field changes temperature, pressure and volume.

    Furthermore, those changes do not involve any exchange of energy as heat with surrounding molecules.

    Suggesting that there is an exchange of energy as work is not an answer because work is not a form of energy but rather a method of transferring energy so if work is done in cooling one molecule and warming another that is still a transfer of energy as heat.

    That is where Kristian goes wrong.

    So, how does one change temperature without adding or removing heat whilst keeping total internal energy the same ?

    The only solution is to transform the internal energy from sensible energy (KE) to non sensible energy (PE) and back again.

    That is exactly what happens during adiabatic uplift and descent.

    Some sources do refer to changes in internal energy during uplift and descent but they are referring to changes in KE and not changes in total internal energy which is KE and PE combined. A matter of sloppy terminology in those sources because one cannot have a change in total internal energy unless internal energy is gained or lost and a gravitational field cannot add or remove internal energy, merely transform it.

    Established science from before the radiative theory of gases took hold.

  113. “So, how does one change temperature without adding or removing heat whilst keeping total internal energy the same ?”

    should read:

    “So, how does one change temperature without exchanging heat with surrounding molecules whilst keeping total energy the same ?”

    We could do with an edit function here.

  114. Ben Wouters says:

    wayne says: November 7, 2014 at 11:42 am

    ” For some reason he refuses to realize that gravity is the only force involved here and it only pulls things downward, never upward, and there is never any actual upward original force existing… ever, it is a differential of vector forces that are all downward but the difference is upward and that is the apparent buoyancy.”
    Sorry Wayne, but I have to disagree with you as well.
    If gravity were the only force in play the atmosphere would be compressed on the surface.
    It isn’t, due to its INTERNAL pressure, caused by its temperature.
    The air near the surface has a pressure that is large enough to PUSH the entire column upwards.
    That the atmosphere doesn’t fly of to space is due to gravity acting against this upward force.
    That is what we call ‘hydrostatic balance’.
    Increase the total internal energy and the atmosphere expands AGAINST gravity.

    see http://www.recreationalflying.com/tutorials/meteorology/section1a.html#gas_laws

  115. Ben,

    It is correct that if you increase TOTAL internal energy the atmosphere expands AGAINST gravity.

    However, I have a problem with the concept of the pressure at the base PUSHING upwards.

    The pressure at the base is caused by gravity PULLING downwards and hydrostatic balance is a consequence of kinetic energy in the gas balancing against that downward force and thereby keeping the gas suspended off the surface.Just you say.

    If one then adds more internal energy then it is true that the gases will rise and a new hydrostatic balance will be established at a new atmospheric volume.

    But I don’t see that as the pressure at the surface increasing any upward pushing because you haven’t increased atmospheric mass so surface pressure remains the same despite the reduced density.

    There has been no increase in surface pressure and so no increase in upward pushing can have occurred contrary to your description.

    Instead the kinetic energy at the surface becomes more than is required to maintain the original hydrostatic balance and the whole atmosphere spontaneously expands to a higher level against gravity with no extra upward pushing required.

    If one adds kinetic energy from uneven surface heating to a parcel of energy at the surface then the same process occurs on a small scale.

    The reduction in density locally results in a spontaneous rise locally and a fall in surface pressure locally until denser air flows in from the sides (and out from above) to restore the original hydrostatic balance.

    Conversion of KE to PE in uplift and PE to KE on descent is key to the whole process because one cannot create or destroy energy, only change its form between sensible and non sensible.

    Anyway, the flaw in your account is that increasing the total energy of the gas does not increase surface pressure so the expansion cannot be a result of increased upward pushing from surface pressure.

    Do you not see ?

  116. AGW theory does not acknowledge the freedom of a gaseous atmosphere to expand further against gravity when the surface temperature is raised and that that expansion is a surface cooling effect because the reduction in density at the surface results in less conduction from the solar energy flowing through the system. Reduce surface density and solar energy flows through the system faster which offsets the potential surface warming from GHGs.

    In reality DWIR and UWIR from GHGs are in balance, UWIR reduces energy returning to the surface in convective descent but DWIR compensates for that at the surface for a zero temperature change at the surface.

    Even if there were an imbalance between that UWIR and DWIR any thermal effect from that imbalance would be negated by an adjustment in the hydrostatic balance.

    That actually resolves a remaining issue with my hypothesis.

    I was proposing that GHGs might require a circulation change (an adjustment in the hydrostatic balance) to balance their thermal effect but if DWIR out to space is in balance with DWIR returning to the surface then no such adjustment is required.

    The net effect of GHGs really is zero because of the reduction in energy returned to the surface on convective descent exactly balancing the increase in DWIR down to the surface.

  117. Kristian says:

    Ben Wouters says, November 8, 2014 at 7:31 am:

    “Sorry Wayne, but I have to disagree with you as well.
    If gravity were the only force in play the atmosphere would be compressed on the surface.
    It isn’t, due to its INTERNAL pressure, caused by its temperature.
    The air near the surface has a pressure that is large enough to PUSH the entire column upwards.
    That the atmosphere doesn’t fly of to space is due to gravity acting against this upward force.
    That is what we call ‘hydrostatic balance’.
    Increase the total internal energy and the atmosphere expands AGAINST gravity.”

    Yes, I agree.

    The atmosphere is, by convention, naturally kept in place by the balance between two opposing forces, the downward one being gravity, or rather, the weight of air pressing down from above.

    The upward force always opposing this weight (expressed by pressure) in a fluid is called ‘buoyancy’. It comes about because the pressure is always higher lower in the column (closer to the centre of gravity) than higher up, so the difference creates a net force acting upwards on each layer (or ‘parcel’) of air, maintaining ‘hydrostatic balance’. I think this is what wayne is getting at.

    http://hyperphysics.phy-astr.gsu.edu/hbase/pbuoy.html

    If you somehow heat a volume of surface air, it will try to expand in place, but as soon as it starts doing so, its volume increases (without an increase in mass) and so its buoyant force will start ‘pushing’ it upwards (buoyancy, after all, is equal to the magnitude of the weight (mg) of fluid displaced). The hydrostatic balance is disturbed and the heated air will automatically rise.

    The more you heat the air, the more it will expand, and the more of heavier air it will displace, so the greater its upward momentum will be, meaning, the higher it will need to go to restabilise.

    I do get what wayne is saying, however, that gravity is the origin of all this happening, so in a way it’s the only real force in operation, buoyancy simply being the result of (or, rather, the reaction to) its downward-acting pull.

  118. Kristian says:

    Stephen Wilde says, November 8, 2014 at 4:30 am:

    “Suggesting that there is an exchange of energy as work is not an answer because work is not a form of energy but rather a method of transferring energy so if work is done in cooling one molecule and warming another that is still a transfer of energy as heat.

    That is where Kristian goes wrong.”

    And all of thermodynamics too, then, I guess … How would you interpret this equation, Stephen?

    ΔU = Q – W

    Let us hear it? What does it say? What is U? What is Q? What is W? How does this equation apply to the adiabatic process?

    http://en.wikipedia.org/wiki/First_law_of_thermodynamics#Description
    http://hyperphysics.phy-astr.gsu.edu/hbase/thermo/heat.html
    http://en.wikipedia.org/wiki/Adiabatic_process#Adiabatic_heating_and_cooling

    You’ve been supplied with these links (and many more) tens of times, Stephen, to back up arguments being made, but you have never shown even the slightest hint of ever having read any them, even opened them. I have even quoted passages from several of them here, highlighted the relevant parts for you. But you act as though you refuse to even lay eyes on it.

    Why aren’t you reading, Stephen? What’s so scary about it? It’s only basic thermodynamics.

    Are you afraid you might find something there that you might not like? That might go directly against your ‘teachings’.

    I simply don’t get you …

  119. wayne says:

    “I think this is what wayne is getting at.”

    Thank you Kristian, that is exactly what I was describing and not a world with the atmosphere collaped upon the surface.

    For other commenters:

    You throw a log into a lake, the lake rises (say 1/10000″) to accomodate the ‘dent’ in the flat surface and if the water was not more dense than the log the log would be pulled by gravity all of the way to the bottom. But it is denser and that creates upward force because that now excess 1/10000″ of water because of the log is wanting to lower that 1/10000″ and smooth out the dent in the flat lake surface and it it THAT downward pull by gravity on the lake that is making the log float. I’ll stand by my words, coming right out of a physics book that I read when I was a kid in junior high.

    It is the same for lighter air, only this time the air parcel is literally at te bottom (not of the lake but of the atmosphere) and if you make it any less dense the downward pull on the atmosphere, not the parcel, is going to push the parcel upward. Why? Because the atmosphere is now denser that the parcel. I’ll stand by my words, coming right out of a physics book read when I was a kid in junior high.

    Gravity never pulls matter upward. Period. Go figure.

  120. Kristian says:

    Yes, that’s a nice way of describing it 🙂

  121. Ben Wouters says:

    wayne says: November 8, 2014 at 11:08 am

    “Thank you Kristian, that is exactly what I was describing and not a world with the atmosphere collaped upon the surface.”

    Reduce the temperature of the surface and atmosphere to 0K and all molecules will be motionless on the surface, tightly packed.
    Increase the temp. above the temp. required to create a gas, and the atmosphere will be ” in the air” due to its internal pressure (molecules bumping into each other).
    The higher the temp. the more vigorous the molecules bump into each other, increasing the internal pressure, thus expanding the total atmosphere.
    Since the mass doesn’t change, the surface pressure remains the same.

  122. Ben said:

    “The higher the temp. the more vigorous the molecules bump into each other, increasing the internal pressure, thus expanding the total atmosphere.
    Since the mass doesn’t change, the surface pressure remains the same.”

    Yes, you’ve got it.

    The expansion is within the atmosphere which pushes out to space but does not push towards the surface. That is proved by the fact that mass does not increase and surface pressure does not rise.

    We see that in fact surface pressure starts to FALL and that drags in denser air from the side AFTER uplift has begun so no pushing from the sides either.

    That is why I previously stated that in a hot air balloon the buoyancy comes from WITHIN the parcel of warmed less dense air contained in the balloon.

    Nothing pushes it up. Instead, the lighter parcel itself spontaneously pushes towards the vacuum of space against the downward pull of gravity.

    That is implicit in all the links provided by Kristian but he and wayne have misunderstood them.

    Warming air locally causes local uplift which produces LOWER pressure beneath so the uplift cannot possibly be caused by pushing from below or from the sides since that would require HIGHER pressure below or at the sides.

    Hence my previous ROFL.

  123. wayne said:

    “It is the same for lighter air, only this time the air parcel is literally at the bottom (not of the lake but of the atmosphere) and if you make it any less dense the downward pull on the atmosphere, not the parcel, is going to push the parcel upward. Why? Because the atmosphere is now denser than the parcel”

    The log stays afloat because the weight of the water displaced by the log is greater than the weight of the log. It rises out of the water until the weight of the water displaced equals the weight of the log.

    Instead,consider that log at the bottom of the lake. Since the water displaced by it is heavier than the log there will be buoyancy WITHIN the log and it will rise towards the surface spontaneously. That rise will produce lower pressure below the log and water will flow in beneath it but the water is not pushing the log up. What is causing the log to rise is its inherent buoyancy in being less heavy than the water around it.

    The same for a warmed parcel of air at the surface. It has buoyancy WITHIN it because it is less heavy than the surrounding air.

    However in the process of rising up through the atmosphere the warmed parcel of air cools and becomes less dense at the same rate as the air around it due to the lapse rate so it doesn’t stop until it reaches a region of warmer air such as an inversion layer.

    During the rise from surface to inversion layer no work is being done against the surrounding molecules as evidenced by the fact that the initial temperature differential that arose at the surface is maintained throughout the rise in height.

    All the work is done against gravity otherwise the temperature differential would NOT be retained.

    Of course there is some leakage to surrounding molecules but not much and such as there is would simply slow the rate of uplift.

    All the uplift that comprises work against gravity simply creates PE at the expense of KE and that PE stays stored in the air keeping the atmosphere suspended permanently against the gravitational field.

    It is the convective overturning cycle constantly destroying and recreating that PE which gives the gaseous atmosphere permanence above the solid surface.

  124. wayne says:

    Comment on A small sceptical voice in the lukewarm wilderness by Ben Wouters
    Today, November 08, 2014, 8 hours ago | Ben Wouters

    wayne says: November 8, 2014 at 11:08 am
    “Thank you Kristian, that is exactly what I was describing and not a world with the atmosphere collapsed upon the surface.”

    Ben: Reduce the temperature of the surface and atmosphere to 0K and all molecules will be motionless on the surface, tightly packed.

    Increase the temp. above the temp. required to create a gas, and the atmosphere will be ” in the air” due to its internal pressure (molecules bumping into each other).

    The higher the temp. the more vigorous the molecules bump into each other, increasing the internal pressure, thus expanding the total atmosphere.

    Since the mass doesn’t change, the surface pressure remains the same.

    What “internal pressure” exactly?

    Should watch out Ben, I think you are catching it from others here.

    Be careful of your thoughts. What you said of the atmosphere when at less that 3 K was correct, and it would be a sea of liquid nitrogen, oxygen and argon sitting on the original surface but try this thought out… imagine putting on some cyro-scuba gear and go to the very bottom of that ten meters of liquid to the old surface and measure the pressure and you will find it is still right at 101325 pascals. Now warm those liquids back up to lets say 150 K. The TOA will be much lower than we find today but go back to the very surface and measure the pressure. Right at 101325 pascals. Pressure *only* has to do with the mass per area further out the gravity well (above you) and the gravitational acceleration. Your last statement seemed to indicate you already realized that so why through i n “internal pressure”. There are others here yjat it will further confuse their thoughts.

    But, then you do started to talk of increased “internal pressure”… well, no… that’s the mis-concept right there, all of pressure is already been accounted for and is constant at any vretuical mass level no matter what the temperature or height is and that is even a dangerous way to start thinking (in my opinion), wrong variable, blaming and using pressure where it is literally density that you should be speaking of. If you add energy to our atmosphere globally, it warms and by warming, density decreases but pressure is invariant in what we have been speaking of so only temperature and density are changing (and therefore height) as the energy flows in. Density gets an undeserved bad rap and pressure is so often used in it’s place at exactly the wrong moment in these comments.

    Said enough again I hope, alwats too long but think on that. If you are also a picker, pick away.

  125. wayne asked:

    “What “internal pressure” exactly?”

    The outward force created by adding kinetic energy (heat) to the gas.

    When one adds heat from an external source then vibration of the molecules increases and the space between them increases.

    That increased demand for space pushes against the gravitational field (an increase in internal pressure).

    The process of pushing against the gravitational field causes the parcel to rise to a new height (work is done against gravity) which is determined by the parcel’s mass and kinetic energy content.

    The rise in height is a result of the warmer parcel doing more work against gravity than the surrounding cooler air and so it spontaneously rises higher with no need for any pushing from the surrounding cooler air molecules.

    In the same way a log totally immersed in water of the same temperature and being lighter than the water will do more work against gravity than the water around it and will rise higher within the water without any upward pushing from the water.

    The process of floating between two mediums of different densities such as the log floating between air and water is quite different.

    Dense objects are heavier because they do more work WITH gravity than against it. Less dense objects do more work AGAINST gravity than do heavier denser objects.

    The log is heavier than air and so does LESS work against gravity than air but is lighter than water and so will do MORE work against gravity than water.

    It will float where the amount of work done by the log against gravity balances the work done against gravity by the air and the water.

    In that situation you can validly say that the density of the water is pushing against the weight of the log but that only works because the densities of the air and the log are less than that of the water and the three forces are in balance.

    If you completely immerse the log in the water then it is the log that has inherent buoyancy because it does more work against gravity due to its relaive lightness and not the water pushing it up.

    If you drop a stone into water do you say that the water pulls it in ?

    No you do not. The stone sinks of its own volition because it has negative buoyancy being heavier than water and therefore doing less work against gravity.

    Similarly lighter objects in a heavier medium rise of their own volition simply because they do more work against gravity at a given temperature.

    Note that the thermal effect of work against gravity is very different for a gas as compared to solids and liquids.

    The molecules of solids and liquids are tightly bound together by the intermolecular force and so do not move apart when work is done against gravity. As a result the temperature change from movement within a gravitational field is miniscule.

    In comparison the temperature change from gases moving within a gravitational field is very large which is why we need the Gas Laws.

  126. Kristian says:

    Stephen, why aren’t you reading the links provided to you? About the 1st Law of Thermodynamics, about ‘heat’ [Q] and ‘work’ [W], and about the adiabatic process respectively. They’re provided for a reason.

    And why do you studiously avoid direct questions to you, but rather just go mumbling something about how we have ‘misunderstood the links’?

    What are you afraid of?

    I’ll ask again: How would you interpret this equation, Stephen?

    ΔU = Q – W

    Let us hear it? What does it say? What is U? What is Q? What is W? How does this equation apply to the adiabatic process?

    Here are the links once more. Please read them:

    http://en.wikipedia.org/wiki/First_law_of_thermodynamics#Description

    http://hyperphysics.phy-astr.gsu.edu/hbase/thermo/heat.html

    http://en.wikipedia.org/wiki/Adiabatic_process#Adiabatic_heating_and_cooling

    Your self-invented physics (you seem to make it up as you go along, to fit with your preferred take on reality) is directly contradicted by ‘established science’, Stephen. And still you just go on and on and on and on …

    Read the links.

  127. Kristian says: November 9, 2014 at 10:09 am

    “Stephen, why aren’t you reading the links provided to you? About the 1st Law of Thermodynamics, about ‘heat’ [Q] and ‘work’ [W], and about the adiabatic process respectively. They’re provided for a reason.
    And why do you studiously avoid direct questions to you, but rather just go mumbling something about how we have ‘misunderstood the links’?
    What are you afraid of?

    I’ll ask again: How would you interpret this equation, Stephen? ΔU = Q – W

    Let us hear it? What does it say? What is U? What is Q? What is W? How does this equation apply to the adiabatic process?”

    Kristian, I will leave it to you and Wayne to attempt to educate Stephen.
    Your, (ΔU = Q – W) is an interesting concept in entropy by some post modern non-scientists.
    The claim is that the difference between energy expended to do work and the energy, (force times distance), transferred accomplishing that resultant act i.e, work integrated over time, must result in an increase in entropy ΔU. In this atmosphere the accumulated entropy U then is simply some Q/(lowest temperature in the, atmosphere), the tropopause. There, all entropy (usless energy) is radiated to space via EMR. The most important function of this atmosphere, is to discard entropy to anything colder, space! It can wonderfully do that, while surface remains suitable for biosphere.
    None of the atmospheric “processes” are adiabatic, every “process” does work; winds, interesting weather, etc, all accumulating entropy at the local coldest. Energy is not conserved in this atmosphere, as all “useless” energy is automagically discarded to space, a completely spontaneous process called thermal EMR. There is no need for the surface to radiate and no possible way for that entropy to move to a higher temperature,like the surface, whatsoever.
    The atmospheric adiabat with no WV is never a process, it is a thermostatic property of N2 and O2 in a gravitational field. That temperature/pressure gradient is the goal of all processes toward that equilibrium state. The atmosphere never makes it, the attempt to that goal, is called weather. There is nothing known about weather, except that it appears to be locally deterministic.
    To determine this, is called meteorology. As Ben can acknowledge, it has less science but more observation than the Farmer’s Almanac. Modern science would prefer a fantasy, to ever admitting “I do not know”. The observers have not the post modern theories, they do have much observation. Consider what happens when theory does not match observation.

  128. Kristian says:

    Will Janoschka says, November 9, 2014 at 12:11 pm:

    “Kristian, I will leave it to you and Wayne to attempt to educate Stephen.
    Your, (ΔU = Q – W) is an interesting concept in entropy by some post modern non-scientists.”

    Will, the 1st Law in the form “ΔU = Q – W” has been with us since Clausius himself. He’s its originator. And it is not about ‘entropy’. It is about ‘internal energy’. [U] is internal energy. [S] is entropy. These are the two cardinal state functions of a thermodynamic system. They are NOT the same. They are separate functions.

    The internal energy of a system (what you would call ‘heat’ or ‘heat content’) changes as you transfer energy to or from it. This energy, in thermodynamics, can be transferred by two different methods. You can transfer it as ‘heat’. This is the [Q] term. Or you can transfer it by doing ‘work’. This is the [W]. Neither Q or W are state functions (like U and S are). They are dynamic functions, processes, transfers of energy. You change the magnitude of the system’s state function, in this case U, by letting the dynamic functions (Q and W) act on it, that is, transferring energy to or from the system.

    This is what this version of the 1st Law is saying. It is one of the fundamentals of Thermodynamics, and is surely not a post-modern invention, Will.

    I am not going to discuss this further with you, because your mental block on this subject is as complete as Stephen’s on most everything concerning atmospheric physical processes.

  129. Kristian,

    Please specify why you think my narrative description is incompatible witrh anything in those links, which I have looked at.

    I cannot see that I have proposed any breach of the first law.

    Work is not a form of energy, it is merely a way that energy can be transferred. Thus applying work to cool the parcel and warm the surroundings is a diabatic transfer of heat and not adiabatic.

    If one applies work against gravity on the other hand then that is adiabatic because no heat is moved from the parcel to the surroundings. Instead the cooling is achieved by changing sensible energy (KE) to non sensible energy (PE).

    There is nothing in your links that runs counter to that as far as I can see.

    I think your problem is that you only recognise sensible energy (KE) and not non sensible energy (PE).

    PE is energy and contributes to the total internal energy of a gas parcel within a gravitational field but you don’t seem to count it as such.

  130. “The first law of thermodynamics is a version of the law of conservation of energy, adapted for thermodynamic systems. The law of conservation of energy states that the total energy of an isolated system is constant; energy can be transformed from one form to another, but cannot be created or destroyed.”

    In light of the above what is the problem with the proposition that the application of work against gravity can transform KE to PE ?

    A gas parcel is an isolated system in the adiabatic scenario so the only way you can comply with the first law whilst cooling when the system is isolated is by transforming the internal energy from sensible to non sensible energy, which is exactly what happens when a lighter less dense parcel rises spontaneously against gravity.

  131. Trick says:

    Stephen 3:40pm: “In light of the above what is the problem with the proposition that the application of work against gravity can transform KE to PE ?”

    The application of work against gravity also changes the gas internal energy p*V term in ways Stephen can’t simply imagine. One has to do the math, the computations, to get nature’s answer.

    “..the only way you can comply with the first law whilst cooling when the system is isolated is by transforming the internal energy from sensible to non sensible energy..”

    No, the p*V term can change internal energy also. In ways Stephen can’t imagine. To make a little easier on the imagination, there is much talk about constant volume processes. And constant pressure processes. Cp and Cv specific enthalpies for gas come from those.

    Simple text book stuff. Stephen can’t imagine these because he doesn’t consult the modern text books; doesn’t have the necessary pre-req.s to comprehend how nature works in a gas parcel.

  132. “No, the p*V term can change internal energy also. In ways Stephen can’t imagine. ”

    What ways would they be ?

    How are such ways relevant ?

  133. Will Janoschka says:

    Kristian says: November 9, 2014 at 2:16 pm

    Will Janoschka says, November 9, 2014 at 12:11 pm:

    (“Kristian, I will leave it to you and Wayne to attempt to educate Stephen.
    Your, (ΔU = Q – W) is an interesting concept in entropy by some post modern non-scientists.”)

    “Will, the 1st Law in the form “ΔU = Q – W” has been with us since Clausius himself. He’s its originator. And it is not about ‘entropy’. It is about ‘internal energy’. [U] is internal energy. [S] is entropy. These are the two cardinal state functions of a thermodynamic system. They are NOT the same. They are separate functions”

    The Clausius 2nd law only involves the spontaneous transfer of energy in the direction of lower potential. Clausius hated the concept of the conservation of anything near this planet. He tried hard to destroy the fantasy concept of “entropy”, to support the 1LTD, with some useless energy divided by some arbitrary temperature. This does not happen in the vicinity of this Earth.

    You are claiming conservation of energy must hold even in a open system like this Earth system. There is no measurement of .EMR from the Sun transferred to the system, or EMR exitance from the system. If there is any difference in the two, “yes” the difference comes from a adjustment of sensible heat (temperature), chemical, latent heat, electrical energy, or mechanical energy ( gravitational PE v.s. mass momentum)!
    Your arbitrary fantasy that “internal” is all but mechanical is a fantasy that has no validity.

    “The internal energy of a system (what you would call ‘heat’ or ‘heat content’) changes as you transfer energy to or from it. This energy, in thermodynamics, can be transferred by two different methods. You can transfer it as ‘heat’. This is the [Q] term. Or you can transfer it by doing ‘work’. This is the [W]. Neither Q or W are state functions (like U and S are). They are dynamic functions, processes, transfers of energy. You change the magnitude of the system’s state function, in this case U, by letting the dynamic functions (Q and W) act on it, that is, transferring energy to or from the system.”

    What total bull shit, the thermal EMR , must be external to your imaginary system. It is spontaneous, and always discards waste energy to the lower temperature.

    “This is what this version of the 1st Law is saying. It is one of the fundamentals of Thermodynamics, and is surely not a post-modern invention, Will”

    Your fake imaginary fantasy of what is in this physical; is only intentional deceit by your post modern non-science.

    “I am not going to discuss this further with you, because your mental block on this subject is as complete as Stephen’s on most everything concerning atmospheric physical processes.”

    I certainly hope so! I do indeed maintain a “block” on total bull shit!

  134. wayne says:

    Dictionary:

    buoyancy (boien-se):

    The upward force that a fluid exerts on an any object that is less dense than itself.

    By showing this definition from a dictionary I assume that it is known by the reader that gases are also termed as a fluid; fluid meaning the ability to flow unlike solids and includes both liquids and gases.

    My words earlier were not quite as smooth and concise as I prefer. Had to call dispatch for backup. I too see nothing mentioned of energy content, pressure, temperature, etc whether something moves up or down or stays still within fluids… only density. My, my… could all of the dictionaries along with all physics texts and lectures all be incorrect?

  135. Trick says:

    Stephen 4:13pm: “What ways would they be ? How are such ways relevant ?”

    Excellent questions. Modern meteorology texts will show by computation & prose exactly. Consult them Stephen. I can try again add to your incomplete imagination using prose (a limited means of communication – no computations).

    As I’ve tried to raise Stephen’s interest in the past, the kinetic theory of molecules as billiard balls was thought good up to the 1860s due to intuition/imagination drawn from solids, liquids. Then gas in boilers/steam engines was found to indicate another gas physics became important. J.C.Maxwell 1868 showed the p*V term is also operating & can’t be ignored – by testing balloons of gas; applied to meteorology, a parcel of gas. This term accounts for the parcel internal energy between molecules which is important in parcel mechanics. This energy is released as balloon/parcel rises, along with KE changing to PE.

    Imagine a birthday party balloon. Helium filled held down by a string, against a wall. You push your finger into the balloon and hold it until T again equalizes with room T. No change in PE. No change in KE. Yet that balloon pushes back on your finger. This is the “springiness” between molecules force that J.C. Maxwell imagined then posted up the computations. Kinetic theory limited to billiard balls became history after that JCM paper got posted up. Stephen hasn’t imagined this force yet, as JCM was able to do.

    This “springiness” results in stored energy (f*d) so has to be accounted for in the parcel rising as it changes with altitude (a function of z height). Stephen can’t just write PE+KE=constant parcel total energy conservation and draw correct conclusions. Because nature of the parcel is using energy conservation as PE+KE+p*V=constant. The energy stored in the “springiness” is released (then stored again on descent) & important to get the right conclusions once the computations are shown.

    Maybe this time I get thru. Dunno. Never succeeded in past.

  136. “The upward force that a fluid exerts on an any object that is less dense than itself.”

    Then explain why low pressure develops beneath rising warm air.

    For a fluid to actively push a lighter object upward it needs to produce higher pressure beneath the lighter object in order to start the process and maintain that higher pressure to keep it going.

    That is not what we observe.

    That definition of buoyancy is limited to the scenario where an object is floating on a fluid surface and as I pointed out above it does make sense in that context but only because the density of the water is pushing up against both the weight of the air above plus the weight of the object to produce a balance that we describe as flotation.

    Once one immerses the lighter object completely the scenario is quite different. Then, the lighter object rises through the denser medium due to its own inherent buoyancy arising from its lighter weight.

    In that situation it leaves lower pressure behind it as it pushes up through the denser medium and that explains low pressure beneath rising air.

    Even in your dictionary definition the density of the water is only passively resisting the weight placed upon it. There is no active pushing upward from the denser medium.

  137. Trick says:

    Will 4:14pm: “You are claiming conservation of energy must hold even in a open system like this Earth system.”

    A good claim borne out by much experimentation. Energy IS conserved in the earth system. No energy is created or destroyed in the vicinity of this earth.

    “There is no measurement of .EMR from the Sun transferred to the system, or EMR exitance from the system.”

    Incorrect Will. So yes. There is. Various CERES/ERBE et. al. are the precision instruments doing/have done that measuring as you’ve been shown before. There is difference between the two EMRs. See slide 4 here, note spell atmosphaere is important:

    Click to access GC_Loeb.pdf

  138. Trick said:

    “Imagine a birthday party balloon. Helium filled held down by a string, against a wall. You push your finger into the balloon and hold it until T again equalizes with room T. No change in PE. No change in KE. Yet that balloon pushes back on your finger.”

    Pushing in the finger increases pressure in the same way as descent to a lower altitude and so temperature increases with PE converting to KE. The fact that after a while the extra heat dissipates is not relevant because you have not permitted the air in the ballon to rise against gravity to a new height where its temperature would match the ambient temperature again. Furthermore you have stopped pressing which allows time for the extra KE to dissipate to the surroundings. In the real world the uplift and descent are a continuous process.

    “This term (p*V) accounts for the parcel internal energy between molecules which is important in parcel mechanics. This energy is released as balloon/parcel rises, along with KE changing to PE.”

    That would be the intermolecular force between molecules which is included in the KE to PE conversion process. As the parcel rises and expands the molecules move further apart which converts energy in the intermolecular force to PE and contributes to the cooling.

  139. wayne says:

    “…. low pressure develops beneath rising warm air.”

    No, first answer this: who or what put that thought that into you mind Stephen?

    Not going to blindly fight your ghosts any longer. Give me some real substance that the statement is based upon; a link, an excerpt from a book and the book’s name… something concrete.

  140. wayne says:

    Oops, looks like I have an extra ‘that’ inserted… please ignore.

  141. Trick says:

    Stephen 7:57pm: Pushes back against J.C. Maxwell 1868 paper as usual. Molecules are no longer simple billard balls after that Stephen.

    “The fact that after a while the extra heat dissipates is not relevant because you have not permitted the air in the ballon to rise against gravity to a new height where its temperature would match the ambient temperature again.”

    No Stephen, you imagine yet again incorrectly. Showing the computations would prove it, see a modern text book of your choosing.. The balloon gas is at the same height no PE change. The kinetic energy of the initial f*d displacement is allowed to dissipate. No gas KE change. No changes. YET there is a force remaining. How so? Explain this force using simple billiard balls analysis. Stephen can’t. This is the force Stephen’s imagination misses in parcel mechanics. Pistons and cylinders do not inhabit the atmosphere.

    “That would be the intermolecular force between molecules which is included in the KE to PE conversion process.”

    Nowhere is that force seen in PE+KE=constant as used in Stephen’s imaginations and previous to 1868. It took JCM to show total internal gas parcel energy as PE+KE+p*V = constant conserved quantity.

  142. wayne says:

    Retake… my last comment sucked! Multiple errors and too harsh.


    …. low pressure develops beneath rising warm air.

    No Stephen, first answer this: Who or what put that thought into your mind?

    Sorry, not going to blindly fight your ghostly imaginations any longer. Give me some real substance that this statement is based upon; a link, an excerpt from a book and the book’s name… something more concrete.

  143. “…. low pressure develops beneath rising warm air.”

    Every low pressure cell around the planet is a consequence of warmed air rising.

    Every convective cell is the same on a smaller scale.

  144. “YET there is a force remaining. How so?”

    Well you stuck your finger into a stretchy rubber membrane and that membrane will try to resist the pressure from your finger as long as you leave it there.

    “The balloon gas is at the same height no PE change”

    Applying pressure to a gas regardless of height converts PE to KE since that pressure reduces the distance between molecules.

  145. wayne says:

    You didn’t answer my question, so I again repeat:

    “…. low pressure develops beneath rising warm air.”

    No Stephen, first answer this: Who or what put that thought into your mind?

    Sorry, not going to blindly fight your ghostly imaginations any longer. Give me some real substance that this statement is based upon; a link, an excerpt from a book and the book’s name… something more concrete.

  146. “It took JCM to show total internal gas parcel energy as PE+KE+p*V = constant conserved quantity.”

    All that has happened there is that the PE arising from the change in elevation has been separated from the PE arising from the change in intermolecular distances for the purpose of a more detailed equation.

    The net result is exactly the same.

    PE plus KE = constant with the p*V function incorporated in the PE element is equally valid.

  147. wayne,

    Every observation and every textbook demonstrates that rising warm air produces low pressure beneath it.

    What do you think a cyclone is ?

    It is a low pressure region with rising warm air at the centre.

  148. “In a cyclone the central air pressure is lower than that of the surrounding environment, and the flow of circulation is clockwise in the Southern Hemisphere and counterclockwise in the Northern Hemisphere. Cyclones are also characterized by low-level convergence and ascending air within the system. ”

    http://www.scholastic.com/teachers/article/cyclone-and-anticyclone

    Come on wayne, you can’t be unaware of this basic stuff.

  149. wayne says:

    So you now immediately jump contexts from innocent single thermals now to roaring cyclones… how classic of you. Ok, no problem, has it never dawned on you that with warm low pressure is also low density and what happens when you have the lowest density at the center of your cyclone?

    Besides you still didn’t answer my original question, so I repeat till you answer:

    “…. low pressure develops beneath rising warm air.”

    No Stephen, first answer this: Who or what put that thought into your mind?

    Sorry, not going to blindly fight your ghostly imaginations any longer. Give me some real substance that this statement is based upon; a link, an excerpt from a book and the book’s name… something more concrete.

  150. Roaring cycles are just large thermals that have lasted long enough to acquire rotation.

    I don’t see why the link I gave you is not the answer to your question.

    I know that the lower density comes first and the lower density air rises spontaneously against gravity because it is lighter than the surrounding denser air.

    The surrounding denser air cannot be doing any pushing otherwise there would be high pressure beneath the ascending air doing the pushing.

    What gave you the idea that there is high pressure at the base of ascending air pushing it upward ?

  151. Should be roaring cyclones, not roaring cycles, obviously.

  152. Consider the convective cycle as a whole.

    Before there can be any descending air producing high pressure at the surface there must first be rising air somewhere else to start the cycle.

    Uplift comes first with descent to the surface only coming later when the circuit completes.

    Thus the initial rise must be spontaneous because at that point there is no adjoining high pressure cell to provide any push.

  153. Here is the difference between spontaneous convection and forced convection:

    http://physics.info/convection/summary.shtml

  154. Trick says:

    Stephen 9:20pm: “Applying pressure to a gas regardless of height converts PE to KE since that pressure reduces the distance between molecules.”

    Yes. Stephen you are on to something important here. The “springiness” between molecules physics; they are not just like billiard balls.

    9:26pm: “All that has happened there is that the PE arising from the change in elevation has been separated from the PE arising from the change in intermolecular distances for the purpose of a more detailed equation.”

    There is more happening now Stephen. The JCM added p*V detail was just what was needed. Since there is no change in balloon elevation, no change in KE, PE changes come from p*V physics that JCM demonstrated. That detail causes correct conclusions about parcel mechanics (namely the Poisson relations) in computations. This is the change from molecules as simple billiard balls to total internal energy=KE+PE+p*V that came from the gas parcel physics JCM discovered by the same sort of balloon test and was reported in 1868.

    “The net result is exactly the same. PE plus KE = constant with the p*V function incorporated in the PE element is equally valid.”

    Net result IS different Stephen, the text book computations you ignore show it. p*V PLUS KE + PE (from height) = total energy conserved in parcel. Stephen’s original PE change came ONLY from height (namely incorrect PE+KE=constant for parcels.)

  155. Kristian says:

    wayne,

    I fear you’re going to have to start aaall the way from the beginning with Stephen on this one. Like: What is actually atmospheric pressure? Is it somehow a quality of the air itself? Seems to me he believes just that.

  156. Ben Wouters says:

    Stephen Wilde says: November 8, 2014 at 3:29 pm

    ” “The higher the temp. the more vigorous the molecules bump into each other, increasing the internal pressure, thus expanding the total atmosphere.
    Since the mass doesn’t change, the surface pressure remains the same.”

    Yes, you’ve got it.”

    Nothing to “get” here. This is in chapter one of “Meteorology for Dummies” (if it exists).
    This should be common knowledge for anyone discussing our atmosphere.

  157. Ben Wouters says:

    wayne says: November 8, 2014 at 10:03 pm

    ” What you said of the atmosphere when at less that 3 K was correct, and it would be a sea of liquid nitrogen, oxygen and argon sitting on the original surface but try this thought out”
    Not my expertise, but wouldn’t it be a ‘ solid’ atmosphere at 0K ?

    ” What “internal pressure” exactly?”
    The same pressure that increases when you heat an enclosed volume of air.
    In the atmosphere the increased pressure from increasing total energy results in an expansion of the atmosphere, creating a new hydrostatic balance situation.

    Pressure at any altitude in the atmosphere is caused by the weight (mass x gravity) of the column above. At the surface this pressure is always the total weight of the column above.
    Consider the 500 hPa level. (= roughly half pressure)
    In the fluid (or solid?) atmosphere it is halfway the height of the fluid (not compressible)
    In your 150K atmosphere it is much lower than presently.
    But in all case it is at the altitude where half the weight of the atmosphere is below and the other half above it.
    So yes, density changes in the above cases. Density also changes with increasing altitude in the atmosphere (compressible)

    http://en.wikipedia.org/wiki/Hydrostatic_equilibrium has this image:
    http://en.wikipedia.org/wiki/Hydrostatic_equilibrium#mediaviewer/File:Hydrostatic_equilibrium.svg

    Where does the upward force come from in your opinion?
    Same as for the sun, why doesn’t it collapse due to its own gravity?
    Answer is thermal pressure.

  158. Ben Wouters says:

    Stephen Wilde says: November 9, 2014 at 7:33 pm

    ” For a fluid to actively push a lighter object upward it needs to produce higher pressure beneath the lighter object in order to start the process and maintain that higher pressure to keep it going.

    That is not what we observe.”

    That is exactly what we DO observe.
    For any parcel or layer the pressure from below is always higher than the pressure from above.
    Pressure REDUCES with altitude. (Or INCREASES with depth for the oceans, same thing)

    If the weight of the parcel or object is lower than the pressure differential it will rise until weight and press. diff. are equal again.

  159. Konrad. says:

    Stephen Wilde says:
    November 9, 2014 at 10:18 pm
    ///////////////////////////////////////////////////
    “Consider the convective cycle as a whole”

    Stephen, vertical convective circulation in a gravity field (for a fluid maintaining an steady temperature distribution) requires energy loss from the fluid higher than energy gain. This is a simple 2D CFD model of one of my gas column experiments –

    -on the left the gas is heated at low and cooled high. On the right heating and cooling are at disparate locations low down, convection stalls and the gas heats.

    Adiabatic cooling of air masses on accent cannot cause buoyancy loss needed to complete the tropospheric convective cycle. For this diabatic energy loss through radiation to space is required. Yet your modelling shows this circulation speeding up for decreased radiative gases. This claim is ayphysical.

  160. Ben,

    I know that pressure decreases with height but it is the relative pressures that matter not absolute pressures.

    Relatively low surface pressure forms beneath warmer lighter rising air which demonstrates that the lighter air is rising spontaneously and not being pushed up by relatively high pressure beneath it.

    Konrad,

    “Adiabatic cooling of air masses on ascent cannot cause buoyancy loss ”

    Yes it does because the atmosphere always cools with height even if there is no radiative loss to space because KE becomes PE with height.

    Your experiment is invalid because you need an unevenly heated surface on a rotating sphere open to space to simulate an accurate real world result.

    Trick,

    “Since there is no change in balloon elevation, no change in KE”

    There is a change in the relative proportions of KE and PE from the increase in pressure caused by your finger which converts some of the PE to KE

    However that is a diabatic process forced from outside the parcel and if you hold the parcel at the same height then the extra KE formed from some of the PE is free to dissipate to the surroundings until thermal equilibrium is regained despite the increase in pressure.

    If you then move the parcel up or down then the process is adiabatic because gravity acts on both the parcel and the surroundings simultaneously. PE changes to KE at the same rate in both the parcel and the surroundings.

    As the parcel moves down, becoming denser and warmer, it constantly moves into regions that are denser and warmer and so at the same pressure and temperature as the parcel itself.

    The pressure and temperature of the parcel changes at exactly the same rate as the pressure and temperature of the surroundings.

    The result is that there will never be any pressure or temperature differentials arising between the parcel and the surroundings except for any that existed before the uplift or descent began.

    If there is no pressure differential there can be no work done.

    If there is no temperature differential there will be no transfer of KE as heat.

    That is why there is no exchange of energy between the parcel and the surroundings in adiabatic uplift and descent

    Are there really no bulbs lighting up yet ?

  161. Trick says:

    Stephen: There is no change in KE or PE. The proportions therefore cannot change. The p*V term is entirely where the information changes.

  162. Kristian says:

    Ben Wouters says, November 10, 2014 at 12:46 am:

    “” What “internal pressure” exactly?”
    The same pressure that increases when you heat an enclosed volume of air.
    In the atmosphere the increased pressure from increasing total energy results in an expansion of the atmosphere, creating a new hydrostatic balance situation.”

    This is a trickier subject than you would think at first, where you really need to mind your p’s and q’s as you move along. Watch your every step carefully, so to say.

    Because, what is ‘pressure’ anyway? It is simply force per area. The subtle point here is, that the force in question needs an actual something to be applied to. That’s why it’s fairly easy to picture in your mind the internal pressure produced by a gas enclosed by an outer wall of some kind, an actual physical boundary. The gas molecules flying around simply bounces off the walls, exerting an overall mean force on it. This force, when divided by the area of the surrounding wall, gives you the internal gas pressure. If you heat the gas, the gas molecules will gain kinetic energy and fly around faster inside the enclosure, thus when hitting the surrounding wall, the impact will be more powerful and the total force will be greater. To restabilise, the enclosure will have to expand until the point where the larger force is spread across a correspondingly larger wall area. Basic stuff, I’m sure you would agree …

    In the atmosphere, however, this all becomes a bit less intuitive. There are no solid, macroscropic barriers (enclosing ‘walls’) as such in the atmosphere.

    And this is where I can sympathise with wayne’s objection to the whole “air parcel” routine. Because an ‘air parcel’ as defined by meteorology is NOT like the air inside a balloon. It is purely a conceptual tool. A useful one, by all means. I use it all the time. But one should never forget that, in reality, it isn’t a real, separate thing. It’s not an object.

    When energy comes off the ground as heat, it simply spreads up and away until it’s all absorbed by the air, and wherever and whenever each little packet of it gets thermalised, individual air molecules get sped up and start hitting its neigbours a bit harder than before. This is a continuous and dispersive process and is not limited to any specifically defined volume of air.

    This is why I would tend to agree with wayne that ‘density’ (mass per volume) is a much more practical property to work with than ‘internal pressure’ when describing a free, natural convective process.

    Convection is also, after all, explained by the use of differences in ‘density’ rather than differences in ‘pressure’.

    ‘Buoyancy’ is also fully explained with external (surrounding) pressure plus internal gas density. Even for a balloon. No real need to invoke ‘internal pressure’.

    Heh, if you bear with me in this one, the buoyancy force acting on this ‘cubical balloon’ is the vector sum of all its external pressure arrows. It goes up. In hydrostatic balance, it is however countered exactly by the weight (mass times gravity) of the ballloon itself, including the air inside.

    If you heat the air inside the balloon, it will expand and the outside pressure will be applied across a larger area. Since the pressure at the bottom is always higher than at the top, increasing the volume of the balloon and thus its outward area, will lead to a greater total upward force acting on the balloon.

    And the balloon will rise.

  163. Kristian says:

    BTW, surface low and high pressure regions do NOT have anything to do with the internal or inherent pressure of the air at the surface. As Stephen seems to think. It has to do with how much air is ultimately stacked on top of a particular part of the surface, or rather, how heavy it is in total. Surface air pressure is solely an expression of the weight of the total air column above it. I find it peculiar that Stephen, a self-proclaimed student of meteorology for 60 years (!), doesn’t appear to know this …

  164. If the balloon doesn’t change height then yes the intermolecular forces (p*V) are entirely where the information changes when you apply pressure from your finger to the balloon but that is miniscule compared to the effect of movement of the entire parcel against gravity.

    Furthermore it does involve changes between KE and PE:

    “If two molecules in a solid are at absolute zero, they have no kinetic energy and their separation is ro . Now consider the two molecules at a higher temperature, with a shared kinetic energy ε . This energy creates an imbalance between the attractive and repulsive forces.

    B r maximum molecules attracted PE max, KE zero

    BC PE converted into KE

    C r = ro equilibrium position PE min KE max

    CA KE converted into PE

    A r minimum molecules repelled PE max, KE zero

    AC PE converted into KE

    C r = ro equilibrium position PE min KE max

    CB KE converted into PE

    In solids molecules can only vibrate small distances about fixed positions. The reason is that the kinetic energy ε is much smaller than the potential energy εo (approx. 10%) . Hence a solid has a fixed shape and volume. ”

    from here:

    http://a-levelphysicstutor.com/matter-mol-force-pe.php

    In gases molecules can vibrate larger distances about moving positions but the same principles apply save that the potential energy of gases is much larger in relation to their kinetic energy as compared to solids.

  165. Trick says:

    Stephen: There are no changes in PE or KE at equilibrium. The p*V term carries all the information in a gas about the added finger force and is certainly not miniscule; p*V is miniscule in solids and liquids not a gas. Find s wiki link on enthalpy and read the cites.

  166. “Surface air pressure is solely an expression of the weight of the total air column above it.”

    Quite so.

    Warmer, expanded, lighter air above the surface means less mass and less weight above the surface and thus relatively low pressure at the surface just as I said.

    Convection is spontaneous with no pushing required once the sun warmed surface passes conducted energy to the air above it thereby reducing density and making it lighter.

    http://en.wikipedia.org/wiki/Atmospheric_convection

    Nothing there about warmer, lighter, less dense air needing to be pushed up by another force.

    The buoyant force is clearly expressed to be within the warmed air and so there is no pushing from outside the warmed air.

  167. Trick,

    You altered the equilibrium by sticking your finger in. The ratio of PE and KE adjusted accordingly.

    The KE dissipated to the surroundings because you stopped the more compressed air in the balloon from descending to a warmer location.

    I’ve never before come across such desperate thrashing about trying to ignore established science.

  168. wayne says:

    Heh, if you bear with me in this one, the buoyancy force acting on this ‘cubical balloon’ is the vector sum of all its external pressure arrows. It goes up. In hydrostatic balance, it is however countered exactly by the weight (mass times gravity) of the balloon itself, including the air inside.

    If you heat the air inside the balloon, it will expand and the outside pressure will be applied across a larger area. Since the pressure at the bottom is always higher than at the top, increasing the volume of the balloon and thus its outward area, will lead to a greater total upward force acting on the balloon.

    Finally — you got the cookie! 😉

    Unfortunately since Stephen either can’t fathom or refuses to fathom density or just plain doesn’t like the variable and is never going to admit it’s existence, after you put all of the factors of pressures on the sides which all cancel and the always greater pressure at the bottom than at the top plus the volume and masses involved, you end up with…tada… density… the density difference and this is the easier of other ways to calculate and compare to see if something immersed within a liquid rises, sinks, or just floats motionless.

    Thanks Kristian for seeing through my words.

    There is more about buoyancy and thermals and that brings in some other real concepts, acceleration and velocity of mass once there is an unrestricted net buoyancy force difference existing. At the initialization of a thermal Stephen has the very first action correct bit no further. Solar energy hits a dark (lets say huge black parking lot) surface and warms the air which expands lowering its density and wala, you have a buoyancy misbalance. But this only causes acceleration at the very first as that air speeds upward pulling in cooler more dense air (which now starts warming) in to take it’s place. But that acceleration only happens for a short distance as terminal upward velocity is reached. From that point upward and onward there is no further acceleration implying no further net upward force, the resistance force vector pointing downward from the velocity now matches and cancels any buoyancy force upward.

    But as long as the sun shines down onto the parking lot you have continuous warming… velocities have already been established upward and radially inward at the bottom and for those velocities to slow to a standstill it would require some opposite force to accelerate in the opposite direction to slow the velocity and that simply does not happen without a cloud breaking the solar input. What does happen is you have a continuous stream of air moving upward exactly as smoke rising from the coal at the end of a cigarette and that is a great way to do some mini-experiments very inexpensively. Put two of your fingers very slowly next to the smoke thermal about six inches above the coal and you will see little effect but some Bernoulli bending it. Do this same thing just next to the coal and you will disrupt the inflow velocity at the bottom that is feeding this column which now has contant velocity in action.

    Not packets and parcels Stephen, streams. Sailplane pilots totally rely on that entire process every good weekend.

  169. “But that acceleration only happens for a short distance as terminal upward velocity is reached. From that point upward and onward there is no further acceleration implying no further net upward force”

    I’m having a problem seeing how that differs from what I said but you have missed out the role of the adiabatic process in maintaining uplift once it has started.

    The initial acceleration is caused by conduction from the surface which involes no pushing from anywhere. After that there is no further net upward force i.e. nothing pushing it upward.

    Instead, the upward speed is maintained by adiabatic processes because pressure and temperature declines with height both in the parcel or stream and in the surrounding air at the same rate.

    The original pressure and temperature difference (density) between the parcel/stream and the surroundings is maintained throughout the adiabatic process so that uplift continues without any further exchange of energy between the parcel/stream and the surroundings until a warmer layer is reached whereupon the temperature and pressure within the parcel/stream is able to reach equilibrium with the warmer surroundings at the higher level.

    “pulling in cooler more dense air (which now starts warming) in to take it’s place. ”

    Exactly. The uplift pulls in cooler more dense air into the region of lower pressure beneath the thermal therefore that cooler denser air is not doing any pushing.

    Will you please now concede that the lighter air rises spontaneously as a result of warming from the surface plus adiabatic uplift and is not ‘pushed’ up ?

  170. Note that the continued rising of the warmer lighter air is not dependent on more warmed air coming up behind it in the stream that you proposed.

    Even if the source region went into shadow the rising air would simply detach from the stream into a parcel and continue rising as long as the lapse rate allowed continuing adiabatic uplift.

    So again, no pushing from below.

  171. wayne says:

    Kristian, while we happen to be exactly on this topic of density let me shoot you a rather abstract concept and see what you think of it and maybe some implications I cannot see yet You seem good at letting your mind handle the abstractness.

    Ok, all matter in this universe is found in moons, planets and suns and on each of these you find density is sorted from top at the edge of free space and the bottom at the center of the gravity well. Seems nature always wants the maximum mass per volume to be as close to the center of the gravity well as possible. I already guess you see that. Solids are the only matter that prevents that to occur in places, like my chair… if it were liquid it would be a puddle pulled by gravity onto the floor.

    But now take our thoughts on buoyancy. Energy tends to always (on the average) expand any mass lowering its density and this density difference will initiate a sort to put them back where they belong in the gradient. But that brings to mind that nature also tends to move any excess energy that is not already within the matter causing its density and that excess energy is always moved upward out of the gravity well because of density differences.

    I think that is enough for you to see my gist. Any thoughts? Could this be the overriding reason any planet cannot dictate its own temperature? That thought hit my mind and I just had to express that to someone. Never have heard such a concept raised in this ‘climate’ discussion.

  172. Trick says:

    Stephen: 5:27pm: “You altered the equilibrium by sticking your finger in. The ratio of PE and KE adjusted accordingly.”

    No alteration in energy equilibrium. Once the balloon returns to room temperature, there is no change in KE and the balloon didn’t move up or down so no change in PE. So no change in their ratio.

    Stephen’s concept of gas molecules as billiard balls total energy = KE+PE fails to detect my finger poking in the balloon surface altering the non-miniscule p*V term. This is exactly what JCM noticed & published in 1868. This is why internal energy (KE+PE) works close enough for solids and liquids & billiard balls; have to use enthalpy (KE+PE+p*V) in meteorology for the natural total energy behavior of a gas. The p*V term captures the force between molecules. Basic text book stuff Stephen, no thrashing; you would learn if only you would read up.

    Stephen continues on same incorrect meteorological direction (parcel TE=PE+KE) to fulfill a political agenda not a science agenda; evidence: Stephen would actually read a text book to find the science (parcel TE=PE+KE+p*V) if Stephen was not covering for a political agenda.

  173. wayne says:

    Stephen: “Exactly. The uplift pulls in cooler more dense air into the region of lower pressure beneath the thermal therefore that cooler denser air is not doing any pushing.”

    You are just simple incorrect there. With high density comes high pressure. The pressure below is also maintained or even increased by the velocities of the inflow radially converging upon a single center point, pushing inward from all directions and that raises pressure also. It possibly takes some calculus to see this negative divergence but it is there. Besides the inflowing is cooler than the warm air just moving upward and that too makes it more dense than the warmer less dense above. Two cases are saying more dense, higher pressure Stephen whether you can visualize it or not. If pressure below was lower you would kill the thermal and I guarantee you that does happen to sailplane pilots, it’s called ‘losing the thermal’ or ‘that thermal just died’. Just don’t always think of thermals as if sending smoke signals with a blanket above a camp fire. I’ve never experienced but rarely such on-and-off thermal action even though I flew for about three or four years in sailplanes years ago. Observations trump for me. Ever play spades?

    One other concept you seem to have incorrect is just how high these normal everyday thermals (not cyclones, tornadoes or towering thunderstorms) go above the surface… not far. I have ridden one up from 1000 ft (330 m) to at most 6000 ft (2 km) but no further… there or lower they all die. On moist days there are cumulus clouds with silky gray bottoms at the top of each but even then the cloud base is rather low when comparing to the entire troposphere. That is the mixing layer. The hydrostatic balance has a huge role from there further upward and is so gentle. It take water vapor to fire them up higher but then we are speaking of much rarer thunderstorms and frontal systems.

    Since you like meteorology look more into why, yes, in calculus with gradients, divergence and curls, these lows and high centers even exist before you start imagining too much on that topic. I read a great meteorology article on that very topic here on the web a couple of years ago but sorry, I saved no link. I put the knowledge into my head then move on.

  174. Trick said:

    “Once the balloon returns to room temperature, there is no change in KE and the balloon didn’t move up or down so no change in PE. So no change in their ratio.”

    If you are still pressing when the extra KE has gone the pressure is still higher than it was before so your continuing manual forcing is still decreasing PE by pushing the molecules closer together than they were at ambient pressure. Your finger has altered the p*V term but not the gravitational PE term.

    No political agenda, by the way. Sign of desperation ?

    wayne said:

    “The pressure below is also maintained or even increased by the velocities of the inflow radially converging upon a single center point, pushing inward from all directions and that raises pressure”

    Only if it flows in faster than the uplift takes air away. Generally that will happen when surface heating below the thermal declines and then the thermal dies. As long as air is rising faster than it flows back in there will be lower surface pressure under the thermal (or cyclone) as compared to the surrounding areas.

    “One other concept you seem to have incorrect is just how high these normal everyday thermals (not cyclones, tornadoes or towering thunderstorms) go above the surface… not far”

    Sailplane pilots prefer days when the inversion layer on a fine day is not too high (maybe up to 2km) The best conditions are fine weather under an anticyclone (descending air higher up) but with enough room for thermals to form between the surface and the inversion layer.

    Those thermals are still basically just miniature cyclones and your view of them is skewed by your sailplane experience which limits your observations to benign fairweather thermals.

  175. Trick says:

    Stephen 8:21pm: “Your finger has altered the p*V term but not the gravitational PE term.”

    Yes. Stephen capitulates. Thank you Stephen. Finally. Now stay the course:

    Parcel TE=KE+PE no detection of alteration in p*V so incorrect.
    Parcel TE=KE+PE+p*V detects the finger push, so parcel enthalpy is correct for gas.

    *******

    Stephen 6:45pm: “The initial acceleration is caused by conduction from the surface which involves no pushing from anywhere.”

    Here Stephen’s imagination really is from disney.com (an entertainment and political company).

    After missing JCM’s post adding p*V to a gas parcel total energy in 1868, Stephen now misses what Newton posted up in the 1600s: F=ma.

    Newton: If there is non-zero acceleration, then the push (F) is non-zero. The push is from parcel buoyancy.

    Buoyancy. A fluid (at rest) with density rho in a volume V is acted on by 2 equal and opposite forces: its weight (rho*g*V) and the integral of the pressure p over the surface of V. If the fluid in V is replaced by a fluid with a different density rho’ in such a way that the pressure distribution is unchanged, a net force acts on the volume, the buoyancy force.

    If rho’ less than rho, the buoyant force is upwards, positively buoyant. If rho’ greater than rho, the buoyant force is downwards, negatively buoyant.

  176. The push up is from buoyancy within the parcel NOT from an external force. It is the proposition that a force external to the parcel pushes the parcel up which is in dispute.

  177. “Parcel TE=KE+PE+p*V detects the finger push, so parcel enthalpy is correct for gas.”

    The p*V term still represents a change in PE though and one can combine the gravitational PE and p*V for the intermolecular PE into the same term so that KE+PE=constant is also correct.

    Trick said there was no change in PE which is wrong because the intermolecular PE changed.

    There is no need to disinguish between the two types of PE when considering the adiabatic process so Trick’s contribution is pure obfuscation.

  178. Trick says:

    Stephen: 1st Read again about the buoyant force. Then realize Trick said there was no change in PE which is correct because the intermolecular PE did not change. The intermolecular force changed, not the intermolecular PE.

  179. wayne says:

    Stephen, do you not realize that there is buoyancy force trying to lift you up into the air right now? No joke Stephen, it’s physics. You are mass and have volume and m/V is density and you are immersed within a liquid too. Is not the reason you do not float away because you are too dense for the upward buoyancy force on your body? Lighten up enough and you would do just that, float away with no force from you. Why will you never read and study instead of flapping your fingers on the keyboard telling everyone else they do not know what they are talking about?

    Well, speaking of reading, studying and writing I must get back where I was before this thread. It has been quite enlightening on multiple fronts.

  180. wayne,

    The density of my body produces negative buoyancy.

    The bouyancy whether negative or positive exists within the mass being considered and is related to its density.

    I have referred you to multiple sources that convection above a heated surface is spontaneous due to the lower density caused by that heating.

    The surrounding air does not push the lighter air up. It rises against gravity of its own volition simply because it is lighter.

  181. “As we bring two hydrogen atoms together, the potential energy falls to a minimum”

    http://www.chem1.com/acad/webtext/states/interact.html

    The PE in the balloon did fall as a result of poking the finger in and leaving it there. Forcing molecules closer together by applying an external force reduces PE.

  182. Kristian says:

    Stephen Wilde says, November 10, 2014 at 9:27 pm:

    “The surrounding air does not push the lighter air up. It rises against gravity of its own volition simply because it is lighter.”

    Kinda reminds me of this one 😉 :

  183. Kristian says:

    wayne says, November 10, 2014 at 7:04 pm:

    “But now take our thoughts on buoyancy. Energy tends to always (on the average) expand any mass lowering its density and this density difference will initiate a sort to put them back where they belong in the gradient. But that brings to mind that nature also tends to move any excess energy that is not already within the matter causing its density and that excess energy is always moved upward out of the gravity well because of density differences.

    I think that is enough for you to see my gist. Any thoughts? Could this be the overriding reason any planet cannot dictate its own temperature? That thought hit my mind and I just had to express that to someone. Never have heard such a concept raised in this ‘climate’ discussion.”

    Are you perhaps referring to something along the lines of what Sir George Simpson of the Royal Meteorological Society pointed out to Guy Callendar in 1938:

    “…but he would like to mention a few points which Mr. Callendar might wish to reconsider. In the first place he thought it was not sufficiently realised by non-meteorologists who came for the first time to help the Society in its study, that it was impossible to solve the problem of the temperature distribution in the atmosphere by working out the radiation. The atmosphere was not in a state of radiative equilibrium, and it also received heat by transfer from one part to another. In the second place, one had to remember that the temperature distribution in the atmosphere was determined almost entirely by the movement of the air up and down. This forced the atmosphere into a temperature distribution which was quite out of balance with the radiation. One could not, therefore, calculate the effect of changing any one factor in the atmosphere …”

    Any radiative (or conductive) attempt at perturbing the gravity-based temperature gradient away from the solar-heated surface, will always be thwarted by buoyancy restoring the balance.

  184. wayne says: November 10, 2014 at 9:14 pm

    “Well, speaking of reading, studying and writing I must get back where I was before this thread. It has been quite enlightening on multiple fronts.”

    Indeed, perhaps some day we can determine the actual role of EMF on and about this planet.

    Kristian says: November 10, 2014 at 11:00 pm

    “Are you perhaps referring to something along the lines of what Sir George Simpson of the Royal Meteorological Society pointed out to Guy Callendar in 1938:

    “…but he would like to mention a few points which Mr. Callendar might wish to reconsider. In the first place he thought it was not sufficiently realised by non-meteorologists who came for the first time to help the Society in its study, that it was impossible to solve the problem of the temperature distribution in the atmosphere by working out the radiation. The atmosphere was not in a state of radiative equilibrium, and it also received heat by transfer from one part to another. In the second place, one had to remember that the temperature distribution in the atmosphere was determined almost entirely by the movement of the air up and down. This forced the atmosphere into a temperature distribution which was quite out of balance with the radiation. One could not, therefore, calculate the effect of changing any one factor in the atmosphere …”

    /:Any radiative (or conductive) attempt at perturbing the gravity-based temperature gradient away from the solar-heated surface, will always be thwarted by buoyancy restoring the balance.”

    Interesting concept, the Please state Gus Kirchhoff’s meaning of “radiative equilibrium”, and the location by location “state” of this atmosphere with respect to radiative equilibrium. Only then can we start to understand the actual process of radiative exitance from this planet. It is nowhere close to the two stream approximation proposed by Climate Clowns and other post modern idiots.

  185. wayne says:

    Kristian, once again I have started a thought without thinking it all the way through of how to explain this to someone else, to see it as clear as myself. I appear to have maybe lost you a bit on the the first two steps. This thought gets into state functions and energy density per volume very much like mass per volume that even supports our atmosphere or any atmosphere and demand the stratification by density and a the bodies atmosheres are athe height (maybe think scale height) that they are. That is, why not some other height? Why do the heights not vary too far from a mean? Is there physics or thermodynamics involved that dictates for x solar energy and y composition and z mass are the heights pre-determined. Those thoughts seem to conflict with Sir George Simpson’s last statement. Let *me* think on this for a while first.

  186. Ben Wouters says:

    Kristian says: November 10, 2014 at 4:48 pm

    ” Ben Wouters says, November 10, 2014 at 12:46 am:

    “” What “internal pressure” exactly?”
    The same pressure that increases when you heat an enclosed volume of air.
    In the atmosphere the increased pressure from increasing total energy results in an expansion of the atmosphere, creating a new hydrostatic balance situation.””

    I did hope it was obvious that I’m talking about the internal pressure of the TOTAL atmosphere.
    Why do you think the atmosphere is not collapsed on the surface due to gravity?
    INTERNAL PRESSURE.
    And this is very much like a volume of air enclosed by a solid structure.
    The Earths surface is the bottom. Since the atmosphere surrounds the entire planet it forms its own walls. The top is a bit more complex, it is formed by GRAVITY..
    So yes the TOTAL atmosphere expands if the total internal energy increases.

    Can we now finally consider ‘hydrostatic balance’ for the TOTAL atmosphere as discussed and move on to convection?

  187. Ben Wouters says:

    Kristian says: November 10, 2014 at 5:01 pm

    ” Surface air pressure is solely an expression of the weight of the total air column above it.”
    Basically agree, but I like to see a calculation for the effect of the descending air in a high pressure area. The mass of all this air ‘crashing’ onto the surface must have a pressure increasing effect.
    Just don’t know much.

  188. Ben Wouters says:

    Stephen Wilde says: November 10, 2014 at 5:24 pm

    ” Warmer, expanded, lighter air above the surface means less mass and less weight above the surface and thus relatively low pressure at the surface just as I said.”
    Wrong again. With a surface pressure of eg 1013 hPa the weight of the column above the surface at the equator or at a pole is exactly the same.
    The column at the equator is expanded much more however.

  189. Ben Wouters says:

    wayne says: November 10, 2014 at 6:15 pm

    Not sure if we’re on the same page regarding convection.
    Thought experiment:
    Iso solar heating the surface we cover an earthlike planet with a floor heating system (FHS).
    (perhaps a shallow layer of water on it to get wv in the air)
    We regulate the whole surface to 290K. When stabile the whole atmosphere is in hydrostatic balance. We also have no wind, no convection and no tropopause / stratosphere and higher layers.
    How high this atmosphere reaches is decided by its ‘internal pressure’ overcoming gravity.
    Basically how much energy is contained within this atmosphere and how fast energy flows through it from surface to space.
    Lets say the ELR is 6,5K/km the average we see on earth.
    Now we heat one spot (eg a circle with diameter 500m) a little extra.
    A bubble forms and when warm enough relative to the surrounding air this bubble is squeezed of the ground by the pressure of the surrounding air. Once it has left the ground as a bubble it is on its own.
    The heating is not enough to heat the incoming surrounding air fast enough to maintain a continuous flow of air upwards (= pulsating thermal)
    While the bubble rises it cools according the DALR. Assuming the temperature on departure was 293K at 1000m its internal air temp is now 293K – 10K = 283K
    Static temperature at 1000m is 290K – 6,5K = 283,5K, so the upward travel of this bubble is almost finished.
    Only hope for higher rising bubbles is to reach their dew point before reaching 1000m.
    Start all over, but now the relative humidity is high, and lets say at 1000m the wv in the bubble begins to condense, just in time. Now the bubble cools at initially something like 5K/km. (SALR)
    and it continues to rise, since the atmosphere now is colder around the bubble on its ascend, and the difference increases. At 2000m the bubbles internal temp. is now 283K – 5K = 278K
    Static temp. 290K – 2×6,5K = 277K.
    As long as condensation occurs the bubble will rise and its buoyancy is actually increasing
    (temp difference with the surrounding air increases)
    When all latent heat is spent, the bubble will cool again according the DALR and soon reach the end of its ascend.

  190. Trick says:

    Stephen 9:34pm: Quotes a chemistry site to obfuscate instead of an atmospheric site to clarify enthalpy and shows how far out of of his expertise Stephen really operates: “As we bring two hydrogen atoms together, the potential energy falls to a minimum”.

    Yes, very true. But this is for chemical bonding Stephen. Have you ever wondered why authors talk about H2 and N2 in the atmosphere and not just H and N? This statement you clip is the reason.

    Two H atoms chemically bond together to create H2 molecules because when that happens “..the potential energy falls to a minimum.”. The two Hs combine fall down the energy valley to a less potential energetic position H2. So nature prefers H2 over 2 H. Same for N2. Like nature prefers a marble rolling down to the lowest point of the valley.

    This process has NOTHING to do with my poking a finger into a balloon. Any H in there is already H2, any N is already N2. For goodness gracious sakes alive Stephen, you are soooo… out of your element.

    Here’s another quote from that site: “Because most chemical changes we deal with take place at constant pressure, it would be tedious to have to explicitly deal with the pressure-volume (p*V) work details that were described above. Fortunately, chemists have found a way around this; they have simply defined a new state function that incorporates and thus hides within itself any terms relating to incidental kinds of work (p*V, electrical, etc.)”

    SEE? Chemists operate experiments on earth surface (mostly) so can get away with constant pressure processes (dp=0). Now Stephen, do you imagine meteorology can get away with dp=0 constant pressure processes?? Is my balloon example constant pressure before and after the process of sticking my finger in it??

    Noooooo……Stephen at disney.com once again comes to an incorrect but entertaining conclusion trying to politically escape his previous concession at 8:21pm.

  191. Trick says:

    Stephen 8:57pm: “The push up is from buoyancy within the parcel NOT from an external force. It is the proposition that a force external to the parcel pushes the parcel up which is in dispute”

    Parcel at rest, the upward push against gravity is the integral of the pressure p over the surface of parcel volume V and is balanced by:

    The push in opposite direction down being the weight of the parcel in gravity field: rho*g*V

    This is nowhere in dispute except in the Imagineering of Stephen at disney.com.

  192. Trick says:

    Stephen 8:57pm: “The push up is from buoyancy within the parcel NOT from an external force. It is the proposition that a force external to the parcel pushes the parcel up which is in dispute”

    One more time, parcel at rest, the upward push against gravity is the integral of the pressure p over the surface of parcel volume V and is balanced by:

    The push in opposite direction down being the weight of the parcel in gravity field: rho*g*V

    This is nowhere in dispute except in the Imagineering of Stephen at disney.com.

  193. Ben Wouters says:

    Trick says: November 11, 2014 at 4:40 am

    ” One more time, parcel at rest, the upward push against gravity is the integral of the pressure p over the surface of parcel volume V and is balanced by:

    The push in opposite direction down being the weight of the parcel in gravity field: rho*g*V ”

    Simpler imo: split in downward push on top of the parcel plus weight of the parcel is countered by the upward push on the bottom of the parcel.
    Push on the side of the parcel is equal from all directions otherwise the parcel moves sideways (wind)

  194. wayne says:

    Ben, you are right, we are not on the same page. Some of what you are saying of bubbles as thermals may be true in the UK north of 50°N latitude but not where I am just north of 35°N latitude. You seem to be coming from thought experiments and from books and I am speaking of real life experiences. I have always been a scientist though not professionally since college and most other sailplane pilots I fles with were not. They loved the way I tended to discect the science from flying and to me it was just a big long first experiment with atmospheric science. I also had a sailboat for twenty years so the played into my knowledge of always air and casual meteorology.

    Your 500 m diameter thermal is a little high for your normal thermal I am speaking of. I wish they were so big for it would make finding and centering yourself within such a huge themal so much easier. In the summer, between May and September in the afternoon 12:00 to 16:00 is when thermals really start rocking. I’d say a median would be a radius of 300 feet or 100 meters. We had a plowed field just adjacent to the runway, don’t know why that field was always plowed, but that was always one you could count on for lift.

    Now take 1/4 of a mile square area of dark plowed field on a mostly clear day, some cumulus, and that has an area of about (rounding) 650,000 m². The solar would always be about 900 W/m² on such days and the dark dirt had already reached it’s maximum temperature by noon. That is about 600 mega joules per second and most days are still or very light winds, anywhere around that field the light wind was inward toward the center (story of a dust devil in the center I just had to run 1/8 of a mile out to get in the center later), and most of that power goes up that thermal. The thermal at 100 m radius has an area of about 600 m² so take 650,000,000/600 is lets say about mega joule per meter² of input energy. On days like that most climb rates would be about 1000 to 1400 ft/min and the plane has a constant 200 ft/min downward glide slope so the upwards is actually 1200 to 1600 ft/min. Let’s stay on the low side (and for easy calculation) 1200 ft/min. That is 20 ft/sec or 6 m/s. That will work real good, think in 20 ft(6m) by 1m² vertical slabs that go up every second and that is about right. So that mega joule per second divided by 1.3 kg/m³ for density and divide by 6 m for the slab height and divide by 1003 J/kg/K to get a temperature boost of the flowing inward air of 120 °C which of course is ridiculously high. So let’s see what we didn’t subtract. That 900 W/m² of solar is most of the culprit, some is window radiation right? Some is the appx 300 W/m² you will see in ESRL daily radiation plots lost near noon. That cuts that in half. This is not a 100% efficient system, some hot air is blown off the field and lost even in light winds, cut it in half again. But as you can see there is more than ample joules per second running your ‘engine’ if you are a sailplane pilot, all you have to do is find them, not too hard, and you can fly for four or five hours non-stop usually stuck quite high near the cloud base. See Ben, here they are continuous, those thermals, and not puffs of packets as you and SW imagine somehow.

    Wish I has a camera in hand that day I stood in the center of a roaring huge dust devil centered in the same field spoken of above. Dirt and straw spinning about and I’d say the wind felt like about 40 to 50 knots near the edge. But in the center, much calmer, wow, rubbing the dust out of my eyes you could look straight up into that funnel just like a mini tornado… awsome! and I’ll never forget that moment.

    I’ll pre-ask, please ignore any mistakes… I’m not even going back through this comment to check it.

  195. Ben Wouters says:

    wayne says: November 11, 2014 at 6:46 am

    ” Ben, you are right, we are not on the same page. Some of what you are saying of bubbles as thermals may be true in the UK north of 50°N latitude but not where I am just north of 35°N latitude.”
    I created a world with equal temperature all around, heat ONE spot to create convection and the discussion diverts into how thermals differ between 35N and 50N. It’s obvious I do not think like a scientist 😉

    I also started flying in gliders. Made a career in aviation (~18.000 plus hours heavy jets) and still fly paragliders. So I think I also have some real live experience with weather and meteorology.
    I gave the thought experiment to see if we agree on the use of the DALR and SALR.
    It seems most posters here believe that they somehow have anything to say about the temperature profile of the static atmosphere (ELR).
    Like to hear your ideas on this. (Did you follow the discussion with Stephen about CAPE?)

    About all the solar energy you mention. Mostly oceans and their surface temperatures rise about 0,5K during a full day of sunshine in the tropics. Not much thermals coming from that. Land is a bit different in that it heats up more unevenly than oceans.

  196. Sir George Simpson of the Royal Meteorological Society pointed out to Guy Callendar in 1938:

    “In the second place, one had to remember that the temperature distribution in the atmosphere was determined almost entirely by the movement of the air up and down.”

    That is my position on the matter and the rest of my comments follow on logically from that.

  197. Christopher says:

    “Stephen Wilde says:
    November 10, 2014 at 8:57 pm
    The push up is from buoyancy within the parcel NOT from an external force. It is the proposition that a force external to the parcel pushes the parcel up which is in dispute.”

    Stephen, have you ever been on board an open boat with very thin walled hull?

    As a thought experiment, consider an open hull with very thin wall, zero for all practical purposes, and a material density equal to the fluid inside the hull. The fluid inside the hull is assumed to be air. The fluid outside the hull has a density larger than the air inside the hull. This hull will definitely float, i.e. it has a positive buoyancy.

    The buoyancy effect on this open boat is entirely caused by the difference in pressure between the inside and the outside of the hull. The upward buoyancy is there as long as the pressure integrated over the outside of the hull is larger than the pressure integrated over the inside. What causes this pressure difference is the difference in density of the fluid outside the hull compared to the density of the fluid inside the hull.

    Reducing the density of the outside fluid towards the density of the fluid inside the hull, reduces the buoyancy effect, and increasing the density of the outside fluid increases the buoyancy effect.

    Since the hull is infinitesimally thin and of equal material density as the fluid inside the hull, closing the hull makes no difference to the buoyancy. It is still caused by the difference in pressure integrated over the inside and the outside of the hull.

    Thus we can conclude that buoyancy is an external effect to this hull caused by the difference in density between the inside fluid and the outside fluid.

    However, the closed hull of the thought experiment above is nothing but your parcels of air. Therefore buoyancy is an external effect also to your parcels of air, caused by the difference in density of the air inside and outside the parcel.

  198. Trick said:

    “Stephen 9:34pm: Quotes a chemistry site to obfuscate instead of an atmospheric site to clarify enthalpy and shows how far out of of his expertise Stephen really operates: “As we bring two hydrogen atoms together, the potential energy falls to a minimum”.

    Chemical processes observe the laws of physics. If you increase pressure in a balloon by poking a finger into it then the molecules move closer together and PE declines. The KE produced in the process then dissipates to the surroundings unless you allow the denser parcel of matter to descend to a lower, warmer height.

    If you then remove the finger, PE increases at the expense of the KE, the temperature falls again and the shortfall is recovered from the surroundings provided you do not allow the less dense parcel of matter to rise to a higher, cooler location.

    Ben said:

    “Wrong again. With a surface pressure of eg 1013 hPa the weight of the column above the surface at the equator or at a pole is exactly the same.

    One observes mostly high pressure cells at the poles and mostly low pressure cells above the equator.

    Trick said:

    “One more time, parcel at rest, the upward push against gravity is the integral of the pressure p over the surface of parcel volume V and is balanced by:

    The push in opposite direction down being the weight of the parcel in gravity field: rho*g*V”

    If the parcel goes out of equilibrium with the surroundings and the surroundings remain constant then that is a matter of adding more energy to the interior of the parcel which becomes more buoyant. and not from external pushing from the surroundings.

    ” Some of what you are saying of bubbles as thermals may be true in the UK north of 50°N latitude but not where I am just north of 35°N latitude.”

    Convection is always and everywhere in the form of bubbles simply because a steady flow from the ground cannot long be maintained at the same rate as energy is taken up in uplift. Often it will appear to be maintained as a single stream for a short while but that is really just an elongated bubble.

    Terrible confusion here about virtually all aspects of meteorology plus complete denial of the nature and effect of adiabatic uplift and descent.

  199. Christopher,

    I explained above why floating between two mediums of different densities (water and air) is a special case. In that instance one can say that the combined weight of the air plus the floating object forces a depression on the water surface and the sides of that depression can validly be said to be pushing against the floating object.

    It is quite different when an object is fully immersed in a surrounding medium of different density.

    In that case the extra buoyancy inherent in the lighter object allows it to rise freely of its own volition through a denser medium.

    In order to become buoyant an air parcel must always become less dense than the density of the medium directly above. After that initial starting point it rises adiabatically because pressure and temperature decline at the same rate in both the parcel and the surroundings. In consequence the density differential between the parcel and the surroundings is maintained and uplift continues indefinitely with no work done against the surroundings (other than gravity) and no heat exchanged with the surroundings until a warmer layer is reached.

    That is spontaneous convection and it does not require upward pressure from the denser medium’ Indeed, all the pressure from the denser medium is downward if the lighter item is at the bottom of the ocean or atmosphere.

    All the buoyancy is within the lighter object which then fights against the total downward pressure from the entire vertical column of the overlying denser medium. The overlying medium need only be denser at the point of contact with the rising parcel, not through the entire vertical column.

    Note that the hot air in a thermal only rises when it becomes less dense than the air immediately. above it.

    People here are trying to say that it is pushed up by the more dense air below but that denser air below is being held down by gravity and cannot do any pushing upward. It is all about buoyancy WITHIN the parcel and the density differential between the parcel and the air immediately above it.

    A knowledge of basic meteorology is completely lacking here.

  200. wayne says:

    Ben, alright! A fellow sailplane pilot and those are few and far between. You might have flown a completely different type of lift, like wind against cliffs or mountains. Does the UK even have such strong thermals as I described? Whoa, I am assuming your a Brit and that may not be so. Correct me if I am wrong.

    And sorry if I’m not following and interpreting you correctly. Skimming too much.

    On the DALR and SALR, no, I don’t see them having anything to say about the temperature profile of a static atmosphere or ELR that I have ever detected. I have not even found the DALR on the other atmospheres to have anything to say of their profiles as I wrote near the top of this thread and that lack of correlation lead me to what I seem to have stumbled upon and that does match the data on each.

  201. wayne said:

    “On the DALR and SALR, no, I don’t see them having anything to say about the temperature profile of a static atmosphere or ELR that I have ever detected. I have not even found the DALR on the other atmospheres to have anything to say of their profiles ”

    The gravity induced lapse rate determines the net rate at which temperature can decline with height without the atmosphere being lost.

    Within and around that figure there can be infinite variations in three dimensions related to the vastly varying composition of differing atmospheres.

    It is that variability within and around the gravitationally induced lapse rate that prevents you observing it directly within any given planetary atmosphere.

    It is there, nonetheless and it is its presence in the background that makes your maths work out from the data on each planet.

  202. Christopher says:

    Stephen Wilde says:
    November 11, 2014 at 10:13 am
    =============

    Stephen, there is nothing in your response that alters the very basic definition of buoyancy that is expressed through my little thought experiment.

    Buoyancy has always been, is and will always be caused by the difference in apparent density of the the volume in question versus the density of the fluid outside the volume. Therefore it is an external effect to the volume.

    It does not matter how this difference in density is created, nor how the volume behaves dynamically as the buoyancy acts on the volume.

  203. Christopher says:

    Stephen Wilde says:
    November 11, 2014 at 10:13 am

    “It is all about buoyancy WITHIN the parcel …”

    This is where you basically are wrong.

    Buoyancy is the effect caused by the difference of the pressure integrated over the outside of the boundary of the volume in question is versus the apparent pressure integrated over the inside of the volume boundary.

    Stephen Wilde says:
    November 11, 2014 at 10:13 am

    “Indeed, all the pressure from the denser medium is downward if the lighter item is at the bottom of the ocean or atmosphere.”

    and

    ” … cannot do any pushing upward … ”

    It seems that you also neglects the fact that at any point in a fluid the pressure is equal in ALL directions. That is why a lower pressure on top of a parcel compared to the pressure below the parcel creates an upward force when integrated over the parcel outside area. Uplift occurs when this force is greater than the apparent weight of the parcel, no matter what the content of the parcel is.

  204. Christopher,

    “the buoyancy acts on the volume.”

    The buoyancy is inside the volume as a result of the volume itself being lighter than the surrounding medium. It is a physical characteristic of the volume and not a force acting on the volume.

    Once it exists it can then enable the volume to spontaneously do work on the surroundings.

    The buoyancy is not an external force acting on the volume.

  205. Christopher says:

    Stephen Wilde says:
    November 11, 2014 at 11:03 am

    “Christopher,

    “the buoyancy acts on the volume.”

    The buoyancy is inside the volume as a result of the volume itself being lighter than the surrounding medium. It is a physical characteristic of the volume and not a force acting on the volume.

    Once it exists it can then enable the volume to spontaneously do work on the surroundings.

    The buoyancy is not an external force acting on the volume.”

    =====

    Buoyancy is NOT a property or a physical characteristic of a volume. It is an effect of the external pressure acting on the volume. This is the core of buoyancy as phenomenon, being it for ships, submarines, gas balloons, or just a limited “parcel of air”.

    I am sorry to say, but I cannot help you more to understanding than I have done so far.

    Your understanding of basic fluid physics as expressed in several of your comments in this thread, is nothing but wrong. What you are trying to establish is a new physics for fluids. This will not succeed.

  206. Christopher said:

    “That is why a lower pressure on top of a parcel compared to the pressure below the parcel creates an upward force ”

    That upward force is balanced by the downward pull of gravity for a zero net effect. Let me explain.

    The less dense parcel starts to rise when it becomes less dense than the air immediately above it as a result of surface heating. At that point there is no air beneath it to provide more pressure than the air above it.

    So the buoyancy is initiated (energy injected into the parcel) whilst there is no upward force.

    Once the buoyancy is present and the parcel has left the ground then and only then is there greater air pressure below than above and uplift continues just as you say.

    But that is then adiabatic uplift which does no work on the surroundings and does not exchange heat with the surroundings because the parcel is constantly moving into regions that have the same temperature and pressure as itself.

    Gravity ensures that the temperature and pressure of both parcel and surroundings decline at exactly the same rate.

    A parcel with the same pressure as its surroundings does no work on the surroundings.

    A parcel with the same temperature as its surroundings exchanges no heat with its surroundings.

    If no work is being done and no heat is being exchanged then there is no upward pushing which is why it is called spontaneous convection.

    What is happening is that the constant pull of the gravitational field is balancing the upward pushing that would otherwise have occurred due to the pressure differential between the top and the bottom of the parcel.

    Neither the initial injection of energy from the surface nor the continuing adiabatic uplift requires external pushing from below.

    All this debate is because the oddities of adiabatic ascent and descent are not generally known except in meteorology.

  207. “Buoyancy is NOT a property or a physical characteristic of a volume. It is an effect of the external pressure acting on the volume”.

    Buoyancy is a consequence of one type or mixture of mass reacting differently to the gravitational field as compared to the mass in the surroundings.

    It is affected by kinetic energy content.

    So, it is a product of the physical characteristics of the mass involved plus its kinetic energy content.

    Lighter and hotter material rises higher against gravity (more buoyant).

    Heavier and colder material sinks lower with gravity (less buoyant).

    In an adiabatic process all work involving uplift or descent is done between the mass and gravity, not between mass and surroundings.

    Hence:

    “Sir George Simpson of the Royal Meteorological Society pointed out to Guy Callendar in 1938:

    “In the second place, one had to remember that the temperature distribution in the atmosphere was determined almost entirely by the movement of the air up and down.””

  208. Let me expand on this point:

    “What is happening is that the constant pull of the gravitational field is balancing the upward pushing that would otherwise have occurred due to the pressure differential between the top and the bottom of the parcel.”

    We have established that for uplift to occur the lower parcel needs to be less dense than the air immediately above it.

    That denser air immediately above the less dense air obviously is being pulled by gravity more than the less dense air below it.

    That additional downward pull by gravity on the higher denser air above the lighter parcel offsets the pushing effect of the denser air below the lighter parcel.

    That is how the interaction between the parcel and its surroundings comes to be negated leaving only the parcel’s interaction with gravity to be considered and that interaction with gravity is dealt with in the adiabatic process.

  209. Christopher says:

    Stephen Wilde says:
    November 11, 2014 at 11:25 am

    and

    Stephen Wilde says:
    November 11, 2014 at 11:36 am

    and

    Stephen Wilde says:
    November 11, 2014 at 12:15 pm

    ====================

    Stephen, your comments above regarding buoyancy cannot be characterized otherwise than partly being nonsense. It is not in line with how buoyancy is DEFINED.

    Before you continue arguing further about anything concerning buoyancy, please read carefully and ensure that you have understood the implications of the following short lecture defining buoyancy:

    http://hyperphysics.phy-astr.gsu.edu/hbase/pbuoy.html

    This is the definition applied in science and engineering in fields where buoyancy is a significant effect. It must definitely also apply within the field of atmospheric physics.

  210. Ben Wouters says:

    Stephen Wilde says: November 11, 2014 at 9:34 am

    ” Ben said:
    “Wrong again. With a surface pressure of eg 1013 hPa the weight of the column above the surface at the equator or at a pole is exactly the same.

    One observes mostly high pressure cells at the poles and mostly low pressure cells above the equator”
    Brilliant observation. And your point is?

  211. Christopher says: November 11, 2014 at 11:19 am

    “Buoyancy is NOT a property or a physical characteristic of a volume. It is an effect of the external pressure acting on the volume. This is the core of buoyancy as phenomenon, being it for ships, submarines, gas balloons, or just a limited “parcel of air”.

    I agree, mathematically I believe this is called “curl” of a gradient field in a force field like gravitational attraction, or an EMR “field strength”. It is the “spontaneous action” required by the field curl, upon any volume greater than zero.
    ————————————————————————-
    Stephen Wilde says: November 11, 2014 at 11:36 am

    “In an adiabatic process all work involving uplift or descent is done between the mass and gravity, not between mass and surroundings.”

    Please demonstrate ” any” adiabatic process? This is a fantasy!

    “Hence: “Sir George Simpson of the Royal Meteorological Society pointed out to Guy Callendar in 1938: “In the second place, one had to remember that the temperature distribution in the atmosphere was determined almost entirely by the movement of the air up and down.”/:” /:.

    Please demonstrate why Simpson is correct? Demonstrate why atmospheric mass motion is a requirement, rather than a consequence, of the required energy transport, in the direction of the curl? Demonstrate that that the whole mass must be uplifted rather than only the low density, high heat energy content, WV molecules, at a much greater velocity? The lapse rate re-thermalization of the atmospheric column appears way to rapid for whole mass motion!
    ———————————————————————————————-
    Stephen Wilde says:November 11, 2014 at 12:15 pm

    That is how the interaction between the parcel and its surroundings comes to be negated leaving only the parcel’s interaction with gravity to be considered and that interaction with gravity is dealt with in the adiabatic process.

    Stephen, you have way too much bliovating, which demonstrates a lack of understanding.
    Demonstrate any atmospheric adiabatic process! All here “have” demonstrated that whatever process you claim, is never adiabatic!

  212. Ben Wouters says:

    wayne says: November 11, 2014 at 10:17 am

    ” Ben, alright! A fellow sailplane pilot and those are few and far between. You might have flown a completely different type of lift, like wind against cliffs or mountains. Does the UK even have such strong thermals as I described? Whoa, I am assuming your a Brit and that may not be so. Correct me if I am wrong.”
    Gliders solely in Holland (I’m Dutch). Strongest I’ve seen while gliding is ~ 4 m/s.
    Paragliding initially winch launched in Holland, later in the Alps and Dolomites.
    Mixed soaring and using the valley wind. Max altitude under a paraglider ~ 1900M above the Austrian Alps. Wonderful.

    Would you comment on the thought experiment I gave earlier.
    I want to get the whole DALR / SALR nonsense out of the way and finally have a serious discussion about the temperature profile of an atmosphere in hydrostatic balance.

    About the 120C temp. boost you initially came up with:
    solar heats the upper part of the surface (50 cm – 1 meter ?)
    This solar heated surface then heats the overlaying air via conduction / radiation.
    This is a pretty slow process, so the temp. increase is much lower than the available solar power indicates.

  213. Ben Wouters says:

    Stephen Wilde says: November 11, 2014 at 11:25 am

    ” Once the buoyancy is present and the parcel has left the ground then and only then is there greater air pressure below than above and uplift continues just as you say.

    But that is then adiabatic uplift which does no work on the surroundings and does not exchange heat with the surroundings because the parcel is constantly moving into regions that have the same temperature and pressure as itself”
    A rising parcel moves into a reducing pressure. It adjusts its internal pressure by expanding, to maintain the same pressure as the surrounding air, and thus cools.
    The temperature of the parcel reduces according the DALR (or SALR). What the temperature of the surrounding static atmosphere is, changes from place to place and moment to moment.
    If the temperature of the parcel becomes equal to that of the surrounding air, convection stops.

  214. Trick says:

    Stephen 9:34am: “If you increase pressure in a balloon by poking a finger into it then the molecules move closer together and PE declines.”

    Nonsense according to Stephen correctly writing 8:21pm: “Your finger has altered the p*V term but not the gravitational PE term.” Stephen lives up to all my expectations, backslides.

  215. The PE between molecules declines but not the gravitational PE.

    Please keep up.

  216. Trick says:

    Stephen 2:19pm: Nonsense. You’ve been told there is no PE between molecules. There is a force. Please keep up. Remember this meteorological science and apply, do not live up to all my expectations by putting up another political based defense:

    Parcel TE=KE+PE no detection of alteration in p*V so incorrect.
    Parcel TE=KE+PE+p*V detects the finger push, so parcel enthalpy is correct for gas.

    Buoyancy. A fluid (at rest) with density rho in a volume V is acted on by 2 equal and opposite forces: its weight (rho*g*V) and the integral of the pressure p over the surface of V. If the fluid in V is replaced by a fluid with a different density rho’ in such a way that the pressure distribution is unchanged, a net force acts on the volume, the buoyancy force.

    If rho’ less than rho, the buoyant force is upwards, positively buoyant. If rho’ greater than rho, the buoyant force is downwards, negatively buoyant.

  217. Christopher, from your link:

    “The behaviour of the three balls would certainly be different upon release from rest in the water. The cork would bob up, the aluminium would sink, and the lead would sink more rapidly. But the buoyant force on each is the same because of identical pressure environments and equal water displacement. The difference in behaviour comes from the comparison of that buoyant force with the weight of the object.”

    The difference in behaviour is related to the different densities of the three balls and not the strength of any pre-existing buoyant force. There is no buoyant force unless density falls locally first.

    So it is in an atmosphere.

    The more one reduces the density of the lighter air the greater is the upward force.

    If the density of the balls in the water or of the air parcel in the atmosphere is the same as the surroundings then there is no buoyant force.

    Therefore it is the lower density of the balls or the air parcels that creates the buoyancy.

    For an air parcel density is reduced by adding more kinetic energy to the parcel than to the surroundings. That added kinetic energy then creates buoyancy WITHIN the parcel.

    The parcel then rises spontaneously without needing to be pushed by the denser air around it.

    The truth is that the reduction in density alters the relationship between the parcel and GRAVITY.

    The fact that it also alters the relationship between the parcel and the surroundings is neither here nor there.

    In the absence of the gravitational field there would be no buoyant forces anywhere regardless of density variations.

    That is why uplift in an atmosphere is adiabatic. The changing relationship of mass with the gravitational field is thermally far more important than any changing relationship with the surroundings.

    That is what Sir George Simpson was pointing out.

    Will’s denial of the existence of adiabatic processes is not acceptable.

    Ben said:

    “A rising parcel moves into a reducing pressure. It adjusts its internal pressure by expanding, to maintain the same pressure as the surrounding air, and thus cools.”

    It also moves into reducing temperature, not just pressure. Therefore the temperature inside the parcel also remains the same as that outside despite the change in height and no exchange of heat can occur when there is no temperature differential.

    You can say that the thermal structure of the atmosphere is infinitely variable so that this doesn’t apply at all times everywhere and that is true but on average, overall the principle holds true.

  218. Trick,

    Nonsense:

    “Figure 1 shows how the potential energy of two molecules and the force between them changes with their separation. ”

    http://www.schoolphysics.co.uk/age16-19/Thermal%20physics/Heat%20energy/text/Forces_between_molecules/index.html

  219. wayne says:
    October 31, 2014 at 9:40 am

    “Stephen, please one point at a time.

    You say that Will Janoschka said: “Will says that the mass of an atmosphere has zero effect on the free flow of electromagneic energy through it and thus zero effect on temperature.”

    First I can not imagine Will Janoschka saying such a thing so probably you must have misunderstood him in some manner or other but you should know better yourself.

    All mass impedes EMR. If you would the time to read at least some of Max Planck’s “Theory of Heat Radiation” (http://www.gutenberg.org/ebooks/40030) you would find Planck himself even says much on this subject and near page 10: “We only take account of the fact that every ray on its path through any medium loses a certain fraction of its intensity.” He is speak of mass and mass absolutely does impede EMR.”

    Wayne,
    Planck is correct, but Kirchhoff modified that concept slightly for mass at radiative equilibrium! The case where at “every” frequency, that mass is radiatively emitting to a lower temperature the same flux as it is absorbing from a higher temperature, at equilibrium there is no change in flux, no change in temperature or energy of that mass, The mass is transparent to the flux. However: Because of any emissivity of that mass at any wavelength it is not transparent to the field strength or the normalized FS called “radiance”.
    In the case of a highly emissive non powered intermediate mass, i.e. the Willis E. infamous steel greenhouse, introduces two more surfaces the inner and the outer at the same intermediate temperature between that of the source and the sink of that EMR The result is always an “apparent” decrease in the emissivity/absorptivity of the source or sink.

    With the Willis E’s shell, an isotropic absorber/emitter. The reduction is to 0.5 of the original, requiring an increase in surface temperature of the source for the same flux. At that equilibrium point, all flux from the source transfers transparently to the sink, through the shell with no attenuation.. No “back radiation”, just like in this atmosphere does, with much different geometry.
    These political activist Climate Clowns have “some” scientific knowledge, but way way less than they think they have. They are the true dangerous terrorists! FEAR, FEAR, FEAR, or ponder with three fingers of Glenfiddich, while carefully sharpening the pitchfork, something useful for hands to do, while pondering! YMMV! -will-

  220. Stephen Wilde says: November 11, 2014 at 2:55 pm

    “Will’s denial of the existence of adiabatic processes is not acceptable.”

    It certainly is to the jury, please demonstrate any evidence of your fantasy?

  221. One of the problems here is that people are accustomed to regarding interactions between objects or materials as always a result of forces between those objects or materials. Hence the use of the so called ‘buoyancy force’

    That is fine for day to day calculations used for practical purposes but it isn’t quite accurate.

    When it comes to the interaction between objects of different densities one has to introduce the differing interactions with the gravitational field.

    Thus buoyancy ceases to be a result of forces between objects but instead becomes a consequence of the differing relationship with gravity of objects with different densities.

    Warmer lighter objects rise against gravity.

    Colder heavier objects fall with gravity.

    It is the variable interactions with gravity that create or destroy buoyancy and not some separate buoyancy force passing between denser or less dense parcels of matter.

  222. “please demonstrate any evidence of your fantasy?”

    “Adiabatic heating occurs in the Earth’s atmosphere when an air mass descends, for example, in a katabatic wind or Foehn or chinook wind flowing downhill over a mountain range. When a parcel of air descends, the pressure on the parcel increases. Due to this increase in pressure, the parcel’s volume decreases and its temperature increases as work is done on the parcel of air, thus increasing the internal energy. The parcel of air is unable to dissipate energy as heat, hence it is considered adiabatically isolated, and its temperature will rise sensibly”

    http://en.wikipedia.org/wiki/Adiabatic_process#Adiabatic_heating_and_cooling

  223. Christopher says:

    Stephen Wilde says:
    November 11, 2014 at 2:55 pm

    “Christopher, from your link:

    “The behaviour of the three balls would certainly be different upon release from rest in the water. The cork would bob up, the aluminium would sink, and the lead would sink more rapidly. But the buoyant force on each is the same because of identical pressure environments and equal water displacement. The difference in behaviour comes from the comparison of that buoyant force with the weight of the object.”

    The difference in behaviour is related to the different densities of the three balls and not the strength of any pre-existing buoyant force. There is no buoyant force unless density falls locally first.”

    Stephen, you are missing the definition of buoyancy when you state the last cited sentence:

    “There is no buoyant force unless density falls locally first”

    according to the definition of buoyancy, there IS a buoyant force no matter what the density is inside as well as outside the air parcel.

    Buoyancy of the air parcel is caused by the pressure acting on the outside of the volume of the air parcel.

    Gravity acts on the mass of the volume inside the air parcel.

    When the buoyancy force acting on the parcel is larger than the gravity force acting on the parcel, the parcel will rise.

    When the buoyancy force acting on the parcel is smaller than the gravity force acting on the parcel, the parcel will sink.

    When the buoyancy force acting on the parcel equals the gravity force acting on the parcel, the parcel will stay in place.

    It is as simple as that.

  224. Trick says:

    Stephen 2:58pm: Wriggles around on google. Did you even read that piece? Certainly you did not comprehend it. A molecule can be displaced in a solid and it will spring back into place when the force is removed using PE+KE=constant. The curve shows solids are elastic. Pull a rubber band apart, extend it, learn about solid KE+PE snap back. Does a gas do that Stephen? Hmmm…maybe Stephen needs to find a piece explaining gas enthalpy in meteorology instead of employing political defenses & obfuscation from fields of chemistry, structure mechanics.

    “In this section we will look at just two adjacent molecules in a solid

    “..shown by M on the graph. This is the equilibrium position for molecules in the solid“.

    M shows 0 PE at equilibrium between molecules even for a solid. For a solid, p*V is negligible. Does the piece even mention p*V? No. Because p*V term is negligible in a solid, this is why JCM added p*V to gas physics in 1868. Some intuition formed from solids was found to fail in a gas as proven by Stephen’s Imagineering.

    Try again. Use science Stephen, use meteorology field, no obfuscation, no politics.

    Parcel TE=KE+PE no detection of alteration in p*V so incorrect.
    Parcel TE=KE+PE+p*V detects the finger push, so parcel enthalpy is correct for gas.

  225. Ben Wouters says:

    Stephen Wilde says: November 11, 2014 at 2:55 pm

    “Ben said:
    “A rising parcel moves into a reducing pressure. It adjusts its internal pressure by expanding, to maintain the same pressure as the surrounding air, and thus cools.”

    It also moves into reducing temperature, not just pressure. Therefore the temperature inside the parcel also remains the same as that outside despite the change in height and no exchange of heat can occur when there is no temperature differential.”

    It moves into the existing temperature profile for the place where the parcel rises.
    This can be anything. Stable profile, very unstable profile. temperature inversions etc.etc.
    In a very unstable atmosphere (high CAPE) the temp. difference between the parcel and the surrounding air increases dramatically, and the parcel accelerates forming large CB’s

    There is no AVERAGE convection. Convection is always a local phenomenon.

  226. Stephen Wilde says: November 11, 2014 at 3:21 pm

    (“please demonstrate any evidence of your fantasy?”)

    “Adiabatic heating occurs in the Earth’s atmosphere when an air mass descends, for example, in a katabatic wind or Foehn or chinook wind flowing downhill over a mountain range. When a parcel of air descends, the pressure on the parcel increases. Due to this increase in pressure, the parcel’s volume decreases and its temperature increases as work is done on the parcel of air, thus increasing the internal energy. The parcel of air is unable to dissipate energy as heat, hence it is considered adiabatically isolated, and its temperature will rise sensibly”
    http://en.wikipedia.org/wiki/Adiabatic_process#Adiabatic_heating_and_cooling

    None of what you claim is adiabatic. The gravitational parts, may be adiabatic, all else is very diabatic. You spout nonsense!

  227. Christopher says: November 11, 2014 at 3:28 pm
    Stephen Wilde says:November 11, 2014 at 2:55 pm

    “Christopher, from your link:
    “The behavior of the three balls would certainly be different upon release from rest in the water. The cork would bob up, the aluminum would sink, and the lead would sink more rapidly. But the buoyant force on each is the same because of identical pressure environments and equal water displacement. The difference in behavior comes from the comparison of that buoyant force with the weight of the object.”

    Interesting, What is the geometry of your three balls within the environment? My three concentric emissive spheres/shells in a vacuum demonstrate the most simple geometry for the so called only radiative energy transfer.

  228. For Trick,

    more on PE and KE between gas molecules:

    http://physics.stackexchange.com/questions/78085/potential-vs-kinetic-energy-of-particles-in-gas

    Christopher said:

    “When the buoyancy force acting on the parcel is larger than the gravity force acting on the parcel, the parcel will rise.

    When the buoyancy force acting on the parcel is smaller than the gravity force acting on the parcel, the parcel will sink.

    When the buoyancy force acting on the parcel equals the gravity force acting on the parcel, the parcel will stay in place.”

    I don’t have any problem with that except that you think that the buoyancy force is some sort of discrete force acting on the parcel from outside.

    In fact, the buoyancy force is just the netted out effect of gravity on two substances of different densities.

    You could reword it thus:

    When the effect of the gravity force on the gaseous surroundings is larger than the gravity force on the parcel the parcel will rise,

    When the gravity force on the gaseous surroundings is less than the gravity force on the parcel the parcel will sink

    When the gravity force on the parcel is equal to the gravity force on the gaseous surroundings the parcel will stay in place.

    So the term ‘buoyancy force’ is just a convenient construct to describe the net balance of the two gravity forces in a given situation where the densities differ.

    It is not a discrete force in its own right.

  229. Roger,

    829, reply posts to your well stated, but controversial article. How many more days to 1000? How many more days to an award?

  230. Trick says:

    Stephen 4:24pm – That blog site is again talking about solids. Meteorology deals with gas. You must not be able to find a relevant site supporting your political views.

  231. It is about gases. Solids are only mentioned in passing.

  232. Christopher.

    A correction for my last post.

    When I referred to the gravity force in each case it should really be ‘the response to the gravity force’ though I suppose the use of the word ‘effect of’ might be sufficient because the effect of the gravity force is density related

  233. Will Janoschka says: November 11, 2014 at 4:35 pm

    Roger,

    “829, reply posts to your well stated, but controversial article. How many more days to 1000? How many more days to an award?”

    Now, 834, Clever politician!

  234. Trick says:

    Stephen – It also shows PE going to 0 intermolecule for the gas. Read it. But it is just a blog discussion, if that’s all you can find to support your political views then shows the trouble you are having with the science.

  235. Roger,
    Please consider the most desperate earthling question ever, “et tu brute?” It is fine to try, ..the most luck. Please hone up on your rapid running skills. now 837 how clever! sdddddddddddddddddddddd “Kitty cat”, I dislike that comment.

  236. wayne says:

    Stephen… very good, I just noticed your comment here and we might be making headway:

    Stephen Wilde says:
    November 11, 2014 at 3:21 pm

    “please demonstrate any evidence of your fantasy?”

    “Adiabatic heating occurs in the Earth’s atmosphere when an air mass descends, for example, in a katabatic wind or Foehn or chinook wind flowing downhill over a mountain range. When a parcel of air descends, the pressure on the parcel increases. Due to this increase in pressure, the parcel’s volume decreases and its temperature increases as work is done on the parcel of air, thus increasing the internal energy. The parcel of air is unable to dissipate energy as heat, hence it is considered adiabatically isolated, and its temperature will rise sensibly”

    http://en.wikipedia.org/wiki/Adiabatic_process#Adiabatic_heating_and_cooling

    I think you are still hiding behind a badly worded Wikipedia paragraph, that is unless you also think both of these are also correct:

    “Adiabatic heating occurs in the Earth’s atmosphere when an air mass descends [due to gravity or downward winds]. When a parcel of air descends, the pressure on the parcel increases. Due to this increase in pressure and density, the parcel’s volume decreases and its temperature increases as work is done on the parcel of air [by the surrounding air at a higher pressure and density], thus increasing the internal energy. The parcel of air is unable to dissipate energy as heat, hence it is considered adiabatically isolated, and its temperature will rise sensibly”

    and since adiabatic is reversible:

    “Adiabatic cooling occurs in the Earth’s atmosphere when an air mass ascends [due to buoyancy or upward winds]. When a parcel of air ascends, the pressure on the parcel decreases. Due to this decrease in pressure and density, the parcel’s volume increases and its temperature decreases as work is done by the parcel of air [on the surrounding air at a lower pressure and density], thus decreasing the internal energy. The parcel of air is unable to gain energy as heat, hence it is considered adiabatically isolated, and its temperature will be lowered sensibly”

    This is what thermodynamics courses teach you unless I myself made some mistake rewording those opposing cases.

    Do you also agree with both of those paragraphs Stephen? I don’t think you do.

  237. wayne,

    I don’t agree with your changes to the wording because it is well established that in adiabatic uplift and descent the work is done with gravity and not the surrounding air.

    I’ve pointed that out with links in support so many times now that further debate on the issue is pointless.

  238. One more chance for you guys to remove your blinkers:

    http://www.recreationalflying.com/tutorials/meteorology/section1a.html#pressure_gradient

    and in particular:

    “As the pressure decreases with height so, in any parcel of air, the downwards pressure over the top of the parcel must be less than the upwards pressure under the bottom. Thus within the parcel there is a vertical component of the pressure gradient force acting upward. Generally this force is balanced by the gravitational force, so the net sum of forces is zero and the parcel floats in equilibrium. This balance of forces is called the hydrostatic balance. When the two forces do not quite balance, the difference is the buoyancy force. This is the upward or downward force exerted on a parcel of air arising from the density difference between the parcel and the surrounding air.”

    which is as I have been explaining it even before finding that link.

    The so called buoyancy force arises solely from the density differences and NOT the fact that pressure at the bottom of the parcel is higher than pressure at the top of the parcel because that aspect is balanced by the gravitational force within the hydrostatic balance.

    The work done as a result of the density difference is all done against gravity and NOT the surrounding air.

    The less dense parcel rises spontaneously with no pushing from below

    Apologies are in order but I won’t hold my breath

  239. More here:

    http://www.recreationalflying.com/tutorials/meteorology/section7.html

    “The kinetic energy is provided by mass; i.e. an air parcel, vertically displaced by a disturbance, will be acted on by gravity because its density differs from its environment. The potential energy of displacement is converted to kinetic energy when buoyancy returns the parcel to its original level”

    An excellent site where accuracy and clarity are paramount since lives are at risk in aviation.

  240. Kristian says:

    Stephen,

    You still haven’t answered my questions to you:

    How is this equation to be understood according to you?

    ΔU = Q – W

    What’s going on here? What process is it describing? What is U? What is Q? And what is W? And how does this equation relate to ‘the adiabatic process’?

    We are also all waiting with bated breaths to hear your explanation of how a parcel of air simply rises spontaneously (“and then a miracle occurs”) from just being ‘lighter’ (less dense), without any external forces pushing or pulling at it at all.

  241. wayne says:

    Yes Stephen, you seem to have it, the net force on the object or parcel is the upward buoyancy force minus the weight, both are in newtons, dynes or pound-force (lbf). If that difference is positive, it rises. If the difference is negative it falls, slowly if the negative difference is small, faster if the difference is large like when a rock is within air, not so much if the rock is within water, a rock will float if within mercury. When the difference is zero it will float motionless because there is zero force difference… you are right, that IS hydrostatic equilibrium. Just keep in mind there is always a subtraction involved of two different forces — buoyancy and weight.

    I had the feeling that eventually you would get it straight or I wouldn’t keep correcting you and only when I see it is needed. I’m also sure the same for the others trying to get you to look at it different.

  242. Face saving nonsense from, a pair of meteorological illiterates.

  243. “the net force on the object or parcel is the upward buoyancy force minus the weight, both are in newtons, dynes or pound-force (lbf). ”

    That seems to illustrate the confusion.

    It is the net force that is the buoyancy force. Mass is either buoyant (rising) or not buoyant (sinking).

    The net force is a result of balancing the internal kinetic energy which works against gravity with the weight which works with gravity.

    One can try and say that the internal kinetic energy provides a buoyant force but if it doesn’t offset the weight then the parcel sinks and so it isn’t then providing buoyancy.

    The problem with my critics here is that they have been trying to say that the buoyancy force is a separate discrete force acting on the parcel from outside the parcel.

    It is not. There is no pushing up of the parcel by a force external to it.

    The correct position for a parcel of air within a gravitational field is derived from the kinetic energy INSIDE the parcel as compared to the weight of the parcel and it is the relationship between the two which leads to buoyancy or not.

    Maybe a subtle distinction but a critical one in terms of accuracy of description.

    That is why you have to add kinetic energy to the inside of a balloon to make it rise. A point I made previously which was criticised by some.

  244. Having got that straight I recommend that my critics revisit the issues of adiabatic uplift and descent plus the conversion of KE to PE and back again.

    Maybe some of you will learn something useful.

  245. Let’s look at another example of wayne’s confusion where he said this:

    “The pressure below (the air parcel) is also maintained or even increased by the velocities of the inflow radially converging upon a single center point, pushing inward from all directions and that raises pressure also.”

    The pressure difference between the bottom of a parcel and the top is a result of the general decline of pressure with height.

    It says nothing about pressure differentials at the surface in the horizontal plane.

    No air will flow in beneath a rising parcel unless, in rising, the parcel leaves lower surface pressure beneath it.

    No matter how low (or high) the surface pressure becomes, there will be no effect on the pressure difference between the top and the bottom of the parcel because that is determined only by the general decline in pressure with height.

    There is really no substitute for a proper understanding of the highly specialised field of meteorology.

    I could supply similarly detailed critiques of all the critical comments made in response to my contributions here but life is too short.

  246. Trick says:

    Stephen 4:10am: “..accuracy of description. That is why you have to add kinetic energy to the inside of a balloon to make it rise.”

    The pilot of a hot air balloon at rest drops a bag of sand off the side of the basket. The balloon rises. A separate discrete force acting on the outside of the hot air balloon i.e. external to the balloon caused it to be pushed up. Like your link says: “When the two forces do not quite balance, the difference is the buoyancy force.”

    4:21am: Your critics are trying to help you come to correct science conclusions Stephen instead of your usual political conclusions.

    “..revisit the issues of adiabatic uplift..”

    Try this – see sec. 1.11 : “An adiabatic process is…An adiabatic temperature change occurs due to the change in pressure and volume…”

    http://www.recreationalflying.com/tutorials/meteorology/section1b.html

    An excellent site where accuracy and clarity are paramount since lives are at risk in aviation. There is more to parcel science than just KE+PE and back again Stephen:

    Parcel TE=KE+PE no detection of alteration in p*V so incorrect.
    Parcel TE=KE+PE+p*V detects the finger push, so parcel enthalpy is correct for gas.

  247. Christopher says:

    Stephen Wilde says:
    November 11, 2014 at 4:24 pm

    “I don’t have any problem with that except that you think that the buoyancy force is some sort of discrete force acting on the parcel from outside.

    In fact, the buoyancy force is just the netted out effect of gravity on two substances of different densities.”

    No, Stephen, I am not “thinking that the buoyancy force is some sort of discrete force acting on the parcel from outside.”

    I am telling you in words what the actual physical-mathematical definition of buoyancy is.

    Mathematically it is the integral over the external surface of the volume in question of the pressure acting normally to the surface. In general this has NOTHING to do with gravity. Gravity comes to play when the pressure over the surface is caused by or related to gravity.

    For a mathematical treatment see e.g. H.P. Greenspan and D.J. Benny: “Calculus – An introduction to applied mathematics”

    http://books.google.no/books?id=E-__4PCIE-EC&pg=PA695&lpg=PA695&dq=mathematics+curl+and+buoyancy&source=bl&ots=N7cp1hFFQw&sig=UhpkvSHEjih4urcOogYU0AK6le4&hl=en&sa=X&ei=0PxiVNfmHYnBOsWGgOgD&ved=0CCMQ6AEwAQ#v=onepage&q=mathematics%20curl%20and%20buoyancy&f=false

    page 695, eq. (14)

    I think you should keep to this definition of buoyancy. That will make communication with scientists and engineers a lot easier for you.

  248. wayne says:

    There you go Stephen, page 695. Read the next to the last paragraph — that is exactly all that I have been trying to explain to you… unsuccessful so far and far too many words but there you have “buoyance force” in a consise format.

    Here’s an even more compact form if you can get the volume simply:
    \displaystyle {\bold{F}_{buoyancy:always\,upward}}={\rho_{surrounding\,fluid}\times{g_{accel}}\times{V_{immersed\,object}}}
    The ‘g’ acceleration here is positive and not a vector unlike the surface integral in the book which has ‘g’ defined properly as a downward vector therefore the also necessary leading negative sign. Hope you can follow that Stephen.

    Listen to Christopher, he’s shooting straight with you.

  249. Ben Wouters says:

    Stephen Wilde says: November 12, 2014 at 4:10 am

    ” The net force is a result of balancing the internal kinetic energy which works against gravity with the weight which works with gravity.”

    Lets try a basketball. Push it underwater eg 1 meter, and let it go.
    It races to the surface, jumps in the air and then falls back to the water.
    How come?
    Its internal kinetic energy didn’t change, gravity didn’t change and its weight didn’t change. Yet the ball falls to the surface. A miracle?

  250. Kristian says:

    Stephen Wilde says, November 12, 2014 at 3:38 am:

    “Face saving nonsense from, a pair of meteorological illiterates.”

    That’s not an answer, Stephen.

    Yet it speaks volumes.

  251. Trick said:

    “A separate discrete force acting on the outside of the hot air balloon i.e. external to the balloon caused it to be pushed up”

    A separate discrete force in the form of a sandbag was added to the side of the balloon to prevent spontaneous uplift and then discarded.

    Christopher said:

    “In general this has NOTHING to do with gravity. Gravity comes to play when the pressure over the surface is caused by or related to gravity.”

    No gravity, no pressure. A collection of parcels of different densities would place no pressure on each other in the absence of gravity.

    Ben said:

    “Lets try a basketball. Push it underwater eg 1 meter, and let it go.
    It races to the surface, jumps in the air and then falls back to the water.
    How come?”

    It is lighter than the water but heavier than the air. No magic. The movement up through the water and down from the air is a consequence of its own internal interaction with gravity.

  252. Ben said:

    “Its internal kinetic energy didn’t change,”

    All movement in a gravitational field involves a change in the proportion of internal energy held as PE and KE.

    With the basketball, being a solid, the change in internal energy is trivial compared to the intermolecular forces and so it doesn’t visibly expand or contract when moved up or down.

    The behaviour of gases is completely different.

  253. Christopher says:

    Stephen Wilde says:
    November 12, 2014 at 10:54 am

    “No gravity, no pressure. A collection of parcels of different densities would place no pressure on each other in the absence of gravity.”

    Stephen, please listen.

    Buoyancy is DEFINED as I have told you in several ways.

    Buoyancy IS NOT restricted to your narrow application with “air parcels”. It is a phenomenon that occur in many other fields of physics, also applications where the gravity field is not present or can be neglected compared to the magnitude of the pressure gradient. You may very well continue to use your private definition of “buoyancy”, but I can guarantee that it will lead to (unnecessary) confusion.

    Why can you not accept the definition common in all other fields of physics and engineering?

  254. Christopher,

    The subject here is air parcels.

    This description (not mine) is correct for air parcels:

    “As the pressure decreases with height so, in any parcel of air, the downwards pressure over the top of the parcel must be less than the upwards pressure under the bottom. Thus within the parcel there is a vertical component of the pressure gradient force acting upward. Generally this force is balanced by the gravitational force, so the net sum of forces is zero and the parcel floats in equilibrium. This balance of forces is called the hydrostatic balance. When the two forces do not quite balance, the difference is the buoyancy force. This is the upward or downward force exerted on a parcel of air arising from the density difference between the parcel and the surrounding air.”

    If you go off on one about vastly different scenarios then it is you who is causing confusion.

    The density differential causes more or less buoyancy WITHIN the parcel which then rises or falls spontaneously in an adiabatic process as per the Gas Laws.

  255. Christopher says:

    Stephen Wilde says:
    November 12, 2014 at 11:56 am

    “Christopher,

    The subject here is air parcels.

    This description (not mine) is correct for air parcels:

    “As the pressure decreases with height so, in any parcel of air, the downwards pressure over the top of the parcel must be less than the upwards pressure under the bottom. Thus within the parcel there is a vertical component of the pressure gradient force acting upward. Generally this force is balanced by the gravitational force, so the net sum of forces is zero and the parcel floats in equilibrium. This balance of forces is called the hydrostatic balance. When the two forces do not quite balance, the difference is the buoyancy force. This is the upward or downward force exerted on a parcel of air arising from the density difference between the parcel and the surrounding air.”

    If you go off on one about vastly different scenarios then it is you who is causing confusion.

    The density differential causes more or less buoyancy WITHIN the parcel which then rises or falls spontaneously in an adiabatic process as per the Gas Laws.”

    Stephen, What is the reference to this description?

    The definition on buoyancy applied in the above citation is NOT in agreement with the definition used generally in physics, applied mathematics, engineering etc. I also doubt that the definition you want to stick to, is the common definition of buoyancy applied in atmospheric physics in general.

    Anyway, to be able to apply the definition you want, you do not circumvent the fact that the net force you call buoyancy is the difference between force (m*a) caused by the gravity and the force defined by the integral of the pressure perpendicular to the surface over the outside surface of the volume in question.

    This is also explicitly stated by your citation:

    “As the pressure decreases with height so, in any parcel of air, the downwards pressure over the top of the parcel must be less than the upwards pressure under the bottom. Thus within the parcel there is a vertical component of the pressure gradient force acting upward.”

    This force is what is called the BUOYANCY FORCE in all other fields of science, technology and engineering than your little corner.

  256. You left out the important bit which is:

    “Generally this force is balanced by the gravitational force, so the net sum of forces is zero and the parcel floats in equilibrium.”

    That means no pushing is going on.

    Then:

    ” When the two forces do not quite balance, the difference is the buoyancy force. This is the upward or downward force exerted on a parcel of air arising from the density difference between the parcel and the surrounding air.”

    so the internal density difference is what pushes up or down and not pushing or pulling from outside.

    You have to research adiabatic uplift and descent because that shows that there is no work done by a rising or falling parcel on the surroundings and thus no pushing or pulling. I have set the reasoning out several times and referred you to authorities but all to no avail it seems.

  257. Kristian says:

    Stephen Wilde says, November 12, 2014 at 10:54 am:

    “A separate discrete force in the form of a sandbag was added to the side of the balloon to prevent spontaneous uplift and then discarded.”

    How specifically is this ‘spontaneous uplift’ of yours working? The air somehow becomes lighter/less dense, and it automatically floats up. Why? How? We all want to know.

  258. Trick says:

    Stephen 10:54am: Yes a sandbag external force could be added. You are making progress.

  259. Kristian says:

    Stephen Wilde says, November 12, 2014 at 1:36 pm:

    “” When the two forces do not quite balance, the difference is the buoyancy force. This is the upward or downward force exerted on a parcel of air arising from the density difference between the parcel and the surrounding air.”

    so the internal density difference is what pushes up or down and not pushing or pulling from outside.”

    Seriously, can’t you read, Stephen? Here is what it says: “This [buoyancy] is the upward or downward force exerted ON a parcel of air … arising from the density difference between the parcel and the surrounding air.”

  260. Kristian says:

    Christopher says, November 12, 2014 at 1:16 pm:

    “The definition on buoyancy applied in the above citation is NOT in agreement with the definition used generally in physics, applied mathematics, engineering etc. I also doubt that the definition you want to stick to, is the common definition of buoyancy applied in atmospheric physics in general.

    Anyway, to be able to apply the definition you want, you do not circumvent the fact that the net force you call buoyancy is the difference between force (m*a) caused by the gravity and the force defined by the integral of the pressure perpendicular to the surface over the outside surface of the volume in question.

    This is also explicitly stated by your citation:

    “As the pressure decreases with height so, in any parcel of air, the downwards pressure over the top of the parcel must be less than the upwards pressure under the bottom. Thus within the parcel there is a vertical component of the pressure gradient force acting upward.”

    This force is what is called the BUOYANCY FORCE in all other fields of science, technology and engineering than your little corner.”

    That’s actually not true. There is in fact some slight confusion about this. Some textbook sources give what you (and me, normally) would call ‘the buoyancy force’ the name ‘the pressure gradient force’. This would be the natural upward net force at hydrostatic equilibrium, meaning there is no uplift or subsidence. The buoyancy, according to these sources, arises only when there is an imbalance between the force of gravity and the pressure gradient force.

    This is, however, pure semantics. The phenomenon itself is exactly the same in both cases. Some people simply prefer to separate between the static case and the case where something is actually happening. This also probably agrees better with the everyday interpretation of the term ‘buoyancy’.

  261. wayne says:

    Much confusion Kristian. I found it a number places on the net. Semantically ‘buoyancy force’ and ‘buoyancy’ and whether something is ‘buoyant’ or not. The later two may have some semantic leeway possibly but the former is tied to equations and needs to be addressed.

    I haven’t been to a Civil Air Patrol meeting at the base since August but some will take it a little more serious that is being tossed about here and besides would a great learning moment for the new young cadets in critical thinking, physics and why you should not just accept things on the internet so easily and also how important definitions can be with their ties to equations.

    But this has nothing to do with Stephen’s problem. His is a separate issue for he cannot even see that if you have a basketball or a blimp full of hydrogen that the force is there always, whether it floats or not. He thinks the blimp somehow powers itself upward and lifts itself. As you shave off mass you will reach the the point of hydrostatic equilibrium and removing even more mass it will accelerate upward irregardless of anything but the mass ‘m’.

    I’ll be writing some letters to a few people to first get some more info. Don’t want anyone or property possibly hurt from a simple misunderstanding. you can’t force some .com site to correct itself but I think most will if this matter is brought to their attention.

  262. wayne says:

    “Generally” Stephen, they say generally and that queues myself that words that follow are to mean, but not strictly. It says “so the net sum of forces”, what forces? To me it is the downward force of weight and the upward force of the ‘buoyancy force’. What two forces ‘F’ camcel for you?

    “When the two forces do not quite balance, the difference is the buoyancy force.”, I’ll take issue with that statement, I don’t think that is correct. If so yiou are then speaking of three forces, the two that cancel and small one that is really the difference between the other original two as if the original two disappear at hydrostatic equilibrium.

  263. Christopher says:

    I agree that it seems to be much confusion about the use of “buoyancy” and “buoyancy force” around.

    Therefore it is important to establish precise definitions of what is actually meant if this discussion is going to make any further progress.

    I think I have given precise definitions of what I call “buoyancy force” and “gravity force” in the present context. The definition of “buoyancy force” is supported by at least one “authoritative” reference (I can easily come up with several if required) and the definition is commonly applied in fluid dynamics. Whether this force is called “buoyancy force” or “pressure gradient force” or any other convenient name is not important as long as it is well defined. The definition of “gravity force” seems to be accepted by everyone in this thread. Then the net effect of the “buoyancy force” and the “gravity force” should also be well defined.

    The definitions I have provided are therefore well suited as basis for further discussion of what happens when an “air parcel” in the atmosphere close to the earth surface is heated.

    I have no problem in accepting other definitions, but then these definitions must be precisely stated. So far I have not seen such alternative definitions. Normally I would also say that if it is not possible to formulate the definitions mathematically, they are not sufficiently precise.

  264. wayne says: November 12, 2014 at 5:39 pm

    “Much confusion Kristian. I found it a number places on the net. Semantically ‘buoyancy force’ and ‘buoyancy’ and whether something is ‘buoyant’ or not. The later two may have some semantic leeway possibly but the former is tied to equations and needs to be addressed.”

    Wayne,
    That force,in a compressible fluid or not, in a gravitational field, is always exactly equal to the difference in weight between a more heavy volume the “displacor” of a volume and the lighter equal volume, “displacee”! I fail to see where this has any bearing on convection, thermals, rising, falling, or whether some “process” is adiabatic, diabatic, or a combination of both.
    Any force resulting in movement of mass over distance is work. Work is generally diabatic and creates entropy. Adiabatic compression, a process, within a rigid adiabatic barrier, is reversible and isentropic. Work is done by a force over distance, but no entropy is generated, and all initial conditions are returned to, upon removal of that force. This atmosphere could do that only if the gravitational force is removed. DO NOT TRY THIS WITHOUT SAFETY GLASSES! There is no corresponding “process” that occurs in this atmosphere.
    Thermostatically, all lateral temperatures are equal, vertically there is both pressure and temperature gradient potentials exactly opposed by the gravitational potential. This only limits a spontaneous transfer of sensible heat or baric energy. If the atmosphere were He, the temperature gradient would oppose the pressure gradient, but both would still be opposed by gravitational potential.

    Back to the original small sceptical voice:

    “The Modtran model successfully predicts the local temperature throughout the troposphere, but how do we determine the extent to which the level of radiative activity at a particular altitude is the effect of the local temperature at that altitude rather than the cause of it?”

    This is nonsense! The Modtran model predicts no temperature at any altitude based on radiative activity. The atmospheric temperature profile is a required input to the model, if none is provided, an internal standard profile is used. This internal standard has been updated many times to match that of current radiosonde averages.
    If proper latitudinal,diurnal and seasonal, temperatures and pressures are used, proper evaluation using the HiTran database can provide much understanding of the actual radiative interaction with this atmosphere. Such effort has never been attempted, and any attempt will be fraught with errors, from lack of understanding of what that database contains.

    “If my reasoning is correct, then how can we resolve the issue of the degree to which the mass of the atmosphere rather than its optical depth is the cause of the warmth of the Earth’s surface compared to that of the Moon?”

    This can only be done by first considering how the atmospheric lapse rate at each location is created and incrementally modified by all local heat transfer methods, including the generation and transfer of radiative flux. Any averages can produce only gross errors. None of the models come anywhere close to the needed requirements for such a resolution! The current combined knowledge of this atmosphere is still negative! It is fantasy. With much effort, and observation, we may come all the way up to knowing nothing. Guess why this negative knowledge is defended and promoted? Consider the money involved!

  265. wayne says:

    Will:

    Wayne,

    That force,in a compressible fluid or not, in a gravitational field, is always exactly equal to the difference in weight between a more heavy volume the “displacor” of a volume and the lighter equal volume, “displacee”! I fail to see where this has any bearing on convection, thermals, rising, falling, or whether some “process” is adiabatic, diabatic, or a combination of both.

    Any force resulting in movement of mass over distance is work.

    Oh, geesh! You are so very close to all I have written above. Really. You just haven’t quite ties them all together as I do yet. I have nothing to say about your that snippet of comment to me except it is correct, right on!

    But a little further down you say “Thermostatically, all lateral temperatures are equal”, ok, generally but not critically, think Will, it is density that is always laterally precisely equal, pressure and temperature may be off a bit because ρ = P/(Rs T) and Rs is a constant. This is why the talk of buoyancy is critical. That is one huge factor as to why the profile of an atmosphere is what it is. On the vertical sorted gradiaents that is strictly density’s role.

    An atmoshere is always attempting (force) to have level pycnal surfaces in water or air (http://www.lbranch.bajaru.com/ap-environmental-science/ap-environmental-science/apes–biomes-november-11/ppt-45–salinity–pycnal/) of equal densitites or buoyancy spontaneously acts to correct the misbalance. Does that help why we are speaking of what we (or me anyway) are speaking?

    Also, on “Any force resulting in movement of mass over distance is work”, yes, and that work is energy, right? F·s . So gravity has zero problems moving the surroundings (denser) downward a very tiny amounts since surrounding’s volume is so huge compared to any small thermal and accelerate that thermal quite fast upward (conservation of mass). But it took other energy to make the thermal less dense in the first place and that is all THAT energy (solar) does, makes that mass of air warmer and less dense and buoyancy is right there to move it up to where it’s density belongs in the density sorted atmopshere. If that warm mass were to ever lose that energy that is making it less dense, of course, it would get denser and would stop or even fall again. The warm air does eventually lose that excess energy high above and that air mass just become a part of the massive surroundings because it is then at it’s proper density surface where it belongs. Make sense?

    On tallbloke’s original question, I’ll try to move back that direction but now carrying what many here seem to agree on.

  266. Ben Wouters says:

    Stephen Wilde says: November 12, 2014 at 1:36 pm

    ““Generally this force is balanced by the gravitational force, so the net sum of forces is zero and the parcel floats in equilibrium.”

    That means no pushing is going on.”
    There is a lot of pushing going on in the atmosphere. If we could switch off gravity instantaneously, the atmosphere would explode into space. Its internal pressure is what keeps it from collapsing to the surface by gravity pulling it down.

  267. Kristian says:

    Ben Wouters says, November 13, 2014 at 2:38 am:

    “There is a lot of pushing going on in the atmosphere. If we could switch off gravity instantaneously, the atmosphere would explode into space. Its internal pressure is what keeps it from collapsing to the surface by gravity pulling it down.”

    Ben, at some point I think you used the term ‘thermal pressure’ instead of ‘internal pressure’ for the atmosphere.

    I think you should stick to that. Because ‘internal pressure’ implies a pressure against an external pressure. The atmosphere taken as a whole is free to expand against no external pressure. There is no pressure from space holding it back. There is only gravity.

    In stellar physics, hydrostatic balance is considered to be maintained simply by the countering effects of the outward ‘thermal pressure’ and the inward ‘gravitational pull’ of the star in question.

    I think such a balance using such terms is more ‘intuitively’ understood and hence more practical concerning the general progression of this discussion.

    Do you agree?

  268. Christopher says:

    Will Janoschka says:
    November 13, 2014 at 12:20 am

    “Back to the original small sceptical voice:

    “The Modtran model successfully predicts the local temperature throughout the troposphere, but how do we determine the extent to which the level of radiative activity at a particular altitude is the effect of the local temperature at that altitude rather than the cause of it?”

    This is nonsense! The Modtran model predicts no temperature at any altitude based on radiative activity. The atmospheric temperature profile is a required input to the model, if none is provided, an internal standard profile is used. This internal standard has been updated many times to match that of current radiosonde averages.
    If proper latitudinal,diurnal and seasonal, temperatures and pressures are used, proper evaluation using the HiTran database can provide much understanding of the actual radiative interaction with this atmosphere. Such effort has never been attempted, and any attempt will be fraught with errors, from lack of understanding of what that database contains.”

    Interesting! Having a scientific background within dynamic systems (structural, hydro and aero) both as developer of FEA software and analysis and interpretation of dynamic systems response (multichannel time-series analysis) I would like to dig into the inner core of this Modtran thing. Do you know of a reference or two that describes the physics and math of this piece of software? I did a quick search on the web, but finds mainly color brochure like descriptions seemingly telling you a lot, but mainly telling you nothing.

  269. Ben Wouters says:

    Kristian says: November 13, 2014 at 6:56 am

    ” I think such a balance using such terms is more ‘intuitively’ understood and hence more practical concerning the general progression of this discussion.

    Do you agree?”

    Seems like semantics to me, but if it can speed up the discussion beyond the present meteo101
    level fine.

  270. wayne says: November 13, 2014 at 1:47 am
    Will: ……

    “Oh, geesh! You are so very close to all I have written above. Really. You just haven’t quite ties them all together as I do yet. I have nothing to say about your that snippet of comment to me except it is correct, right on!”

    I am not sure I wish to tie them all together “as” you do! Words, words, words, Which is correct for your intended purpose! Promote understanding, Create fear, promote political agenda, be a tr0ll? Not a question for you Wayne, but to all! What is your purpose, who is your audience, which word is most effective on your audience for your purpose. If you can be straightforward with your purpose and intended audience (may be “just an aside comment to all”). All will know whether to read on.
    Wayne, I agree with most of what you write, but sometimes it requires yet another beer, before I agree!

    “But a little further down you say “Thermostatically, all lateral temperatures are equal”, ok, generally but not critically, think Will, it is density that is always laterally precisely equal, pressure and temperature may be off a bit because ρ = P/(Rs T) and Rs is a constant.”

    OK I should have written “In one thermostatic state”, then explained that “a thermostatic state” is one where no temperatures or fluxes (fluxi) are varying, whether spontaneous or not! Thermodynamically, of this atmosphere earthlings will go no further in misunderstanding.

    “This is why the talk of buoyancy is critical. That is one huge factor as to why the profile of an atmosphere is what it is. On the vertical sorted gradients that is strictly density’s role.”

    Only in a compressible fluid! Tugboat captains have no use for density. Airship pilots likely consider it even in dreams. Submariners worry about thermoclines and salinity, but seldom in terms of mass/volume ratio.

    “An atmoshere is always attempting (force) to have level pycnal surfaces in water or air (http://www.lbranch.bajaru.com/ap-environmental-science/ap-environmental-science/apes–biomes-november-11/ppt-45–salinity–pycnal/) of equal densities or buoyancy spontaneously acts to correct the misbalance. Does that help why we are speaking of what we (or me anyway) are speaking?”

    It appears that everything this planet does is spontaneous action to correct imbalance whether provoked by Solar irradiance, gravitational effects of “nearby” masses, or all effalumps leaning eastward, trying to stop the damned rotation. Spontaneous vs forced!
    However “pycnal”, no definition. isopycnal (Oceanography) Hyper-/hypo- “pycnal” maybe.
    Wayne, you are almost as hard over on “density” as Stephen is on “parcel”!! Even Glenfiddich gives no understanding.

    “Also, on “Any force resulting in movement of mass over distance is work”, yes, and that work is energy, right? F·s.”

    I can flip back and forth between the meaning of energy, and work, with the same units, as easily as I can flip the meaning of irradiance, and radiative flux, with the same units. The meanings are often in opposition. Just try to understand the lab technician screaming “alles ist kaput”, meaning “correctly”, the power went out again, and we have to start over.

    “If that warm mass were to ever lose that energy that is making it less dense, of course, it would get denser and would stop or even fall again. The warm air does eventually lose that excess energy high above and that air mass just become a part of the massive surroundings because it is then at it’s proper density surface where it belongs. Make sense?”

    Try using “sensible heat” instead of energy and warm. Then understanding that latent heat must, before precipitation, spontaneously convert to sensible heat, providing all power for thermal EMR only in the direction of lower field strength! Only after all that happens, can your density increase.

  271. Ben Wouters says: November 13, 2014 at 9:44 am
    Kristian says: November 13, 2014 at 6:56 am
    (” I think such a balance using such terms is more ‘intuitively’ understood and hence more practical concerning the general progression of this discussion. Do you agree?”)

    “Seems like semantics to me, but if it can speed up the discussion beyond the present meteo101
    level fine.”

    Ben, It is all semantics, the use of written or spoken symbols, to convey your understanding to those with a different POV, promoting their understanding and agreement on symbols. At times the symbols used, can personally mean KING, or in the pot ’till you are tender.

  272. Christopher says: November 13, 2014 at 7:04 am
    Will Janoschka says: November 13, 2014 at 12:20 am (“Back to the original small sceptical voice:…”)

    “Do you know of a reference or two that describes the physics and math of this piece of software? I did a quick search on the web, but finds mainly color brochure like descriptions seemingly telling you a lot, but mainly telling you nothing.”

    I can help some, but most you can find on the Web, are attempts to sell what is free. All is in FORTRAN. Except for the database, LoTran, ModTran, HiTran, even FasTran software are only of historical interest.

    Harvard University is the current conservator of the whole 60’s-70’s AF Cambridge Labs program to determine the ability for atmospheric seeing at any wavelength interval through the whole mess of spectrally absorptive, dispersive gasses, aerosols, and other buoyant junk. The program was militarily twofold. Where can you see them, and where can they not see you.
    All is based on the measurement of attenuation of temporal, or spatial modulation of radiant flux in any waveband. Any use of that data “as” atmospheric attenuation of radiative flux “itself”, rather than attenuation of variance, at or near atmospheric radiative equilibrium, must Judicially be knowledgeable, deliberate, scientific fraud, for financial or political gain. This cannot be an oops! Way too many know!

  273. Ben Wouters says: November 13, 2014 at 2:38 am

    “There is a lot of pushing going on in the atmosphere. If we could switch off gravity instantaneously, the atmosphere would explode into space. Its internal pressure is what keeps it from collapsing to the surface by gravity pulling it down.”

    Ben, Just to pick nits, Would that not be a detonation, rather than an explosion? What is the speed of sound in this atmosphere if we switch off gravity instantaneously.? DO NOT TRY THIS WITHOUT SAFETY GLASSES! I like the phrase ” blow up in your face”.. switching off gravity within any interval is iffy. Who is measuring? Great way to confirm or deny gravity waves tho!

  274. Ben Wouters says:

    Will Janoschka says: November 13, 2014 at 4:37 pm

    ” Ben, Just to pick nits, Would that not be a detonation, rather than an explosion?”
    I don’t know and I don’t care. As long as its clear that the internal thermal pressure of our atmosphere keeps it ” in the air”.
    The atmosphere MUST contain thermal energy, every single molecule of it, otherwise it will be on the surface. This energy comes mostly from the solar heated surface.
    (clouds and ozone disturb this simple picture)
    As long as the energy loss from all levels is re-supplied by the sun we have a balanced situation.
    If we can find how this energy is distributed upwards, we have an explanation for the average lapse rate of ~ 6,5K/km for the total atmosphere.
    Lets concentrate on this iso re-inventing the well established mechanism for convection.
    Pse forget all nonsense about the DALR and SALR having anything to do with the temperature profile of the total atmosphere.

  275. Kristian says:

    Ben Wouters says, November 14, 2014 at 12:58 am:

    “Pse forget all nonsense about the DALR and SALR having anything to do with the temperature profile of the total atmosphere.”

    What do you mean by ‘total atmosphere’, Ben? If you actually mean the “total” atmosphere, from surface and all the way to its upper reaches into space, then this statement seems to me to be somewhat of a tautology, rather self-evident. Since an ‘adiabatic lapse rate’ would only arise where there is thorough mixing of air (that is, actual movement up and down the column), OF COURSE it wouldn’t have anything to do with the temperature profile above the tropopause.

    Inside the troposphere, however, the gravity-based ALR has EVERYTHING to do with the multitude of local/regional temperature profiles always varying through space and time, but ultimately adding up to the MEAN GLOBAL profile. How can one think otherwise? It’s about overall stability vs. no overall stability. Radiative-convective equilibrium.

  276. Ben Wouters says: November 14, 2014 at 12:58 am

    “The atmosphere MUST contain thermal energy, every single molecule of it, otherwise it will be on the surface. This energy comes mostly from the solar heated surface.”

    OK, The atmospheric volume is determined by the sensible heat only,(not internal energy which has all kinds of energy besides those concerning pressure, volume, and temperature). Your thermal energy is that energy that is a function of all three.

    “As long as the energy loss from all levels is re-supplied by the sun we have a balanced situation.”

    Whoa! (horse stop),
    I agree, but: Explain this energy loss from all levels! What is lost and how? Is it only lost as it is resupplied, at that location? What happens at night? Does the loss go away at night?

    “If we can find how this energy is distributed upwards, we have an explanation for the average lapse rate of ~ 6,5K/km for the total atmosphere.”

    OK, How does some average lapse rate have anything to do with is, right here, right now?

    “Lets concentrate on this iso re-inventing the well established mechanism for convection.”

    OK, add sensible heat to “a volume” of air. the volume increases to become “b volume” of air with the same weight as before the heat was added. A surrounding equal “volume b” with no additional sensible heat weighs more than the heated equal volume of air. the difference in weight is the force that lifts (displaces) the heated volume upward, sliding under that volume, replacing it. The new air, in turn has sensible heat added and repeats the process. Please understand that this is not a 1,2,3,4 process! All processes are continuous and proportional, until the geometry changes, (perhaps running out of sensible heat to add). “Volume a” and the larger “volume b” can be any volume, down to the molecule, and anywhere within this atmosphere.
    This whole convection is powered by only the sensible heat added. It creates interesting things for Wayne’ glider. How does the average value of – 6.5 degree Celsius per kilometer affect this process?
    It would be the same if the lapse rate were zero!

    My real question is that except for van der waals forces, at any temperature, each molecule has the same volume, but the WV molecule has much less weight. How does that affect the rate of the convection process? Does the WV molecule lift more rapidly? Why?

  277. Ben Wouters says:

    Kristian says: November 14, 2014 at 1:04 pm

    ” Inside the troposphere, however, the gravity-based ALR has EVERYTHING to do with the multitude of local/regional temperature profiles always varying through space and time, but ultimately adding up to the MEAN GLOBAL profile. How can one think otherwise?”
    Assuming that with gravity based ALR you mean the DALR (and SALR after condensation has started), these are ONLY relevant WITHIN a parcel that rises or sinks in the static atmosphere.
    A rising parcel cools adiabatically, so has no influence on the surrounding air. Once it has reached its density level it floats there ,having the same temperature as the surrounding air, again no change in the temperature of the static atmosphere. This is very basic meteorology.
    see eg http://www.recreationalflying.com/tutorials/meteorology/section1b.html#adiabatic_processes
    or
    http://www.tornadochaser.net/capeclass.html
    especially page 3.

  278. Ben Wouters says:

    Will Janoschka says: November 14, 2014 at 1:06 pm

    ” Explain this energy loss from all levels! What is lost and how?”
    Surface radiates partly direct to space (atmospheric window). Each level in the atmosphere radiates and some of that radiation may also go directly to space. This is apparently a pretty slow process, since the temps at altitude do not change nearly as fast as those near the surface.

    “It would be the same if the lapse rate were zero!”
    Shows you have no clue about how convection works. With an ELR of zero no convection is possible. The rising parcel cools at 9,8 K/km, so within a few meters it will be cooler than the surrounding air, and convections stops.

    ” My real question is that except for van der waals forces, at any temperature, each molecule has the same volume, but the WV molecule has much less weight. How does that affect the rate of the convection process? Does the WV molecule lift more rapidly? Why?”
    For whatever the reason air tends to move in coherent volumes, not as single molecules.
    A volume of air at 15C with eg 1000 molecules and relative humidity (RH) 0% has no wv molecules.
    At 100% RH it holds 10 wv molecules, that replace 10 other molecules (N2 or O2).
    This is the maximum ‘carrying capacity’ for air at 15C. Adding more wv (or cooling this air) will result in condensation, given enough nuclei to condense on.
    So air with wv is slightly more buoyant than air at 0% RH. This can show up as a slightly earlier release of a thermal, and/or a slightly higher ascend speed.

    “OK, How does some average lapse rate have anything to do with is, right here, right now?”
    The average ELR has evened out all disrupting processes that distribute energy over the globe.
    My position is that if we could have the whole surface at an equal temperature like 290K, the ELR would be close to this average 6,5K/km. So understanding this average ELR will give us clues as to how the energy transport from solar heated surface to space works.

  279. Kristian says: November 14, 2014 at 1:04 pm

    “Inside the troposphere, however, the gravity-based ALR has EVERYTHING to do with the multitude of local/regional temperature profiles always varying through space and time, but ultimately adding up to the MEAN GLOBAL profile. How can one think otherwise? It’s about overall stability vs. no overall stability. Radiative-convective equilibrium.”

    If one were ever to think, they must think your spouting is Climate Clown nonsense! Only Climate Clowns try to defend a mean global anything, that has absolutely nothing to do with the intensely non linear processes in this atmosphere.
    Gravity does establish a thermostatic “apparent rigid adiabatic barrier” at all points within the troposphere. This prevent “only” the expected spontaneous heat and baric diffusion in opposition to the gravitational potential difference. That adiabat plays no part in the interesting atmospheric thermodynamic processes, always tending toward equilibrium, but never reaching anything but radiative equilibrium. Your “Radiative-convective equilibrium.” is but another fantasy direct from the Climate Clown Choir Book.! Show even one location with thermodynamic stability? The mean of all un-stabilities, is still unstable, and still meaningless!

  280. Ben Wouters says: November 14, 2014 at 4:16 pm
    Will Janoschka says: November 14, 2014 at 1:06 pm

    (” Explain this energy loss from all levels! What is lost and how?”)
    “Surface radiates partly direct to space (atmospheric window). Each level in the atmosphere radiates and some of that radiation may also go directly to space. This is apparently a pretty slow process, since the temps at altitude do not change nearly as fast as those near the surface.”

    Thank you Ben, and I agree. Minor correction, and my only observations of this atmosphere, is that all thermal EMR flux, must be in a direction of lower EMR potential (lower temperature)
    Downward flux is Climate Clown nonsense, and is never required for understanding of how EMR energy transfer actually happens.

    (“It would be the same if the lapse rate were zero!”)
    “Shows you have no clue about how convection works. With an ELR of zero no convection is possible. The rising parcel cools at 9,8 K/km, so within a few meters it will be cooler than the surrounding air, and convections stops.”

    Here I respectfully disagree. For understanding, the mass of the air, sensible heat, volume, and temperature does not change from some lapse rate. That “volume b” slowly defuses some excess sensible heat to the lower temperature surrounds, shrinking in volume. Even with zero lapse rate
    the surrounds are at lower temperature than the rising volume with more sensible heat. As long as the volume “at any volume” has less weight than an equal surround volume, convection must continue.

    (” My real question is that except for van der waals forces, at any temperature, each molecule has the same volume, but the WV molecule has much less weight. How does that affect the rate of the convection process? Does the WV molecule lift more rapidly? Why?”)
    “For whatever the reason air tends to move in coherent volumes, not as single molecules.”

    OK I cannot argue with years of observation that I have not observed. What measurements have been made that demonstrate that N2, O2, CO2, Ar, and H2O convect at exactly the same velocity as a coherent limited volume, under all conditions?

    (“OK, How does some average lapse rate have anything to do with is, right here, right now?”)
    The average ELR has evened out all disrupting processes that distribute energy over the globe.

    “My position is that if we could have the whole surface at an equal temperature like 290K, the ELR would be close to this average 6,5K/km. So understanding this average ELR will give us clues as to how the energy transport from solar heated surface to space works.”

    OK, You are claiming that the average or mean, affects the variance, and promotes understanding.
    Your claim is as baffling to me as is hydrostatic balance in a compressive fluid. I throw some balls into the air and they disappear!

  281. Kristian says:

    Ben Wouters says, November 14, 2014 at 3:45 pm:

    “Assuming that with gravity based ALR you mean the DALR (and SALR after condensation has started), these are ONLY relevant WITHIN a parcel that rises or sinks in the static atmosphere.”

    No, they constitute the benchmark temperature falloff rate around which the actual temperature profile is forced to fluctuate through an always ongoing radiative-convective struggle (radiation working towards steepening the ELR, convection working towards bringing it back down).

    “A rising parcel cools adiabatically, so has no influence on the surrounding air.”

    Er, it expands into the surrounding air, so does work on it, thus transferring energy to it.

    “Once it has reached its density level it floats there ,having the same temperature as the surrounding air, again no change in the temperature of the static atmosphere. This is very basic meteorology.”

    Change in temperature? Why would there be a ‘change in temperature’? The ELR is never at any one time exactly like the DALR or the SALR at any particular place. It’s not the ‘same’ thing. However, the ELR is totally tied to the ALR (with or without condensation). It is not allowed to simply be whatever, on average – annually, globally. That’s what I’m saying. You seem to be saying it’s totally independent.

    Yes, the ALR is only set/determined by gravity and ‘specific heat’ (DALR), plus the release of latent heat in the atmospheric column (SALR). And the ELR is rather set/determined by the interaction of radiation and convection. But its mean radiative/convective equilibrium is directly and fundamentally constrained by the ALR.

  282. Kristian says: November 14, 2014 at 7:30 pm
    Ben Wouters says, November 14, 2014 at 3:45 pm:

    “Assuming that with gravity based ALR you mean the DALR (and SALR after condensation has started), these are ONLY relevant WITHIN a parcel that rises or sinks in the static atmosphere.”

    No, they constitute the benchmark temperature falloff rate around which the actual temperature profile is forced to fluctuate through an always ongoing radiative-convective struggle (radiation working towards steepening the ELR, convection working towards bringing it back down).

    “A rising parcel cools adiabatically, so has no influence on the surrounding air.”

    Er, it expands into the surrounding air, so does work on it, thus transferring energy to it.“

    Yes, the ALR is only set/determined by gravity and ‘specific heat’ (DALR), plus the release of latent heat in the atmospheric column (SALR). And the ELR is rather set/determined by the interaction of radiation and convection.”

    Thank you, You demonstrate that temperature at any altitude is dominated by convection and latent heat. Now using that temperature and emissivity of each atmospheric WV molecule we can calculate the total EMR exitance at that altitude.

  283. wayne says:

    Will, Kristian, Ben:

    I seem to have progressed a bit in the last few days and back to my very original comment on page #1:

    A small sceptical voice in the lukewarm wilderness

    and that once again has to do with just the global mean troposphere, no higher levels considered… yet.

    I always thought you could take all of the t.d./hydrostatic eq. equations *and* the kinetic theory equations and would end up with the same amount of thermal energy that keeps an atmosphere supported at its proper height and end up with the same answer. Seems I am not correct there. T.d.’s IGL says when speaking of one mole that PV=RT and if instead of one mole, if you take one unit volume you get this eq., P/ρ = Rs T. Then you have the h.s.eq. equations, log(P/P0)/log(T/T0)-k=0 and this equation with about sixteen related variables involved all works perfectly also, thoroughly checked. That relation when manipulated algebraically gives you the more familiar:

    P = P0(T/T0)^(k)

    22632.56=101325·(216.65/288.15)^(5.2558), that’s correct.

    T = T0(P/P0)^(1/k)

    216.65=288.15·(22632.56/101325)^(1/5.2558), that’s correct.

    ρ = ρ0(T/T0)^(k-1)

    0.363926=1.225·(216.65/288.15)^(5.2558-1), and that’s correct.

    and so on at any point in all of the tropos, there are more, and they all hold as expected anywhere on any tropo. Problem is, it is the ‘k’ value that’s the problem. So follow this.

    In forming the exact value for ‘k’ in each tropo, the k that ends up being something like this, and they all produce the same value for k, as (γ/γ-1) or (M·G / R·L) or (G / Rs·L) or (cp/Rs) and about five more including a possible D.O.F. that are all just algebraic manipulation of the same base values involved for mass, space, time and temperature for ‘k’ always ends up unitless as it should be.

    No problem so far. But my problem was this ‘k’ does not ends up correct for any of the four tropos if you use published values (and assumed lab measured values in containers) for the cp and cv. The cp’s are off by right at 3/2 in all of the four body’s cases or you could say there is some missing multiplier, not adiabatic ‘g/cp’ as some trivally assume. Drove me nutty trying to locate where this ‘3/2’ difference was occurring and more important ‘why’.

    Take just Earth since you all would be more familiar with the values here, you can look them up, just so far for simplicity I am going to use the US Std. Atm. values like the examples just above.

    Let’s calculate the Thermal Energy within some unit volume the Ben mentioned. This is the energy that keeps any atmosphere supported at a more or less constant height for a given temperature.

    By t.d., P·V is energy any at our surface at ASL you find 1.225/0.0289644=42.2933 moles in this control unit volume so:

    42.2933 mol · 8.31432 J/mol/K · 288.15 K = 101325 joules of supporting energy. Remember that.

    Now look at this same problem from the Kinetic Theory’s viewpoint. The rms velocity at 288.15 K is v=(3·R·T/M)^(1/2) so (3·8.31432·288.15/0.0289644)^(1/2) is 498.1396 m/s.

    Feed that into the calculation of the energy present in a unit volume from the kinetic side:
    3/2 · R · T · (ρ/M) = 3/2 · 8.31432 J/mol/K · 288.15 K · (1.225 kg / 0.0289644 kg/mol) = 151988 joules, not 101325 joules per IGL.
    What is the ratio ?? 151987.6/101325 = 1.5 or 3/2.

    THERE IT IS! That is what I could never answer. This in essence means not g/cp using cp=1005.7 for air but cp=1508.6 for air and now that ‘k’ perfectly reproduces the entire Std. Atm. profile. Venus now matches. Jupiter’s Galileo data matches. The very bottom 10 km of the Titan Huygen probe’s data seems to match but you have to leave that 3/2 in the equations.

    Seems t.d. is primarily in jars and containers with no convection considered but the kinetic theory has no such constraints, only velocities, and it give a contained-energy 150% higher in all of the examples I’ve checked so far.

    What that also seems to mean when speaking of g/cp, is not g/1005.7 = 9.75 K/km for the lapse but g/1508.6 = 6.5 K/km that is the literal adiabatic lapse where buoyancy spontaneously sorts air masses whose densities are out of order, heavier above lighter because of the energy contained that sets the respective densities.

    I am getting closer and closer to what just might need a descriptive paper to lay all of this out. Since I have never been stuck and limited in any one branch of science I tend to look at things from a different viewpoint than someone with a structured view drilled into their heads, one way to look at some subject. In some respects, all of them are off just a bit and are never looking at both sides of the coin at once.

    Please excuse and even correct any mis-wording as you read this. I’m not so good at communicating such deep subjects, Remember, I am not a professional engineer or scientist and many times lack the proper words or phrase.

    Hope you will take the time to understand that said above.

  284. wayne says: November 14, 2014 at 8:41 pm

    Will, Kristian, Ben:

    “I seem to have progressed a bit in the last few days and back to my very original comment on page #1: https://tallbloke.wordpress.com/2014/10/04/a-small-sceptical-voice-in-the-lukewarm-wilderness/comment-page-1/#comment-89985 and that once again has to do with just the global mean troposphere, no higher levels considered… yet. Wayne

    Great Wayne,
    You are hung up on the mean, that has no data, no information, or any understanding!
    All is contained within the variance, and especially in the outliers. Please try to not go backwards, ask your kitty cat!

    “Please excuse and even correct any mis-wording as you read this. I’m not so good at communicating such deep subjects, Remember, I am not a professional engineer or scientist and many times lack the proper words or phrase.”

    You are completely innocent of any mis-wording, your writing clearly demonstrates a concept, that is not yet accepted!

    Hope you will take the time to understand that said above.

    For understanding , you owe 1.75 liters of Glenfiddich!

  285. wayne says:

    Sorry fellows, I forgot to put in the KE per volume using the rms velocity to get the same value:

    KE/V = (1/2) · (n·M) · v²

    (1/2) · (42.2933 mol · 0.0289644 kg/mol) · (498.1396 m/s)² = 151988 joules/V

    That is from yet another direction and once again 150% of the IGL PV of 101325 using unit-control-volumes.

    So no wonder the scientists that formed the US Std. Atm. used 150% of 1005 J/kg/K for the cp.

  286. wayne says:

    Will, I don’t understand what *you* are looking for. You complain of averages and say there is no data but I’ve got hundreds of thousands of radiosonde soundings that ARE my data. I like to speak of climate of atmospheres which is just averaged bi-daily soundings so we (hopefully) will come to some agreement on what happens to our atmosphere over time.

    What I am showing is that all atmospheres dance to the same equations… there is no, zero, influence from some 0.0004 fraction of CO2, or of 0.0008 fraction of CO2. The tropospheres care less. Now in the stratosphere upward there are radiation influences, in fact radiation up above is what sets where the lapse profile is to the left (colder) or right (warmer) on all of the P Skew-T plots. It is convection and not merely warmer rising but also colder sinking that makes out 6.5 K/lm or Venus’s 7.7 K/m or Jupiter’s 2.2 K/km temperature/height gradients be what they are. The variables ruling any differences are ‘g’ gravity, ‘M’ average molecular mass, the gas constant, the cp and cv of that particular mixture of gases… no radiation included if you notice, not in the optical thick tropospheres at least while lapse gradients are linear.

    Say in some equations what you are envisioning and maybe I can see clearer what you are seeking. I’ve tried hard to make it perfectly clear what I am seeking.

  287. wayne says:

    Maybe I should also say I am not personally looking at thunderstorms or fast moving mixing layer thermals but more on the tune of convection that is changing height at meters per hour, very subtle and slow up in the tropospheres.

    Everyone has probably had the experience with helium balloons. Same principle, density differences. Once a day after a birthday my grandson had some that were losing their lightness as the helium leaked out. Those balloons came to just hydrostatic equilibrium the next day. You could raise them up by the ceiling and sit and watch them take ten minutes to very slowly sink to the floor… that kind of convection is what is in my mind as I look at radiosonde soundings. Some displacements cooler than normal or warmer than normal take five days to very slowly re-sort always seeking back to ‘the mean’.

    Maybe that will help you to see where I am coming from. ANY difference in temperature that causes density differences will, sometimes VERY slowly, move to make the lapse as constant as possible and that is the mean I am dealing with explaining what I seem to have stumbled upon. Probably someone has already addressed this very topic but I search and cannot find such papers or chapters in meteorology or atmospheric physics or planetary atmospheric books or papers. If so I wouldn’t be taking my time and would just point to it.

  288. wayne says: November 14, 2014 at 10:06 pm

    “Will, I don’t understand what *you* are looking for. You complain of averages and say there is no data but I’ve got hundreds of thousands of radiosonde soundings that ARE my data. I like to speak of climate of atmospheres which is just averaged bi-daily soundings so we (hopefully) will come to some agreement on what happens to our atmosphere over time.”

    Wayne, I have no problem with your consideration of the mean of anything. There must be some “stuff” in that aggregation !

    “What I am showing is that all atmospheres dance to the same equations… there is no, zero, influence from some 0.0004 fraction of CO2, or of 0.0008 fraction of CO2. The tropospheres care less. Now in the stratosphere upward there are radiation influences, in fact radiation up above is what sets where the lapse profile is to the left (colder) or right (warmer) on all of the P Skew-T plots. It is convection and not merely warmer rising but also colder sinking that makes out 6.5 K/lm or Venus’s 7.7 K/m or Jupiter’s 2.2 K/km temperature/height gradients be what they are. The variables ruling any differences are ‘g’ gravity, ‘M’ average molecular mass, the gas constant, the cp and cv of that particular mixture of gases… no radiation included if you notice, not in the optical thick tropospheres at least while lapse gradients are linear.”

    You are ignoring the effect of the atmospheric thermal EMR discard of entropy to space, at every altitude. You seem to merge that with lapse rate.

    “Say in some equations what you are envisioning and maybe I can see clearer what you are seeking. I’ve tried hard to make it perfectly clear what I am seeking.”:

    Wayne, I cannot, the radiative effects in this atmosphere, make the Navier Stokes partial differential equations seem trivial, but yet are totally dependent on them!

    I only wish to add my unexplainable measurements, to your efforts regarding “no one knows”.

  289. wayne says:

    I get you Will but the radiation you speak of is wrapped within the hydrostatic equilibrium that I speak of. To me I have suspicion that is exactly what the additional 500 joules per mole is representing. All matter radiates and yes, all of the troposphere is radiating outward from all levels so when I say it doesn’t matter of the radiation what I mean is that each of these troposphere all have that lw radiation leakage upward smoothly at all levels or you would be warps that never go away and you can see that not in our troposphere but in Venus’s profile. As the lower cloud levels are transversed there is a very slight warp in it’s profiles, all of them that my more idealistic and mathematically perfect view does not show. Mine is perfectly linear, the real one is warped at about 45-50 km by a few degrees. That is non-normal radiative caused right there.

    I use my viewpoint as a ruler to measure the instantaneous deviations with. No radiosonde sounding ever follows this ‘ruler’. There are always warps and pushed to the cooler or warmer side as you go from the surface to where that gradient fails to no longer be linear and that is exactly where radiation effects start to overpower what hydrostatics can perform of moving energy up and down the columns.

    Or that is what seems correct so far. All of the data and numbers seem to indicate that.

  290. wayne says:
    November 15, 2014 at 12:07 am

    “I get you Will but the radiation you speak of is wrapped within the hydrostatic equilibrium that I speak of. ”
    That radiative exitance has nothing to do with your hydrostatic equilibrium.
    The radiative exit flux has only to do with temperature and emissivity of WV molecules and opposing radiative potential from the slightly lower temperature WV molecules above!

    “To me I have suspicion that is exactly what the additional 500 joules per mole is representing. All matter radiates and yes, all of the troposphere is radiating outward from all levels so when I say it doesn’t matter of the radiation what I mean is that each of these troposphere all have that lw radiation leakage upward smoothly at all levels.”

    You claim radiation leakage upward, what nonsense. The outward radiative flux, at every level is the deliberate discard of entropy to a lower field strength”.

    “I use my viewpoint as a ruler to measure the instantaneous deviations with. No radiosonde sounding ever follows this ‘ruler’. There are always warps and pushed to the cooler or warmer side as you go from the surface to where that gradient fails to no longer be linear and that is exactly where radiation effects start to overpower what hydrostatics can perform of moving energy up and down the columns.”

    Wayne, your viewpoint, while comprehensive, ignores the radiative, and wishes to cover that by some mystical mechanical cloak.

  291. Ben Wouters says:

    Kristian says: November 14, 2014 at 7:30 pm

    I’m not engaging in another Stephen type of discussion.
    Your idea of the DALR and SALR is total nonsense.
    Study http://www.recreationalflying.com/tutorials/meteorology/index.html
    It is a very good base to gain knowledge of basic meteorology.

    See http://www.britannica.com/EBchecked/topic/330402/lapse-rate
    for a definition of the 2 kinds of lapse rates.

  292. Ben Wouters says:

    Will Janoschka says: November 14, 2014 at 6:12 pm

    ” With an ELR of zero no convection is possible. The rising parcel cools at 9,8 K/km, so within a few meters it will be cooler than the surrounding air, and convections stops.”

    Here I respectfully disagree”
    Fine, but you’re wrong. A rising parcel (lets follow eg 1m^3) rises into an ever DECREASING pressure environment. It continuously adjust its internal pressure to the surrounding pressure by EXPANDING. Expanding air COOLS. As long as no condensation occurs the cooling rate is 9,8K/km, the DALR.
    So in a zero ELR environment the rising air is in no time COLDER than the surrounding air and stops rising.
    Personally observed when circling inside a rising thermal. Close to the ground thermals are small, and you have to fly tight circles to stay within the thermal, and not enter the surrounding sinking air.
    Higher up the thermal is much larger and relaxed circling is possible.

    ” What measurements have been made that demonstrate that N2, O2, CO2, Ar, and H2O convect at exactly the same velocity as a coherent limited volume, under all conditions?”
    The very good match between calculated cloudbase and actual cloudbase is proof that the wv from the surface moves within the rising air and starts condensing at the calculated altitude.

  293. Ben Wouters says:

    wayne says: November 14, 2014 at 8:41 pm

    “Let’s calculate the Thermal Energy within some unit volume the Ben mentioned. This is the energy that keeps any atmosphere supported at a more or less constant height for a given temperature.”
    Perhaps time for a concept that may explain the ELR.
    If you calculate the thermal energy for our tropopause at various levels, we should see a decrease with altitude of this energy.
    Now calculate the potential energy for the same levels and we see an increase.
    How does this look when adding the two energies for each level in the tropopause?
    Should say something about the situation nature is trying to achieve, but never realizes.

  294. Ben Wouters says:

    wayne says: November 14, 2014 at 10:06 pm

    ” Now in the stratosphere upward there are radiation influences,”
    Unclear what you’re saying here. The temperature increase with altitude in the stratosphere is due to solar uv interacting with ozone. So the temperature decrease from the surface upwards is interrupted by this local heating effect.

  295. Kristian says:

    Ben Wouters says, November 15, 2014 at 3:13 am:

    “I’m not engaging in another Stephen type of discussion.
    Your idea of the DALR and SALR is total nonsense.
    Study http://www.recreationalflying.com/tutorials/meteorology/index.html
    It is a very good base to gain knowledge of basic meteorology.

    See http://www.britannica.com/EBchecked/topic/330402/lapse-rate
    for a definition of the 2 kinds of lapse rates.”

    You don’t even seem to get what I’m trying to say to you, Ben. I am not saying the two lapse rates ARE THE SAME THING. Read what I’m writing. I’m saying that the ELR is not independent from the ALR. It is totally bounded by it. This is common knowledge, basic meteorology. Frankly I’m surprised it’s managed to pass you by.

    http://www.st-andrews.ac.uk/~dib2/climate/lapserates.html
    http://eesc.columbia.edu/courses/ees/climate/lectures/atm_phys.html

    But I agree, let’s not continue this discussion. I have no desire to engage in any discussions with persons who don’t read what other people are writing before dismissing it with the use of ad hominems.

  296. Kristian says:

    wayne says, November 14, 2014 at 10:06 pm:

    “What I am showing is that all atmospheres dance to the same equations… there is no, zero, influence from some 0.0004 fraction of CO2, or of 0.0008 fraction of CO2. The tropospheres care less.”

    But this is fairly common knowledge, isn’t it? The atmospheric pressure and density gradients fall off in an approximate exponential fashion from surface up, thus leaving temperatures to drop rather linearly with altitude.

    Since the normal temperature profile of a troposphere (where bulk air is constantly moving up and down the atmospheric column) – the ELR – is fundamentally constrained by the adiabatic lapse rate (the ALR), the tight interaction between radiation always trying to steepen the ELR and convection always working to bring it back down, ensures that, in a steady state (dynamically stable) atmosphere, its mean value stays equal to the ALR. This is the ‘radiative-convective equilibrium’.

    This in turn means that there is nothing CO2 could ever do to warm the surface, because that would have to involve reducing the ELR.

    It can’t and won’t work.

  297. wayne says:

    Kristian,

    Thanks for giving that some thought. When you say “It can’t and won’t work.”, what is the ‘It’?

    Also you say “Since the normal temperature profile of a troposphere (where bulk air is constantly moving up and down the atmospheric column) – the ELR – is fundamentally constrained by the adiabatic lapse rate (the ALR), the tight interaction between radiation always trying to steepen the ELR and convection always working to bring it back down, ensures that, in a steady state (dynamically stable) atmosphere, its mean value stays equal to the ALR. This is the ‘radiative-convective equilibrium’.” and you seem to think all of the atmospheres have a different ALR and ELR and you say they are all at the ALR which to me just means you have never taken the time to search for the equations that all tropospheres operate under.

    Think of it this way. Lets say you want ‘one single equation’ that will allow you to draw a temperature/pressure graph (the profile) of any troposphere that matches all of the data mankind has gathered so far of numerous tropospheres and using only a small number of controling parameters. One stipulation — these parameters cannot have anythig to do with radiation. Now, if you could ever find such a single equation does that not say something right back to you?

    I did just that, and I didn’t think such an equation would be possible but I wanted to see where the problems would occur making this feat impossible and by looking at the mis-matches you could gather some insight into the reason why a h2/he troposhere operates differently that a co2/n2 troposphere or why a n2/ch4 troposphere operated differently than a n2/o2/ar troposphere.

    It just really surprised me that after a couple of years searching instead I did find such a single equation that lets you programically, using each planet or moons troposphere using each respective parameters (ie: cv, g, M, radius, one P/T point), draw any of them matching the data from probes and balloons. To me that shouts at you trying to tell you something — they are all in common.

  298. wayne says:

    Oh Kristian, also I found none of them operate under such an ‘ALR’. That ALR is well described (see: γ/(γ-1), gamma being the heat cap. ratio) countless places, books, wiki, articles but that is the one I found none of them match. That is exactly why I wrote here, more like 2/(γ-1), then you can match them all.